Harr CC Flashcards

1
Q

Which formula correctly describes the relationship between absorbance and %T ?

A. A = 2 – log %T
B. A = log 1/T
C. A = –log T
D. All of these options

A

D. All of these options

Absorbance is proportional to the inverse log of transmittance.
A = –log T = log 1/T
Multiplying the numerator and denominator by 100 gives:
A = log (100/100 X T)
100 X T = %T, substituting %T
for 100 X T gives:
A = log 100/%T
A = log 100 – log %T
A = 2.0 – log %T

For example, if %T = 10.0, then:
A = 2.0 – log 10.0
log 10.0 = 1.0
A = 2.0–1.0 = 1.0

How well did you know this?
1
Not at all
2
3
4
5
Perfectly
2
Q

A solution that has a transmittance of 1.0 %T would have an absorbance of:

A. 1.0
B. 2.0
C. 1%
D. 99%

A

B. 2.0

A = 2.0 – log %T
A = 2.0 – log 1.0
The log of 1.0 = 0
A = 2.0

How well did you know this?
1
Not at all
2
3
4
5
Perfectly
3
Q

In absorption spectrophotometry:

A. Absorbance is directly proportional to transmittance
B. Percent transmittance is directly proportional to concentration
C. Percent transmittance is directly proportional to the light path length
D. Absorbance is directly proportional to concentration

A

D. Absorbance is directly proportional to
concentration

How well did you know this?
1
Not at all
2
3
4
5
Perfectly
4
Q

Which wavelength would be absorbed strongly by a red-colored solution?

A. 450 nm
B. 585 nm
C. 600 nm
D. 650 nm

A

A. 450 nm

How well did you know this?
1
Not at all
2
3
4
5
Perfectly
5
Q

A green-colored solution would show highest transmittance at:

A. 475 nm
B. 525 nm
C. 585 nm
D. 620 nm

A

B. 525 nm

How well did you know this?
1
Not at all
2
3
4
5
Perfectly
6
Q

SITUATION: A technologist is performing an enzyme assay at 340 nm using a visible-range
spectrophotometer. After setting the wavelength and adjusting the readout to zero %T with the light path blocked, a cuvette with deionized water is inserted. With the light path fully open and the 100%T control at maximum, the instrument readout will not rise above 90%T. What is the most appropriate first course of action?

A. Replace the source lamp
B. Insert a wider cuvette into the light path
C. Measure the voltage across the lamp terminals
D. Replace the instrument fuse

A

A. Replace the source lamp

How well did you know this?
1
Not at all
2
3
4
5
Perfectly
7
Q

Which type of monochromator produces the purest monochromatic light in the UV range?

A. A diffraction grating and a fixed exit slit
B. A sharp cutoff filter and a variable exit slit
C. Interference filters and a variable exit slit
D. A prism and a variable exit slit

A

D. A prism and a variable exit slit

How well did you know this?
1
Not at all
2
3
4
5
Perfectly
8
Q

Which monochromator specification is required in order to measure the true absorbance of a
compound having a natural absorption bandwidth of 30 nm?

A. 50-nm bandpass
B. 25-nm bandpass
C. 15-nm bandpass
D. 5-nm bandpass

A

D. 5-nm bandpass

How well did you know this?
1
Not at all
2
3
4
5
Perfectly
9
Q

Which photodetector is most sensitive to low levels of light?

A. Barrier layer cell
B. Photodiode
C. Diode array
D. Photomultiplier tube

A

D. Photomultiplier tube

How well did you know this?
1
Not at all
2
3
4
5
Perfectly
10
Q

Which condition is a common cause of stray light?

A. Unstable source lamp voltage
B. Improper wavelength calibration
C. Dispersion from second-order spectra
D. Misaligned source lamp

A

C. Dispersion from second-order spectra

How well did you know this?
1
Not at all
2
3
4
5
Perfectly
11
Q

A linearity study is performed on a visible spectrophotometer at 650 nm and the following
absorbance readings are obtained:

Concentration of Standard Absorbance
10.0 mg/dL 0.20
20.0 mg/dL 0.41
30.0 mg/dL 0.62
40.0 mg/dL 0.79
50.0 mg/dL 0.92

The study was repeated using freshly prepared standards and reagents, but results were identical
to those shown. What is the most likely cause of these results?

A. Wrong wavelength used
B. Insufficient chromophore concentration
C. Matrix interference
D. Stray light

A

D. Stray light

How well did you know this?
1
Not at all
2
3
4
5
Perfectly
12
Q

Which type of filter is best for measuring stray light?

A. Wratten
B. Didymium
C. Sharp cutoff
D. Neutral density

A

C. Sharp cutoff

How well did you know this?
1
Not at all
2
3
4
5
Perfectly
13
Q

Which of the following materials is best suited for verifying the wavelength calibration of a spectrophotometer?

A. Neutral density filters
B. Potassium dichromate solutions traceable to the
National Bureau of Standards reference
C. Wratten filters
D. Holmium oxide glass

A

D. Holmium oxide glass

How well did you know this?
1
Not at all
2
3
4
5
Perfectly
14
Q

Why do many optical systems in chemistry analyzers utilize a reference light path?

A. To increase the sensitivity of the measurement
B. To minimize error caused by source lamp fluctuation
C. To obviate the need for wavelength adjustment
D. To reduce stray light effects

A

B. To minimize error caused by source lamp fluctuation

How well did you know this?
1
Not at all
2
3
4
5
Perfectly
15
Q

Which component is required in a spectrophotometer in order to produce a spectral absorbance curve?

A. Multiple monochromators
B. A reference optical beam
C. Photodiode array
D. Laser light source

A

C. Photodiode array

How well did you know this?
1
Not at all
2
3
4
5
Perfectly
16
Q

The half-band width of a monochromator is defined by:

A. The range of wavelengths passed at 50% maximum transmittance
B. One-half the lowest wavelength of optical purity
C. The wavelength of peak transmittance
D. One-half the wavelength of peak absorbance

A

A. The range of wavelengths passed at 50% maximum transmittance

How well did you know this?
1
Not at all
2
3
4
5
Perfectly
17
Q

The reagent blank corrects for absorbance caused by:

A. The color of reagents
B. Sample turbidity
C. Bilirubin and hemolysis
D. All of these options

A

A. The color of reagents

How well did you know this?
1
Not at all
2
3
4
5
Perfectly
18
Q

A plasma sample is hemolyzed and turbid. What is required to perform a sample blank in order
to correct the measurement for the intrinsic absorbance of the sample when performing a
spectrophotometric assay?

A. Substitute deionized water for the sample
B. Dilute the sample 1:2 with a standard of known concentration
C. Substitute saline for the reagent
D. Use a larger volume of the sample

A

C. Substitute saline for the reagent

How well did you know this?
1
Not at all
2
3
4
5
Perfectly
19
Q

Which instrument requires a highly regulated DC power supply?

A. A spectrophotometer with a barrier layer cell
B. A colorimeter with multilayer interference filters
C. A spectrophotometer with a photomultiplier tube
D. A densitometer with a photodiode detector

A

C. A spectrophotometer with a photomultiplier tube

How well did you know this?
1
Not at all
2
3
4
5
Perfectly
20
Q

Which statement regarding reflectometry is true?

A. The relation between reflectance density and concentration is linear
B. Single-point calibration can be used to determine concentration
C. 100% reflectance is set with an opaque film called a white reference
D. The diode array is the photodetector of choice

A

C. 100% reflectance is set with an opaque film called a white reference

How well did you know this?
1
Not at all
2
3
4
5
Perfectly
21
Q

Bichromatic measurement of absorbance can correct for interfering substances if:

A. The contribution of the interferent to absorbance is the same at both wavelengths
B. Both wavelengths pass through the sample simultaneously
C. The side band is a harmonic of the primary wavelength
D. The chromogen has the same absorbance at both wavelengths

A

A. The contribution of the interferent to absorbance is the same at both wavelengths

How well did you know this?
1
Not at all
2
3
4
5
Perfectly
22
Q

Which instrument requires a primary andsecondary monochromator?

A. Spectrophotometer
B. Atomic absorption spectrophotometer
C. Fluorometer
D. Nephelometer

A

C. Fluorometer

How well did you know this?
1
Not at all
2
3
4
5
Perfectly
23
Q

Which of the following statements about fluorometry is accurate?

A. Fluorometry is less sensitive than spectrophotometry
B. Fluorometry is less specific than spectrophotometry
C. Unsaturated cyclic molecules are often fluorescent
D. Fluorescence is directly proportional to temperature

A

C. Unsaturated cyclic molecules are often fluorescent

How well did you know this?
1
Not at all
2
3
4
5
Perfectly
24
Q

Which of the following components is not needed in a chemiluminescent immunoassay analyzer?

A. Source lamp
B. Monochromator
C. Photodetector
D. Wash station

A

A. Source lamp

How well did you know this?
1
Not at all
2
3
4
5
Perfectly
25
Q

Which substance is used to generate the light signal in electrochemiluminescence?

A. Acridinium
B. Luminol
C. Dioxetane phosphate
D. Ruthenium

A

D. Ruthenium

How well did you know this?
1
Not at all
2
3
4
5
Perfectly
26
Q

Light scattering when the wavelength is greater than 10 times the particle diameter is described by:

A. Rayleigh’s law
B. The Beer–Lambert law
C. Mie’s law
D. The Rayleigh–Debye law

A

A. Rayleigh’s law

How well did you know this?
1
Not at all
2
3
4
5
Perfectly
27
Q

Which statement regarding nephelometry is true?

A. Nephelometry is less sensitive than absorption spectrophotometry
B. Nephelometry follows Beer’s law
C. The optical design is identical to a turbidimeter except that a HeNe laser light source is used
D. The detector response is directly proportional to concentration

A

D. The detector response is directly proportional to concentration

How well did you know this?
1
Not at all
2
3
4
5
Perfectly
28
Q

The purpose of the nebulizer in an atomic absorption spectrophotometer that uses a flame is to:

A. Convert ions to atoms
B. Cause ejection of an outer shell electron
C. Reduce evaporation of the sample
D. Burn off organic impurities

A

A. Convert ions to atoms

How well did you know this?
1
Not at all
2
3
4
5
Perfectly
29
Q

A flameless atomic absorption spectrophotometer dehydrates and atomizes a sample using:

A. A graphite capillary furnace
B. An electron gun
C. A thermoelectric semiconductor
D. A thermospray platform

A

A. A graphite capillary furnace

How well did you know this?
1
Not at all
2
3
4
5
Perfectly
30
Q

When measuring lead in whole blood using atomic absorption spectrophotometry, what reagent is
required to obtain the needed sensitivity and precision?

A. Lanthanum
B. Lithium
C. Triton X-100
D. Chloride

A

C. Triton X-100

How well did you know this?
1
Not at all
2
3
4
5
Perfectly
31
Q

Interference in atomic absorption spectrophotometry caused by differences in viscosity is called:

A. Absorption interference
B. Matrix effect
C. Ionization interference
D. Quenching

A

B. Matrix effect

How well did you know this?
1
Not at all
2
3
4
5
Perfectly
32
Q

All of the following are required when measuring magnesium by atomic absorption spectrophotometry except:

A. A hollow cathode lamp with a magnesium cathode
B. A chopper to prevent optical interference from magnesium emission
C. A monochromator to isolate the magnesium emission line at 285 nm
D. A 285-nm reference beam to correct for background absorption

A

D. A 285-nm reference beam to correct for background absorption

How well did you know this?
1
Not at all
2
3
4
5
Perfectly
33
Q

When measuring calcium by atomic absorption spectrophotometry, which is required?

A. An organic extraction reagent to deconjugate calcium from protein
B. An internal standard
C. A magnesium chelator
D. Lanthanum oxide to chelate phosphates

A

D. Lanthanum oxide to chelate phosphates

How well did you know this?
1
Not at all
2
3
4
5
Perfectly
34
Q

Ion selective analyzers using undiluted samples have what advantage over analyzers that use a
diluted sample?

A. Can measure over a wider range of concentration
B. Are not subject to pseudohyponatremia caused by high lipids
C. Do not require temperature equilibration
D. Require less maintenance

A

B. Are not subject to pseudohyponatremia caused
by high lipids

How well did you know this?
1
Not at all
2
3
4
5
Perfectly
35
Q

Select the equation describing the potential that develops at the surface of an ion-selective
electrode.

A. van Deemter equation
B. van Slyke equation
C. Nernst equation
D. Henderson–Hasselbalch equation

A

C. Nernst equation

How well did you know this?
1
Not at all
2
3
4
5
Perfectly
36
Q

The reference potential of a silver–silver chloride electrode is determined by the:

A. Concentration of the potassium chloride filling solution
B. Surface area of the electrode
C. Activity of total anion in the paste covering the electrode
D. The concentration of silver in the paste covering the electrode

A

A. Concentration of the potassium chloride filling
solution

How well did you know this?
1
Not at all
2
3
4
5
Perfectly
37
Q

The term RT/nF in the Nernst equation defines the:

A. Potential at the ion-selective membrane
B. Slope of the electrode
C. Decomposition potential
D. Isopotential point of the electrode

A

B. Slope of the electrode

How well did you know this?
1
Not at all
2
3
4
5
Perfectly
38
Q

The ion-selective membrane used to measure potassium is made of:

A. High-borosilicate glass membrane
B. Polyvinyl chloride dioctylphenyl phosphonate ion exchanger
C. Valinomycin gel
D. Calomel

A

C. Valinomycin gel

How well did you know this?
1
Not at all
2
3
4
5
Perfectly
39
Q

The response of a sodium electrode to a 10-fold increase in sodium concentration should be:

A. A 10-fold drop in potential
B. An increase in potential of approximately 60 mV
C. An increase in potential of approximately 10 mV
D. A decrease in potential of approximately 10 mV

A

B. An increase in potential of approximately 60 mV

How well did you know this?
1
Not at all
2
3
4
5
Perfectly
40
Q

Which of the electrodes below is a current-producing (amperometric) rather than a
voltage-producing (potentiometric) electrode?

A. Clark electrode
B. Severinghaus electrode
C. pH electrode
D. Ionized calcium electrode

A

A. Clark electrode

How well did you know this?
1
Not at all
2
3
4
5
Perfectly
41
Q

Which of the following would cause a “response” error from an ion-selective electrode for sodium
when measuring serum but not the calibrator?

A. Interference from other electrolytes
B. Protein coating the ion-selective membrane
C. An overrange in sodium concentration
D. Protein binding to sodium ions

A

B. Protein coating the ion-selective membrane

How well did you know this?
1
Not at all
2
3
4
5
Perfectly
42
Q

In polarography, the voltage needed to cause depolarization of the cathode is called the:

A. Half-wave potential
B. Isopotential point
C. Decomposition potential
D. Polarization potential

A

C. Decomposition potential

How well did you know this?
1
Not at all
2
3
4
5
Perfectly
43
Q

Persistent noise from an ion-selective electrode is most often caused by:

A. Contamination of sample
B. Blocked junction at the salt bridge
C. Overrange from high concentration
D. Improper calibration

A

B. Blocked junction at the salt bridge

How well did you know this?
1
Not at all
2
3
4
5
Perfectly
44
Q

Which element is reduced at the cathode of a Clark polarographic electrode?

A. Silver
B. Oxygen
C. Chloride
D. Potassium

A

B. Oxygen

How well did you know this?
1
Not at all
2
3
4
5
Perfectly
45
Q

Which of the following statements accurately characterizes the coulometric titration of chloride?

A. The indicator electrodes generate voltage
B. Constant current must be present across the generator electrodes
C. Silver ions are formed at the generator cathode
D. Chloride concentration is inversely proportional to titration time

A

B. Constant current must be present across the
generator electrodes

How well did you know this?
1
Not at all
2
3
4
5
Perfectly
46
Q

In the coulometric chloride titration:

A. Acetic acid in the titrating solution furnishes the counter ion for reduction
B. The endpoint is detected by amperometry
C. The titrating reagent contains a phosphate buffer to keep pH constant
D. Nitric acid (HNO3) is used to lower the solubility of AgCl

A

B. The endpoint is detected by amperometry

How well did you know this?
1
Not at all
2
3
4
5
Perfectly
47
Q

Which of the following compounds can interfere with the coulometric chloride assay?

A. Bromide
B. Ascorbate
C. Acetoacetate
D. Nitrate

A

A. Bromide

How well did you know this?
1
Not at all
2
3
4
5
Perfectly
48
Q

All of the following compounds contribute to the osmolality of plasma except:

A. Lipids
B. Creatinine
C. Drug metabolites
D. Glucose

A

A. Lipids

How well did you know this?
1
Not at all
2
3
4
5
Perfectly
49
Q

One mole per kilogram H2O of any solute will cause all of the following except:

A. Lower the freezing point by 1.86°C
B. Raise vapor pressure by 0.3 mm Hg
C. Raise the boiling point by 0.52°C
D. Raise osmotic pressure by 22.4 atm

A

B. Raise vapor pressure by 0.3 mm Hg

How well did you know this?
1
Not at all
2
3
4
5
Perfectly
50
Q

What component of a freezing point osmometer measures the sample temperature?

A. Thermistor
B. Thermocouple
C. Capacitor
D. Electrode

A

A. Thermistor

How well did you know this?
1
Not at all
2
3
4
5
Perfectly
51
Q

What type of measuring circuit is used in a freezing point osmometer?

A. Electrometer
B. Potentiometer
C. Wheatstone bridge
D. Thermal conductivity bridge

A

C. Wheatstone bridge

How well did you know this?
1
Not at all
2
3
4
5
Perfectly
52
Q

Which measurement principle is employed in a vapor pressure osmometer?

A. Seebeck
B. Peltier
C. Hayden
D. Darlington

A

A. Seebeck

How well did you know this?
1
Not at all
2
3
4
5
Perfectly
53
Q

The freezing point osmometer differs from the vapor pressure osmometer in that only the freezing
point osmometer:

A. Cools the sample
B. Is sensitive to ethanol
C. Requires a thermoelectric module
D. Requires calibration with aqueous standards

A

B. Is sensitive to ethanol

How well did you know this?
1
Not at all
2
3
4
5
Perfectly
54
Q

The method for measuring iron or lead by plating the metal and then oxidizing it is called:

A. Polarography
B. Coulometry
C. Anodic stripping voltometry
D. Amperometry

A

C. Anodic stripping voltometry

How well did you know this?
1
Not at all
2
3
4
5
Perfectly
55
Q

The term isocratic is used in high-performance liquid chromatography (HPLC) to mean the:

A. Mobile phase is at constant temperature
B. Stationary phase is equilibrated with the mobile phase
C. Mobile phase consists of a constant solvent composition
D. Flow rate of the mobile phase is regulated

A

C. Mobile phase consists of a constant solvent composition

How well did you know this?
1
Not at all
2
3
4
5
Perfectly
56
Q

The term reverse phase is used in HPLC to indicate that the mobile phase is:

A. More polar than the stationary phase
B. Liquid and the stationary phase is solid
C. Organic and the stationary phase is aqueous
D. A stronger solvent than the stationary phase

A

A. More polar than the stationary phase

How well did you know this?
1
Not at all
2
3
4
5
Perfectly
57
Q

What is the primary means of solute separation in HPLC using a C18 column?

A. Anion exchange
B. Size exclusion
C. Partitioning
D. Cation exchange

A

C. Partitioning

How well did you know this?
1
Not at all
2
3
4
5
Perfectly
58
Q

The most commonly used detector for clinical gas–liquid chromatography (GLC) is based upon:

A. Ultraviolet light absorbance at 254 nm
B. Flame ionization
C. Refractive index
D. Thermal conductance

A

B. Flame ionization

How well did you know this?
1
Not at all
2
3
4
5
Perfectly
59
Q

What type of detector is used in high-performance liquid chromatography with electrochemical
detection (HPLC–ECD)?

A. Calomel electrode
B. Conductivity electrode
C. Glassy carbon electrode
D. Polarographic electrode

A

C. Glassy carbon electrode

How well did you know this?
1
Not at all
2
3
4
5
Perfectly
60
Q

In gas chromatography, the elution order of volatiles is usually based upon the:

A. Boiling point
B. Molecular size
C. Carbon content
D. Polarity

A

A. Boiling point

How well did you know this?
1
Not at all
2
3
4
5
Perfectly
61
Q

Select the chemical that is used in most HPLC procedures to decrease solvent polarity.

A. Hexane
B. Nonane
C. Chloroform
D. Acetonitrile

A

D. Acetonitrile

How well did you know this?
1
Not at all
2
3
4
5
Perfectly
62
Q

In thin-layer chromatography (TLC), the distance the solute migrates divided by the distance the
solvent migrates is the:

A. tR
B. Kd
C. Rf
D. pK

A

C. Rf

How well did you know this?
1
Not at all
2
3
4
5
Perfectly
63
Q

Which reagent is used in thin-layer chromatography (TLC) to extract cocaine metabolites from urine?

A. Acid and sodium chloride
B. Alkali and organic solvent
C. Chloroform and sodium acetate
D. Neutral solution of ethyl acetate

A

B. Alkali and organic solvent

How well did you know this?
1
Not at all
2
3
4
5
Perfectly
64
Q

What is the purpose of an internal standard in HPLC and GC methods?

A. To compensate for variation in extraction and injection
B. To correct for background absorbance
C. To compensate for changes in flow rate
D. To correct for coelution of solutes

A

A. To compensate for variation in extraction and injection

How well did you know this?
1
Not at all
2
3
4
5
Perfectly
65
Q

What is the confirmatory method for measuring drugs of abuse?

A. HPLC
B. Enzyme-multiplied immunoassay technique (EMIT)
C. Gas chromatography with mass spectroscopy (GC-MS)
D. TLC

A

C. Gas chromatography with mass spectroscopy (GC-MS)

How well did you know this?
1
Not at all
2
3
4
5
Perfectly
66
Q

The fragments typically produced and analyzed in methods employing mass spectroscopy are
typically:
A. Of low molecular size ranging from 10–100 daltons
B. Cations caused by electron loss or proton attachment
C. Anions caused by bombarding the molecule with an electron source
D. Neutral species formed after excited molecules form a stable resonance structure

A

B. Cations caused by electron loss or proton attachment

How well did you know this?
1
Not at all
2
3
4
5
Perfectly
67
Q

What component is used in a GC-MS but not used in an LC-MS?

A. Electron source
B. Mass filter
C. Detector
D. Vacuum

A

A. Electron source

How well did you know this?
1
Not at all
2
3
4
5
Perfectly
68
Q

What process is most often used in LC-MS to introduce the sample into the mass filter?

A. Electrospray ionization
B. Chemical ionization
C. Electron impact ionization
D. Fast atom bombardment

A

A. Electrospray ionization

How well did you know this?
1
Not at all
2
3
4
5
Perfectly
69
Q

In mass spectroscopy, the term base peak typically refers to:

A. The peak with the lowest mass
B. The peak with the most abundance
C. A natural isotope of the molecular ion
D. The first peak to reach the mass detector

A

B. The peak with the most abundance

How well did you know this?
1
Not at all
2
3
4
5
Perfectly
70
Q

Which method is the most useful when screening for errors of amino and organic acid metabolism?

A. Two-dimensional thin-layer chromatography
B. Gas chromatography
C. Electrospray ionization tandem-mass spectroscopy
D. Inductively charged coupled-mass spectroscopy

A

C. Electrospray ionization tandem-mass spectroscopy

How well did you know this?
1
Not at all
2
3
4
5
Perfectly
71
Q

In tandem-mass spectroscopy, the first mass filter performs the same function as:

A. The ion source
B. The chromatography column
C. Extraction
D. The vacuum system

A

B. The chromatography column

How well did you know this?
1
Not at all
2
3
4
5
Perfectly
72
Q

SITUATION: A GC-MS analysis using nitrogen as the carrier gas shows an extensively noisy baseline.
A sample of the solvent used for the extraction procedure, ethyl acetate, was injected and showed
the same noise. Results of an Autotune test showed the appearance of a base peak at 16 with
two smaller peaks at 17 and 18. These results indicate:

A. The solvent is contaminated
B. The carrier gas is contaminated
C. There is electrical noise in the detector
D. The ion source is dirty

A

B. The carrier gas is contaminated

How well did you know this?
1
Not at all
2
3
4
5
Perfectly
73
Q

Why is vacuum necessary in the mass filter of a mass spectrometer?

A. Ionization does not occur at atmospheric pressure
B. It prevents collision between fragments
C. It removes electrons from the ion source
D. It prevents contamination

A

B. It prevents collision between fragments

How well did you know this?
1
Not at all
2
3
4
5
Perfectly
74
Q

What method is used to introduce the sample into a mass spectrometer for analysis of a trace element?

A. Electrospray ionization
B. Laser desorption
C. Inductively charged plasma (ICP) ionization
D. Direct injection

A

C. Inductively charged plasma (ICP) ionization

How well did you know this?
1
Not at all
2
3
4
5
Perfectly
75
Q

Which component is needed for a thermal cycler to amplify DNA?

A. Programmable heating and cooling unit
B. Vacuum chamber with zero head space
C. Sealed airtight constant-temperature chamber
D. Temperature-controlled ionization chamber

A

A. Programmable heating and cooling unit

How well did you know this?
1
Not at all
2
3
4
5
Perfectly
76
Q

In real-time PCR, what value is needed in order to etermine the threshold?

A. Background signal
B. Melt temperature
C. Maximum fluorescence
D. Threshold cycle

A

A. Background signal

How well did you know this?
1
Not at all
2
3
4
5
Perfectly
77
Q

Given the following real-time PCR amplification curve, what is the threshold cycle?

A. 15
B. 20
C. 25
D. 30

A

C. 25

How well did you know this?
1
Not at all
2
3
4
5
Perfectly
78
Q

In addition to velocity, what variable is also needed to calculate the relative centrifugal force
(g force) of a centrifuge?

A. Head radius
B. Angular velocity coefficient
C. Diameter of the centrifuge tube
D. Ambient temperature in degrees Centigrade

A

A. Head radius

How well did you know this?
1
Not at all
2
3
4
5
Perfectly
79
Q

Which of the following situations is likely to cause an error when weighing with an electronic
analytical balance?

A. Failure to keep the knife edge clean
B. Failure to close the doors of the balance before reading the weight
C. Oxidation on the surface of the substitution weights
D. Using the balance without allowing it to warm up for at least 10 minutes

A

B. Failure to close the doors of the balance before reading the weight

How well did you know this?
1
Not at all
2
3
4
5
Perfectly
80
Q

When calibrating a semiautomatic pipet that has a fixed delivery of 10.0 μL using a gravimetric
method, what should be the average weight of deionized water transferred?

A. 10.0 μg
B. 100.0 μg
C. 1.0 mg
D. 10.0 mg

A

D. 10.0 mg

How well did you know this?
1
Not at all
2
3
4
5
Perfectly
81
Q

Which of the following represents the Henderson–Hasselbalch equation as applied to blood pH?

A. pH = 6.1 + log HCO3–/PCO2
B. pH = 6.1 + log HCO3–/(0.03 × PCO2)
C. pH = 6.1 + log dCO2/HCO3–
D. pH = 6.1 + log (0.03 × PCO2)/HCO3–

A

B. pH = 6.1 + log HCO3–/(0.03 × PCO2)

How well did you know this?
1
Not at all
2
3
4
5
Perfectly
82
Q

What is the PO2 of calibration gas containing 20.0% O2, when the barometric pressure is 30 in.?

A. 60 mm Hg
B. 86 mm Hg
C. 143 mm Hg
D. 152 mm Hg

A

C. 143 mm Hg

How well did you know this?
1
Not at all
2
3
4
5
Perfectly
83
Q

What is the blood pH when the partial pressure of carbon dioxide (PCO2) is 60 mm Hg and the
bicarbonate concentration is 18 mmol/L?

A. 6.89
B. 7.00
C. 7.10
D. 7.30

A

C. 7.10

How well did you know this?
1
Not at all
2
3
4
5
Perfectly
84
Q

Which of the following best represents the reference (normal) range for arterial pH?

A. 7.35–7.45
B. 7.42–7.52
C. 7.38–7.68
D. 6.85–7.56

A

A. 7.35–7.45

How well did you know this?
1
Not at all
2
3
4
5
Perfectly
85
Q

What is the normal ratio of bicarbonate to dissolved carbon dioxide (HCO3–:dCO2) in arterial blood?

A. 1:10
B. 10:1
C. 20:1
D. 30:1

A

C. 20:1

How well did you know this?
1
Not at all
2
3
4
5
Perfectly
86
Q

What is the PCO2 if the dCO2 is 1.8 mmol/L?

A. 24 mm Hg
B. 35 mm Hg
C. 60 mm Hg
D. 72 mm Hg

A

C. 60 mm Hg

How well did you know this?
1
Not at all
2
3
4
5
Perfectly
87
Q

In the Henderson–Hasselbalch expression pH = 6.1 + log HCO3–/dCO2, the 6.1 represents:

A. The combined hydration and dissociation constants for CO2 in blood at 37°C
B. The solubility constant for CO2 gas
C. The dissociation constant of H2O
D. The ionization constant of sodium bicarbonate (NaHCO3)

A

A. The combined hydration and dissociation constants for CO2 in blood at 37°C

How well did you know this?
1
Not at all
2
3
4
5
Perfectly
88
Q

Which of the following contributes the most to the serum total CO2?

A. PCO2
B. dCO2
C. HCO3–
D. Carbonium ion

A

C. HCO3–

How well did you know this?
1
Not at all
2
3
4
5
Perfectly
89
Q

In addition to sodium bicarbonate, what other substance contributes most to the amount of base
in the blood?

A. Hemoglobin concentration
B. Dissolved O2 concentration
C. Inorganic phosphorus
D. Organic phosphate

A

A. Hemoglobin concentration

How well did you know this?
1
Not at all
2
3
4
5
Perfectly
90
Q

Which of the following effects results from exposure of a normal arterial blood sample to
room air?

A. PO2 increased PCO2 decreased pH increased
B. PO2 decreased PCO2 increased pH decreased
C. PO2 increased PCO2 decreased pH decreased
D. PO2 decreased PCO2 decreased pH decreased

A

A. PO2 increased PCO2 decreased pH increased

How well did you know this?
1
Not at all
2
3
4
5
Perfectly
91
Q

Which of the following formulas for O2 content is correct?

A. O2 content = %O2 saturation/100 × Hgb g/dL × 1.39 mL/g + (0.0031 × PO2)
B. O2 content = PO2 × 0.0306 mmol/L/mm
C. O2 content = O2 saturation × Hgb g/dL × 0.003 mL/g
D. O2 content = O2 capacity × 0.003 mL/g

A

A. O2 content = %O2 saturation/100 × Hgb g/dL × 1.39 mL/g + (0.0031 × PO2)

How well did you know this?
1
Not at all
2
3
4
5
Perfectly
92
Q

The normal difference between alveolar and arterial PO2 (PAO2–PaO2 difference) is:

A. 3 mm Hg
B. 10 mm Hg
C. 40 mm Hg
D. 50 mm Hg

A

B. 10 mm Hg

How well did you know this?
1
Not at all
2
3
4
5
Perfectly
93
Q

A decreased PAO2–PaO2 difference is found in:

A. A/V (arteriovenous) shunting
B. V/Q (ventilation/perfusion) inequality
C. Ventilation defects
D. All of these options

A

C. Ventilation defects

How well did you know this?
1
Not at all
2
3
4
5
Perfectly
94
Q

The determination of the oxygen saturation of hemoglobin is best accomplished by:

A. Polychromatic absorbance measurements of a whole-blood hemolysate
B. Near infrared transcutaneous absorbance measurement
C. Treatment of whole blood with alkaline dithionite prior to measuring absorbance
D. Calculation using PO2 and total hemoglobin by direct spectrophotometry

A

A. Polychromatic absorbance measurements of a whole-blood hemolysate

How well did you know this?
1
Not at all
2
3
4
5
Perfectly
95
Q

Correction of pH for a patient with a body temperature of 38°C would require:

A. Subtraction of 0.015
B. Subtraction of 0.01%
C. Addition of 0.020
D. Subtraction of 0.020

A

A. Subtraction of 0.015

How well did you know this?
1
Not at all
2
3
4
5
Perfectly
96
Q

Select the anticoagulant of choice for blood gas studies.

A. Sodium citrate 3.2%
B. Lithium heparin 100 U/mL blood
C. Sodium citrate 3.8%
D. Ammonium oxalate 5.0%

A

B. Lithium heparin 100 U/mL blood

How well did you know this?
1
Not at all
2
3
4
5
Perfectly
97
Q

What is the maximum recommended storage time and temperature for an arterial blood gas sample drawn in a plastic syringe?

Storage Time Temperature
a. 10 min 2°C–8°C
b. 20 min 2°C–8°C
c. 30 min 2°C–8°C
d. 30 min 22°C

A

d. 30 min 22°C

How well did you know this?
1
Not at all
2
3
4
5
Perfectly
98
Q

A patient’s blood gas results are as follows:
pH = 7.26
dCO2 = 2.0 mmol/L
HCO3– = 29 mmol/L

These results would be classified as:

A. Metabolic acidosis
B. Metabolic alkalosis
C. Respiratory acidosis
D. Respiratory alkalosis

A

C. Respiratory acidosis

How well did you know this?
1
Not at all
2
3
4
5
Perfectly
99
Q

A patient’s blood gas results are:
pH = 7.50
PCO2 = 55 mm Hg
HCO3– = 40 mmol/L

These results indicate:

A. Respiratory acidosis
B. Metabolic alkalosis
C. Respiratory alkalosis
D. Metabolic acidosis

A

B. Metabolic alkalosis

How well did you know this?
1
Not at all
2
3
4
5
Perfectly
100
Q

Which set of results is consistent with uncompensated respiratory alkalosis?

A. pH 7.70 HCO3 30 mmol/L PCO2 25 mm Hg
B. pH 7.66 HCO3 22 mmol/L PCO2 20 mm Hg
C. pH 7.46 HCO3 38 mmol/L PCO2 55 mm Hg
D. pH 7.36 HCO3 22 mmol/L PCO2 38 mm Hg

A

B. pH 7.66 HCO3 22 mmol/L PCO2 20 mm Hg

How well did you know this?
1
Not at all
2
3
4
5
Perfectly
101
Q

Which of the following will shift the O2 dissociation curve to the left?

A. Anemia
B. Hyperthermia
C. Hypercapnia
D. Alkalosis

A

D. Alkalosis

How well did you know this?
1
Not at all
2
3
4
5
Perfectly
102
Q

In which circumstance will the reporting of calculated oxygen saturation of hemoglobin based on PO2, PCO2, pH, temperature, and hemoglobin be in error?

A. Carbon monoxide poisoning
B. Diabetic ketoacidosis
C. Patient receiving oxygen therapy
D. Assisted ventilation for respiratory failure

A

A. Carbon monoxide poisoning

How well did you know this?
1
Not at all
2
3
4
5
Perfectly
103
Q

Which would be consistent with partially compensated respiratory acidosis?

A. pH PCO2 Bicarbonate
increased increased increased
B. pH PCO2 Bicarbonate
increased decreased decreased
C. pH PCO2 Bicarbonate
decreased decreased decreased
D. pH PCO2 Bicarbonate
decreased increased increased

A

D. pH PCO2 Bicarbonate
decreased increased increased

How well did you know this?
1
Not at all
2
3
4
5
Perfectly
104
Q

Which condition results in metabolic acidosis with severe hypokalemia and chronic alkaline urine?

A. Diabetic ketoacidosis
B. Phenformin-induced acidosis
C. Renal tubular acidosis
D. Acidosis caused by starvation

A

C. Renal tubular acidosis

How well did you know this?
1
Not at all
2
3
4
5
Perfectly
105
Q

Which of the following mechanisms is responsible for metabolic acidosis?

A. Bicarbonate deficiency
B. Excessive retention of dissolved CO2
C. Accumulation of volatile acids
D. Hyperaldosteronism

A

A. Bicarbonate deficiency

How well did you know this?
1
Not at all
2
3
4
5
Perfectly
106
Q

Which of the following disorders is associated with lactate acidosis?

A. Diarrhea
B. Renal tubular acidosis
C. Hypoaldosteronism
D. Alcoholism

A

D. Alcoholism

How well did you know this?
1
Not at all
2
3
4
5
Perfectly
107
Q

Which of the following is the primary mechanism of compensation for metabolic acidosis?

A. Hyperventilation
B. Release of epinephrine
C. Aldosterone release
D. Bicarbonate excretion

A

A. Hyperventilation

How well did you know this?
1
Not at all
2
3
4
5
Perfectly
108
Q

The following conditions are all causes of alkalosis. Which condition is associated with respiratory
(rather than metabolic) alkalosis?

A. Anxiety
B. Hypovolemia
C. Hyperaldosteronism
D. Hypoparathyroidism

A

A. Anxiety

How well did you know this?
1
Not at all
2
3
4
5
Perfectly
109
Q

Which of the following conditions is associated with both metabolic and respiratory alkalosis?

A. Hyperchloremia
B. Hypernatremia
C. Hyperphosphatemia
D. Hypokalemia

A

D. Hypokalemia

How well did you know this?
1
Not at all
2
3
4
5
Perfectly
110
Q

Which of the following conditions is associated with both metabolic and respiratory alkalosis?

A. Hyperchloremia
B. Hypernatremia
C. Hyperphosphatemia
D. Hypokalemia

A

B. Hypernatremia

How well did you know this?
1
Not at all
2
3
4
5
Perfectly
111
Q

Which of the following conditions is classified as normochloremic acidosis?

A. Diabetic ketoacidosis
B. Chronic pulmonary obstruction
C. Uremic acidosis
D. Diarrhea

A

A. Diabetic ketoacidosis

How well did you know this?
1
Not at all
2
3
4
5
Perfectly
112
Q

Which PCO2 value would be seen in maximally compensated metabolic acidosis?

A. 15 mm Hg
B. 30 mm Hg
C. 40 mm Hg
D. 60 mm Hg

A

A. 15 mm Hg

How well did you know this?
1
Not at all
2
3
4
5
Perfectly
113
Q

A patient has the following arterial blood gas results:
pH = 7.56
PCO2 = 25 mm Hg
PO2 = 100 mm Hg
HCO3– = 22 mmol/L

These results are most likely the result of which condition?

A. Improper specimen collection
B. Prolonged storage
C. Hyperventilation
D. Hypokalemia

A

C. Hyperventilation

How well did you know this?
1
Not at all
2
3
4
5
Perfectly
114
Q

Why are three levels used for quality control of pH and blood gases?

A. Systematic errors can be detected earlier than with two controls
B. Analytical accuracy needs to be greater than for other analytes
C. High, normal, and low ranges must always be evaluated
D. A different level is needed for pH, PCO2, and PO2

A

A. Systematic errors can be detected earlier than with two controls

How well did you know this?
1
Not at all
2
3
4
5
Perfectly
115
Q

A single-point calibration is performed between each blood gas sample in order to:

A. Correct the electrode slope
B. Correct electrode and instrument drift
C. Compensate for temperature variance
D. Prevent contamination by the previous sample

A

B. Correct electrode and instrument drift

How well did you know this?
1
Not at all
2
3
4
5
Perfectly
116
Q

In which condition would hypochloremia be expected?

A. Respiratory alkalosis
B. Metabolic acidosis
C. Metabolic alkalosis
D. All of these options

A

C. Metabolic alkalosis

How well did you know this?
1
Not at all
2
3
4
5
Perfectly
117
Q

Given the following serum electrolyte data, determine the anion gap.
Na = 132 mmol/L
Cl = 90 mmol/L
HCO3– = 22 mmol/L

A. 12 mmol/L
B. 20 mmol/L
C. 64 mmol/L
D. Cannot be determined from the information provided

A

B. 20 mmol/L

How well did you know this?
1
Not at all
2
3
4
5
Perfectly
118
Q

Which of the following conditions will cause an increased anion gap?

A. Diarrhea
B. Hypoaldosteronism
C. Hyperkalemia
D. Renal failure

A

D. Renal failure

How well did you know this?
1
Not at all
2
3
4
5
Perfectly
119
Q

Alcoholism, liver failure, and hypoxia induce acidosis by causing:

A. Depletion of cellular NAD+
B. Increased excretion of bicarbonate
C. Increased retention of PCO2
D. Loss of carbonic anhydrase

A

A. Depletion of cellular NAD+

How well did you know this?
1
Not at all
2
3
4
5
Perfectly
120
Q

Which of the following is the primary mechanism causing respiratory alkalosis?

A. Hyperventilation
B. Deficient alveolar diffusion
C. Deficient pulmonary perfusion
D. Parasympathetic inhibition

A

A. Hyperventilation

How well did you know this?
1
Not at all
2
3
4
5
Perfectly
121
Q

Which condition can result in acidosis?

A. Cystic fibrosis
B. Vomiting
C. Hyperaldosteronism
D. Excessive O2 therapy

A

D. Excessive O2 therapy

How well did you know this?
1
Not at all
2
3
4
5
Perfectly
122
Q

Which of the following conditions is associated with an increase in ionized calcium (Cai) in the
blood?

A. Alkalosis
B. Hypoparathyroidism
C. Hyperalbuminemia
D. Malignancy

A

D. Malignancy

How well did you know this?
1
Not at all
2
3
4
5
Perfectly
123
Q

Which of the following laboratory results is consistent with primary hypoparathyroidism?

A. Low calcium; high inorganic phosphorus Pi
B. Low calcium; low Pi
C. High calcium; high Pi
D. High calcium; low Pi

A

A. Low calcium; high inorganic phosphorus Pi

How well did you know this?
1
Not at all
2
3
4
5
Perfectly
124
Q

Which of the following conditions is associated with hypophosphatemia?

A. Rickets
B. Multiple myeloma
C. Renal failure
D. Hypervitaminosis D

A

A. Rickets

How well did you know this?
1
Not at all
2
3
4
5
Perfectly
125
Q

Which of the following tests is consistently abnormal in osteoporosis?

A. High urinary calcium
B. High serum Pi
C. Low serum calcium
D. High urine or serum N-telopeptide of type 1 collagen

A

D. High urine or serum N-telopeptide of type 1 collagen

How well did you know this?
1
Not at all
2
3
4
5
Perfectly
126
Q

Which of the following is a marker for bone formation?

A. Osteocalcin
B. Tartrate resistant acid phosphatase (TRAP)
C. Urinary pyridinoline and deoxypyridinoline
D. Urinary C-telopeptide and N-telopeptide crosslinks (CTx and NTx)

A

A. Osteocalcin

How well did you know this?
1
Not at all
2
3
4
5
Perfectly
127
Q

What role do CTx and NTx play in the management of osteoporosis?

A. Increased urinary excretion is diagnostic of early stage disease
B. Increased levels indicate a low risk of developing osteoporosis
C. Decreased urinary excretion indicates a positive response to treatment
D. The rate of urinary excretion correlates with the stage of the disease

A

C. Decreased urinary excretion indicates a positive response to treatment

How well did you know this?
1
Not at all
2
3
4
5
Perfectly
128
Q

What role does vitamin D measurement play in the management of osteoporosis?

A. Vitamin D deficiency must be demonstrated to establish the diagnosis
B. Vitamin D is consistently elevated in osteoporosis
C. A normal vitamin D level rules out osteoporosis
D. Vitamin D deficiency is a risk factor for developing osteoporosis

A

D. Vitamin D deficiency is a risk factor for developing osteoporosis

How well did you know this?
1
Not at all
2
3
4
5
Perfectly
129
Q

Which statement best describes testing recommendations for vitamin D?

A. Vitamin D testing should be reserved only for those persons who demonstrate hypercalcemia
of an undetermined cause
B. Vitamin D testing should be specific for the 1,25(OH)D3 form
C. Testing should be for total vitamin D when screening for deficiency
D. Vitamin D testing should not be performed if the patient is receiving a vitamin D supplement

A

C. Testing should be for total vitamin D when screening for deficiency

How well did you know this?
1
Not at all
2
3
4
5
Perfectly
130
Q

The serum level of which of the following laboratory tests is decreased in both VDDR
and VDRR?

A. Vitamin D
B. Calcium
C. Pi
D. Parathyroid hormone

A

C. Pi

How well did you know this?
1
Not at all
2
3
4
5
Perfectly
131
Q

Which of the following is the most accurate measurement of Pi in serum?

A. Rate of unreduced phosphomolybdate formation at 340 nm
B. Measurement of phosphomolybdenum blue at 680 nm
C. Use of aminonaptholsulfonic acid to reduce phosphomolybdate
D. Formation of a complex with malachite green dye

A

A. Rate of unreduced phosphomolybdate formation at 340 nm

How well did you know this?
1
Not at all
2
3
4
5
Perfectly
132
Q

What is the percentage of serum calcium that is ionized (Cai)?

A. 30%
B. 45%
C. 60%
D. 80%

A

B. 45%

How well did you know this?
1
Not at all
2
3
4
5
Perfectly
133
Q

Which of the following conditions will cause erroneous Cai results? Assume that the samples
are collected and stored anaerobically, kept at 4°C until measurement, and stored for no
longer than 1 hour.

A. Slight hemolysis during venipuncture
B. Assay of whole blood collected in sodium oxalate
C. Analysis of serum in a barrier gel tube stored at 4°C until the clot has formed
D. Analysis of whole blood collected in sodium heparin, 20 U/mL (low-heparin tube)

A

B. Assay of whole blood collected in sodium oxalate

How well did you know this?
1
Not at all
2
3
4
5
Perfectly
134
Q

Which of the following conditions is associated with a low serum magnesium?

A. Addison’s disease
B. Hemolytic anemia
C. Hyperparathyroidism
D. Pancreatitis

A

D. Pancreatitis

How well did you know this?
1
Not at all
2
3
4
5
Perfectly
135
Q

When measuring calcium with the complexometric dye o-cresolphthalein
complexone, magnesium is kept from interfering by:

A. Using an alkaline pH
B. Adding 8-hydroxyquinoline
C. Measuring at 450 nm
D. Complexing to EDTA

A

B. Adding 8-hydroxyquinoline

How well did you know this?
1
Not at all
2
3
4
5
Perfectly
136
Q

Which electrolyte measurement is least affected by hemolysis?

A. Potassium
B. Calcium
C. Pi
D. Magnesium

A

B. Calcium

How well did you know this?
1
Not at all
2
3
4
5
Perfectly
137
Q

Which of the following conditions is associated with hypokalemia?

A. Addison’s disease
B. Hemolytic anemia
C. Digoxin intoxication
D. Alkalosis

A

D. Alkalosis

How well did you know this?
1
Not at all
2
3
4
5
Perfectly
138
Q

Which of the following conditions is most likely to produce an elevated plasma potassium?

A. Hypoparathyroidism
B. Cushing’s syndrome
C. Diarrhea
D. Digitalis overdose

A

D. Digitalis overdose

How well did you know this?
1
Not at all
2
3
4
5
Perfectly
139
Q

Which of the following values is the threshold critical value (alert or action level) for low plasma
potassium?

A. 1.5 mmol/L
B. 2.0 mmol/L
C. 2.5 mmol/L
D. 3.5 mmol/L

A

C. 2.5 mmol/L

How well did you know this?
1
Not at all
2
3
4
5
Perfectly
140
Q

Which electrolyte is least likely to be elevated in renal failure?

A. Potassium
B. Magnesium
C. Inorganic phosphorus
D. Sodium

A

D. Sodium

How well did you know this?
1
Not at all
2
3
4
5
Perfectly
141
Q

Which of the following is the primary mechanism for vasopressin (ADH) release?

A. Hypovolemia
B. Hyperosmolar plasma
C. Renin release
D. Reduced renal blood flow

A

B. Hyperosmolar plasma

How well did you know this?
1
Not at all
2
3
4
5
Perfectly
142
Q

Which of the following conditions is associated with hypernatremia?

A. Diabetes insipidus
B. Hypoaldosteronism
C. Burns
D. Diarrhea

A

A. Diabetes insipidus

How well did you know this?
1
Not at all
2
3
4
5
Perfectly
143
Q

Which of the following values is the threshold critical value (alert or action level) for high plasma
sodium?

A. 150 mmol/L
B. 160 mmol/L
C. 170 mmol/L
D. 180 mmol/L

A

B. 160 mmol/L

How well did you know this?
1
Not at all
2
3
4
5
Perfectly
144
Q

Which of the following conditions is associated with total body sodium excess?

A. Renal failure
B. Hyperthyroidism
C. Hypoparathyroidism
D. Diabetic ketoacidosis

A

A. Renal failure

How well did you know this?
1
Not at all
2
3
4
5
Perfectly
145
Q

Which of the following conditions is associated with hyponatremia?

A. Diuretic therapy
B. Cushing’s syndrome
C. Diabetes insipidus
D. Nephrotic syndrome

A

A. Diuretic therapy

How well did you know this?
1
Not at all
2
3
4
5
Perfectly
146
Q

Which of the following conditions involving electrolytes is described correctly?

A. Pseudohyponatremia occurs only when undiluted samples are measured
B. Potassium levels are slightly higher in heparinized plasma than in serum
C. Hypoalbuminemia causes low total calcium but does not affect Cai
D. Hypercalcemia may be induced by low serum magnesium

A

C. Hypoalbuminemia causes low total calcium but does not affect Cai

How well did you know this?
1
Not at all
2
3
4
5
Perfectly
147
Q

Which of the following laboratory results is usually associated with cystic fibrosis?

A. Sweat chloride greater than 60 mmol/L
B. Elevated serum sodium and chloride
C. Elevated fecal trypsin activity
D. Low glucose

A

A. Sweat chloride greater than 60 mmol/L

How well did you know this?
1
Not at all
2
3
4
5
Perfectly
148
Q

When performing a sweat chloride collection, which of the following steps will result in
analytical error?

A. Using unweighed gauze soaked in pilocarpine nitrate on the inner surface of the forearm to
stimulate sweating
B. Collecting more than 75 mg of sweat in 30 minutes
C. Leaving the preweighed gauze on the inside of the arm exposed to air during collection
D. Rinsing the collected sweat from the gauze pad using chloride titrating solution

A

C. Leaving the preweighed gauze on the inside of the arm exposed to air during collection

How well did you know this?
1
Not at all
2
3
4
5
Perfectly
149
Q

Which electrolyte level best correlates with plasma osmolality?

A. Sodium
B. Chloride
C. Bicarbonate
D. Calcium

A

A. Sodium

How well did you know this?
1
Not at all
2
3
4
5
Perfectly
150
Q

Which formula is most accurate in predicting plasma osmolality?

A. Na + 2(Cl) + BUN + glucose
B. 2(Na) + 2(Cl) + glucose + urea
C. 2(Na) + (glucose ÷ 18) + (BUN ÷ 2.8)
D. Na + Cl + K + HCO3

A

C. 2(Na) + (glucose ÷ 18) + (BUN ÷ 2.8)

How well did you know this?
1
Not at all
2
3
4
5
Perfectly
151
Q

Which of the following biochemical processes is promoted by insulin?

A. Glycogenolysis
B. Gluconeogenesis
C. Lipolysis
D. Uptake of glucose by cells

A

D. Uptake of glucose by cells

How well did you know this?
1
Not at all
2
3
4
5
Perfectly
152
Q

Which of the following hormones promotes hyperglycemia?

A. Calcitonin
B. Growth hormone
C. Aldosterone
D. Renin

A

B. Growth hormone

How well did you know this?
1
Not at all
2
3
4
5
Perfectly
153
Q

Which of the following is characteristic of type 1 diabetes mellitus?

A. Requires an oral glucose tolerance test for diagnosis
B. Is the most common form of diabetes mellitus
C. Usually occurs after age 40
D. Requires insulin replacement to prevent ketosis

A

D. Requires insulin replacement to prevent ketosis

How well did you know this?
1
Not at all
2
3
4
5
Perfectly
154
Q

Which of the following is characteristic of type 2 diabetes mellitus?

A. Insulin levels are consistently low
B. Most cases require a 3-hour oral glucose tolerance test to diagnose
C. Hyperglycemia is often controlled without insulin replacement
D. The condition is associated with unexplained weight loss

A

C. Hyperglycemia is often controlled without insulin replacement

How well did you know this?
1
Not at all
2
3
4
5
Perfectly
155
Q

Which of the following results falls within the diagnostic criteria for diabetes mellitus?

A. Fasting plasma glucose of 120 mg/dL
B. Two-hour postprandial plasma glucose of 160 mg/dL
C. Two-hour plasma glucose of 180 mg/dL following a 75 g oral glucose challenge
D. Random plasma glucose of 250 mg/dL and presence of symptoms

A

D. Random plasma glucose of 250 mg/dL and presence of symptoms

How well did you know this?
1
Not at all
2
3
4
5
Perfectly
156
Q

Select the most appropriate adult reference range for fasting blood glucose.

A. 40–105 mg/dL (2.22–5.82 mmol/L)
B. 60–140 mg/dL (3.33–7.77 mmol/L)
C. 65–99 mg/dL (3.61–5.50 mmol/L)
D. 75–150 mg/dL (4.16–8.32 mmol/L)

A

C. 65–99 mg/dL (3.61–5.50 mmol/L)

How well did you know this?
1
Not at all
2
3
4
5
Perfectly
157
Q

When preparing a patient for an oral glucose tolerance test (OGTT), which of the following
conditions will lead to erroneous results?

A. The patient remains ambulatory for 3 days prior to the test
B. Carbohydrate intake is restricted to below 150 g/day for 3 days prior to test
C. No food, coffee, tea, or smoking is allowed 8 hours before and during the test
D. Administration of 75 g of glucose is given to an adult patient following a 10–12-hour fast

A

B. Carbohydrate intake is restricted to below 150 g/day for 3 days prior to test

How well did you know this?
1
Not at all
2
3
4
5
Perfectly
158
Q

Which of the following 2-hour glucose challenge results would be classified as impaired glucose
tolerance (IGT)?
Two-hour serum glucose:

A. 130 mg/dL
B. 135 mg/dL
C. 150 mg/dL
D. 204 mg/dL

A

C. 150 mg/dL

How well did you know this?
1
Not at all
2
3
4
5
Perfectly
159
Q

Which statement regarding gestational diabetes mellitus (GDM) is correct?

A. Is diagnosed using the same oral glucose tolerance criteria as in nonpregnancy
B. Converts to diabetes mellitus after pregnancy in 60%–75% of cases
C. Presents no increased health risk to the fetus
D. Is defined as glucose intolerance originating during pregnancy

A

D. Is defined as glucose intolerance originating during pregnancy

How well did you know this?
1
Not at all
2
3
4
5
Perfectly
160
Q

Which of the following findings is characteristic of all forms of clinical hypoglycemia?

A. A fasting blood glucose value below 55 mg/dL
B. High fasting insulin levels
C. Neuroglycopenic symptoms at the time of low blood sugar
D. Decreased serum C peptide

A

C. Neuroglycopenic symptoms at the time of low blood sugar

How well did you know this?
1
Not at all
2
3
4
5
Perfectly
161
Q

Which statement regarding glycated (glycosylated)Hgb (G-Hgb) is true?

A. Has a sugar attached to the C-terminal end of the β chain
B. Is a highly reversible aminoglycan
C. Reflects the extent of glucose regulation in the 8- to 12-week interval prior to sampling
D. Will be abnormal within 4 days following an episode of hyperglycemia

A

C. Reflects the extent of glucose regulation in the 8- to 12-week interval prior to sampling

How well did you know this?
1
Not at all
2
3
4
5
Perfectly
162
Q

What is the American Diabetes Association recommended cutoff value for adequate control of
blood glucose in diabetics as measured by glycated hemoglobin?

A. 5%
B. 6.5%
C. 9.5%
D. 11%

A

B. 6.5%

How well did you know this?
1
Not at all
2
3
4
5
Perfectly
163
Q

Which statement regarding measurement of Hgb A1c is true?

A. Levels do not need to be done fasting
B. Both the labile and stable Hgb A1c fractions are measured
C. Samples should be measured within 2 hours of collection
D. The assay must be done by chromatography

A

A. Levels do not need to be done fasting

How well did you know this?
1
Not at all
2
3
4
5
Perfectly
164
Q

Which statement regarding measurement of Hgb A1c is true?

A. Levels do not need to be done fasting
B. Both the labile and stable Hgb A1c fractions are measured
C. Samples should be measured within 2 hours of collection
D. The assay must be done by chromatography

A

B. Both the labile and stable Hgb A1c fractions are measured

How well did you know this?
1
Not at all
2
3
4
5
Perfectly
165
Q

Evaluate the following chromatogram of a whole-blood hemolysate, and identify the cause
and best course of action.

A. Result is not reportable because hemoglobin F is present and interferes
B. The result is not reportable because hemoglobin C is present and interferes
C. The result is not reportable because labile hemoglobin A1c is present
D. The result is reportable; neither hemoglobin F or C interfere

A

D. The result is reportable; neither hemoglobin F or C interfere

How well did you know this?
1
Not at all
2
3
4
5
Perfectly
166
Q

Which statement best describes the use of the Hgb A1C test?

Peak Calibrated Retention Time Peak Area
% Area % Area
Alb 0.60 0.25 12500
F 0.50 0.50 11300
LA1c 0.75 0.70 15545
A1c 6.2 0.90 45112
P3 2.6 1.60 57489
Ao 48.0 1.8 994813
C 43.0 2.00 926745

A. Should be used for monitoring glucose control only
B. May be used for both diagnosis and monitoring
C. Should be used only to monitor persons with type 1 diabetes
D. May be used only to monitor persons with type 2 diabetes

A

B. May be used for both diagnosis and monitoring

How well did you know this?
1
Not at all
2
3
4
5
Perfectly
167
Q

According to American Diabetes Association criteria, which result is consistent with a diagnosis
of impaired fasting glucose?

A. 99 mg/dL
B. 117 mg/dL
C. 126 mg/dL
D. 135 mg/dL

A

B. 117 mg/dL

How well did you know this?
1
Not at all
2
3
4
5
Perfectly
168
Q

What is the recommended cutoff for the early detection of chronic kidney disease in diabetics
using the test for microalbuminuria?

A. >30 mg/g creatinine
B. >80 mg/g creatinine
C. >200 mg/g creatinine
D. >80 mg/L

A

A. >30 mg/g creatinine

How well did you know this?
1
Not at all
2
3
4
5
Perfectly
169
Q

In addition to measuring blood glucose, Hgb A1c, and microalbumin, which test should be done on
diabetic persons once per year?

A. Urine glucose
B. Urine ketones
C. Plasma fructosamines
D. Estimated glomerular filtration rate

A

D. Estimated glomerular filtration rate

How well did you know this?
1
Not at all
2
3
4
5
Perfectly
170
Q

Which testing situation is appropriate for the use of point-of-care whole-blood glucose methods?

A. Screening for type 2 diabetes mellitus
B. Diagnosis of diabetes mellitus
C. Monitoring of blood glucose control in type 1 and type 2 diabetics
D. Monitoring diabetics for hyperglycemic episodes only

A

C. Monitoring of blood glucose control in type 1 and type 2 diabetics

How well did you know this?
1
Not at all
2
3
4
5
Perfectly
171
Q

Which of the following is the reference method for measuring serum glucose?

A. Somogyi–Nelson
B. Hexokinase
C. Glucose oxidase
D. Glucose dehydrogenase

A

B. Hexokinase

How well did you know this?
1
Not at all
2
3
4
5
Perfectly
172
Q

Polarographic methods for glucose analysis are based upon which principle of measurement?

A. Nonenzymatic oxidation of glucose
B. The rate of O2 depletion
C. Chemiluminescence caused by formation of adenosine triphosphate (ATP)
D. The change in electrical potential as glucose is oxidized

A

B. The rate of O2 depletion

How well did you know this?
1
Not at all
2
3
4
5
Perfectly
173
Q

In addition to polarography, what other electrochemical method can be used to measure glucose in plasma?

A. Conductivity
B. Potentiometry
C. Anodic stripping voltammetry
D. Amperometry

A

D. Amperometry

How well did you know this?
1
Not at all
2
3
4
5
Perfectly
174
Q

Select the enzyme that is most specific for β-D-glucose.

A. Hexokinase
B. G-6-PD
C. Phosphohexisomerase
D. Glucose oxidase

A

D. Glucose oxidase

How well did you know this?
1
Not at all
2
3
4
5
Perfectly
175
Q

Select the coupling enzyme used in the hexokinase method for glucose.

A. Glucose-6-phosphate dehydrogenase
B. Peroxidase
C. Glucose dehydrogenase
D. Glucose-6-phosphatase

A

A. Glucose-6-phosphate dehydrogenase

How well did you know this?
1
Not at all
2
3
4
5
Perfectly
176
Q

Select the coupling enzyme used in the hexokinase method for glucose.

A. Glucose-6-phosphate dehydrogenase
B. Peroxidase
C. Glucose dehydrogenase
D. Glucose-6-phosphatase

A

C. Glucose dehydrogenase

How well did you know this?
1
Not at all
2
3
4
5
Perfectly
177
Q

Which of the following is a potential source of error in the hexokinase method?

A. Galactosemia
B. Hemolysis
C. Sample collected in fluoride
D. Ascorbic acid

A

B. Hemolysis

How well did you know this?
1
Not at all
2
3
4
5
Perfectly
178
Q

Which statement about glucose in cerebrospinal fluid (CSF) is correct?

A. Levels below 40 mg/dL occur in septic meningitis, cancer, and multiple sclerosis
B. CSF glucose is normally the same as the plasma glucose level
C. Hyperglycorrhachia is caused by dehydration
D. In some clinical conditions, the CSF glucose can be greater than the plasma glucose

A

A. Levels below 40 mg/dL occur in septic meningitis, cancer, and multiple sclerosis

How well did you know this?
1
Not at all
2
3
4
5
Perfectly
179
Q

In peroxidase-coupled glucose methods, which reagent complexes with the chromogen?

A. Nitroprusside
B. Phenol
C. Tartrate
D. Hydroxide

A

B. Phenol

How well did you know this?
1
Not at all
2
3
4
5
Perfectly
180
Q

Point-of-care-tests (POCTs) for whole-blood glucose monitoring are based mainly on the use of:

A. Glucose oxidase as the enzyme
B. Amperometric detection
C. Immunochromatography
D. Peroxidase coupling reactions

A

B. Amperometric detection

How well did you know this?
1
Not at all
2
3
4
5
Perfectly
181
Q

What effect does hematocrit have on POCT tests for whole-blood glucose monitoring?

A. Low hematocrit decreases glucose readings on all devices
B. High hematocrit raises glucose readings on all devices
C. The effect is variable and dependent on the enzyme/coenzyme system
D. Low hematocrit raises readings and high hematocrit lowers readings unless corrected

A

D. Low hematocrit raises readings and high hematocrit lowers readings unless corrected

How well did you know this?
1
Not at all
2
3
4
5
Perfectly
182
Q

Which of the following is classified as a mucopolysaccharide storage disease?

A. Pompe’s disease
B. von Gierke disease
C. Hers’ disease
D. Hurler’s syndrome

A

D. Hurler’s syndrome

How well did you know this?
1
Not at all
2
3
4
5
Perfectly
183
Q

Identify the enzyme deficiency responsible for type 1 glycogen storage disease (von Gierke’s disease).

A. Glucose-6-phosphatase
B. Glycogen phosphorylase
C. Glycogen synthetase
D. β-Glucosidase

A

A. Glucose-6-phosphatase

How well did you know this?
1
Not at all
2
3
4
5
Perfectly
184
Q

Which of the following abnormal laboratory results is found in von Gierke’s disease?

A. Hyperglycemia
B. Increased glucose response to epinephrine administration
C. Metabolic alkalosis
D. Hyperlipidemia

A

D. Hyperlipidemia

How well did you know this?
1
Not at all
2
3
4
5
Perfectly
185
Q

The D-xylose absorption test is used for the differential diagnosis of which two diseases?

A. Pancreatic insufficiency from malabsorption
B. Primary from secondary disorders of glycogen synthesis
C. Type 1 and type 2 diabetes mellitus
D. Generalized from specific carbohydrate intolerance

A

A. Pancreatic insufficiency from malabsorption

How well did you know this?
1
Not at all
2
3
4
5
Perfectly
186
Q

Which of the following statements aboutcarbohydrate intolerance is true?

A. Galactosemia results from deficiency of galactose-1-phosphate (galactose-1-PO4) uridine diphosphate transferase
B. Galactosemia results in a positive glucose oxidase test for glucose in urine
C. Urinary galactose is seen in both galactosemia and lactase deficiency
D. A galactose tolerance test is used to confirm a diagnosis of galactosemia

A

A. Galactosemia results from deficiency of galactose-1-phosphate (galactose-1-PO4) uridine diphosphate transferase

How well did you know this?
1
Not at all
2
3
4
5
Perfectly
187
Q

Which of the following statements regarding iron metabolism is correct?

A. Iron absorption is decreased by alcohol ingestion
B. Normally, 40%–50% of ingested iron is absorbed
C. The daily requirement is higher for pregnant and menstruating women
D. Absorption increases with the amount of iron in the body stores

A

C. The daily requirement is higher for pregnant and menstruating women

How well did you know this?
1
Not at all
2
3
4
5
Perfectly
188
Q

Which of the following statements regarding iron metabolism is correct?

A. Iron absorption is decreased by alcohol ingestion
B. Normally, 40%–50% of ingested iron is absorbed
C. The daily requirement is higher for pregnant and menstruating women
D. Absorption increases with the amount of iron in the body stores

A

B. Normally, 40%–50% of ingested iron is absorbed

How well did you know this?
1
Not at all
2
3
4
5
Perfectly
189
Q

Which of the following is associated with low serum iron and high total iron-binding capacity (TIBC)?

A. Iron deficiency anemia
B. Hepatitis
C. Nephrosis
D. Noniron deficiency anemias

A

A. Iron deficiency anemia

How well did you know this?
1
Not at all
2
3
4
5
Perfectly
190
Q

Which condition is associated with the lowest percent saturation of transferrin?

A. Hemochromatosis
B. Anemia of chronic infection
C. Iron deficiency anemia
D. Noniron deficiency anemia

A

C. Iron deficiency anemia

How well did you know this?
1
Not at all
2
3
4
5
Perfectly
191
Q

Which condition is most often associated with a high serum iron level?

A. Nephrosis
B. Chronic infection or inflammation
C. Polycythemia vera
D. Noniron deficiency anemias

A

D. Noniron deficiency anemias

How well did you know this?
1
Not at all
2
3
4
5
Perfectly
192
Q

Which of the following is likely to occur first in iron deficiency anemia?

A. Decreased serum iron
B. Increased TIBC
C. Decreased serum ferritin
D. Increased transferrin

A

C. Decreased serum ferritin

How well did you know this?
1
Not at all
2
3
4
5
Perfectly
193
Q

Which formula provides the best estimate of serum TIBC?

A. Serum transferrin in mg/dL × 0.70 = TIBC (μg/dL)
B. Serum transferrin in mg/dL × 1.43 = TIBC (μg/dL)
C. Serum iron (μg/dL)/1.2 + 0.06 = TIBC (μg/dL)
D. Serum Fe (μg/dL) × 1.25 = TIBC (μg/dL)

A

B. Serum transferrin in mg/dL × 1.43 = TIBC (μg/dL)

How well did you know this?
1
Not at all
2
3
4
5
Perfectly
194
Q

Which statement regarding the diagnosis of iron deficiency is correct?

A. Serum iron levels are always higher at night than during the day
B. Serum iron levels begin to fall before the body stores become depleted
C. A normal level of serum ferritin rules out iron deficiency
D. A low serum ferritin is diagnostic of iron deficiency

A

D. A low serum ferritin is diagnostic of iron deficiency

How well did you know this?
1
Not at all
2
3
4
5
Perfectly
195
Q

Which statement about iron methods is true?

A. Interference from Hgb can be corrected by a serum blank
B. Colorimetric methods measure binding of Fe2+ to a ligand such as ferrozine
C. Atomic absorption is the method of choice for measurement of serum iron
D. Serum iron can be measured by potentiometry

A

B. Colorimetric methods measure binding of Fe2+ to a ligand such as ferrozine

How well did you know this?
1
Not at all
2
3
4
5
Perfectly
196
Q

Which of the following statements regarding the TIBC assay is correct?

A. All TIBC methods require addition of excess iron to saturate transferrin
B. All methods require the removal of unbound iron
C. Measurement of TIBC is specific for transferrin-bound iron
D. The chromogen used must be different from the one used for measuring serum iron

A

A. All TIBC methods require addition of excess iron to saturate transferrin

How well did you know this?
1
Not at all
2
3
4
5
Perfectly
197
Q

Which of the following statements regarding the metabolism of bilirubin is true?

A. It is formed by hydrolysis of the α methene bridge of urobilinogen
B. It is reduced to biliverdin prior to excretion
C. It is a by-product of porphyrin production
D. It is produced from the destruction of RBCs

A

D. It is produced from the destruction of RBCs

How well did you know this?
1
Not at all
2
3
4
5
Perfectly
198
Q

Bilirubin is transported from reticuloendothelial cells to the liver by:

A. Albumin
B. Bilirubin-binding globulin
C. Haptoglobin
D. Transferrin

A

A. Albumin

How well did you know this?
1
Not at all
2
3
4
5
Perfectly
199
Q

In the liver, bilirubin is conjugated by addition of:

A. Vinyl groups
B. Methyl groups
C. Hydroxyl groups
D. Glucuronyl groups

A

D. Glucuronyl groups

How well did you know this?
1
Not at all
2
3
4
5
Perfectly
200
Q

Which enzyme is responsible for the conjugation of bilirubin?

A. β-Glucuronidase
B. UDP-glucuronyl transferase
C. Bilirubin oxidase
D. Biliverdin reductase

A

B. UDP-glucuronyl transferase

How well did you know this?
1
Not at all
2
3
4
5
Perfectly
201
Q

The term δ-bilirubin refers to:

A. Water-soluble bilirubin
B. Free unconjugated bilirubin
C. Bilirubin tightly bound to albumin
D. Direct-reacting bilirubin

A

C. Bilirubin tightly bound to albumin

How well did you know this?
1
Not at all
2
3
4
5
Perfectly
202
Q

Which of the following processes is part of the normal metabolism of bilirubin?

A. Both conjugated and unconjugated bilirubin are excreted into the bile
B. Methene bridges of bilirubin are reduced by intestinal bacteria forming urobilinogens
C. Most of the bilirubin delivered into the intestine is reabsorbed
D. Bilirubin and urobilinogen reabsorbed from the intestine are mainly excreted by the kidneys

A

B. Methene bridges of bilirubin are reduced by intestinal bacteria forming urobilinogens

How well did you know this?
1
Not at all
2
3
4
5
Perfectly
203
Q

Which of the following is a characteristic of conjugated bilirubin?

A. It is water soluble
B. It reacts more slowly than unconjugated bilirubin
C. It is more stable than unconjugated bilirubin
D. It has the same absorbance properties as unconjugated bilirubin

A

A. It is water soluble

How well did you know this?
1
Not at all
2
3
4
5
Perfectly
204
Q

Which of the following statements regarding urobilinogen is true?

A. It is formed in the intestines by bacterial reduction of bilirubin
B. It consists of a single water-soluble bile pigment
C. It is measured by its reaction with p-aminosalicylate
D. In hemolytic anemia, it is decreased in urine and feces

A

A. It is formed in the intestines by bacterial reduction of bilirubin

How well did you know this?
1
Not at all
2
3
4
5
Perfectly
205
Q

Which statement regarding bilirubin metabolism is true?

A. Bilirubin undergoes rapid photo-oxidation when exposed to daylight
B. Bilirubin excretion is inhibited by barbiturates
C. Bilirubin excretion is increased by chlorpromazine
D. Bilirubin is excreted only as the diglucuronide

A

A. Bilirubin undergoes rapid photo-oxidation when exposed to daylight

How well did you know this?
1
Not at all
2
3
4
5
Perfectly
206
Q

Which condition is caused by deficient secretion of bilirubin into the bile canaliculi?

A. Gilbert’s disease
B. Neonatal hyperbilirubinemia
C. Dubin–Johnson syndrome
D. Crigler–Najjar syndrome

A

C. Dubin–Johnson syndrome

How well did you know this?
1
Not at all
2
3
4
5
Perfectly
207
Q

In hepatitis, the rise in serum conjugated bilirubin can be caused by:

A. Secondary renal insufficiency
B. Failure of the enterohepatic circulation
C. Enzymatic conversion of urobilinogen to bilirubin
D. Extrahepatic conjugation

A

B. Failure of the enterohepatic circulation

How well did you know this?
1
Not at all
2
3
4
5
Perfectly
208
Q

Which of the following is a characteristic of obstructive jaundice?

A. The ratio of direct to total bilirubin is greater than 1:2
B. Conjugated bilirubin is elevated, but unconjugated bilirubin is normal
C. Urinary urobilinogen is increased
D. Urinary bilirubin is normal

A

A. The ratio of direct to total bilirubin is greater than 1:2

How well did you know this?
1
Not at all
2
3
4
5
Perfectly
209
Q

Which of the following would cause an increase in only the unconjugated bilirubin?

A. Hemolytic anemia
B. Obstructive jaundice
C. Hepatitis
D. Hepatic cirrhosis

A

A. Hemolytic anemia

How well did you know this?
1
Not at all
2
3
4
5
Perfectly
210
Q

Which form of hyperbilirubinemia is caused by an inherited absence of UDP-glucuronyl transferase?

A. Gilbert’s syndrome
B. Rotor syndrome
C. Crigler–Najjar syndrome
D. Dubin–Johnson syndrome

A

C. Crigler–Najjar syndrome

How well did you know this?
1
Not at all
2
3
4
5
Perfectly
211
Q

Which statement regarding total and direct bilirubin levels is true?

A. Total bilirubin level is a less sensitive and specific marker of liver disease than the direct level
B. Direct bilirubin exceeds 3.5 mg/dL in most cases of hemolytic anemia
C. Direct bilirubin is normal in cholestatic liver disease
D. The ratio of direct to total bilirubin exceeds 0.40 in hemolytic anemia

A

A. Total bilirubin level is a less sensitive and specific marker of liver

How well did you know this?
1
Not at all
2
3
4
5
Perfectly
212
Q

Which statement best characterizes serum bilirubin levels in the first week following delivery?

A. Serum bilirubin 24 hours after delivery should not exceed the upper reference limit for adults
B. Jaundice is usually first seen 48–72 hours postpartum in neonatal hyperbilirubinemia
C. Serum bilirubin above 5.0 mg/dL occurring 2–5 days after delivery indicates hemolytic or
hepatic disease
D. Conjugated bilirubin accounts for about 50% of the total bilirubin in neonates

A

B. Jaundice is usually first seen 48–72 hours postpartum in neonatal hyperbilirubinemia

How well did you know this?
1
Not at all
2
3
4
5
Perfectly
213
Q

Which form of jaundice occurs within days of delivery and usually lasts 1–3 weeks, but is not
due to normal neonatal hyperbilirubinemia or hemolytic disease of the newborn?

A. Gilbert syndrome
B. Lucey –Driscoll syndrome
C. Rotor syndrome
D. Dubin–Johnson syndrome

A

B. Lucey –Driscoll syndrome

How well did you know this?
1
Not at all
2
3
4
5
Perfectly
214
Q

A lab measures total bilirubin by the Jendrassik–Grof bilirubin method with sample blanking. What would be the effect of moderate hemolysis on the test result?

A. Falsely increased due to optical interference
B. Falsely increased due to release of bilirubin from RBCs
C. Falsely low due to inhibition of the diazo reaction by hemoglobin
D. No effect due to correction of positive interference by sample blanking

A

C. Falsely low due to inhibition of the diazo reaction by hemoglobin

How well did you know this?
1
Not at all
2
3
4
5
Perfectly
215
Q

Which reagent is used in the Jendrassik–Grof method to solubilize unconjugated bilirubin?

A. 50% methanol
B. N-butanol
C. Caffeine
D. Acetic acid

A

C. Caffeine

How well did you know this?
1
Not at all
2
3
4
5
Perfectly
216
Q

Which statement about colorimetric bilirubin methods is true?

A. Direct bilirubin must react with diazo reagent under alkaline conditions
B. Most methods are based upon reaction with diazotized sulfanilic acid
C. Ascorbic acid can be used to eliminate interference caused by Hgb
D. The color of the azobilirubin product is independent of pH

A

B. Most methods are based upon reaction with diazotized sulfanilic acid

How well did you know this?
1
Not at all
2
3
4
5
Perfectly
217
Q

Which statement regarding the measurement of bilirubin by the Jendrassik–Grof method is
correct?

A. The same diluent is used for both total and direct assays to minimize differences in reactivity
B. Positive interference by Hgb is prevented by the addition of HCl after the diazo reaction
C. The color of the azobilirubin product is intensified by the addition of ascorbic acid
D. Fehling’s reagent is added after the diazo reaction to reduce optical interference by hemoglobin

A

D. Fehling’s reagent is added after the diazo reaction to reduce optical interference by hemoglobin

How well did you know this?
1
Not at all
2
3
4
5
Perfectly
218
Q

A neonatal bilirubin assay performed at the nursery by bichromatic direct spectrophotometry
is 4.0 mg/dL. Four hours later, a second sample assayed for total bilirubin by the Jendrassik–Grof
method gives a result of 3.0 mg/dL. Both samples are reported to be hemolyzed. What is the most
likely explanation of these results?

A. Hgb interference in the second assay
B. δ-Bilirubin contributing to the result of the first assay
C. Falsely high results from the first assay caused by direct bilirubin
D. Physiological variation owing to premature hepatic microsomal enzymes

A

A. Hgb interference in the second assay

How well did you know this?
1
Not at all
2
3
4
5
Perfectly
219
Q

In the enzymatic assay of bilirubin, how is measurement of both total and direct bilirubin
accomplished?

A. Using different pH for total and direct assays
B. Using UDP glucuronyl transferase and bilirubin reductase
C. Using different polarity modifiers
D. Measuring the rate of absorbance decrease at different time intervals

A

A. Using different pH for total and direct assays

How well did you know this?
1
Not at all
2
3
4
5
Perfectly
220
Q

What is the principle of the transcutaneous bilirubin assay?

A. Conductivity
B. Amperometric inhibition
C. Multiwavelength reflectance photometry
D. Infrared spectroscopy

A

C. Multiwavelength reflectance photometry

How well did you know this?
1
Not at all
2
3
4
5
Perfectly
221
Q

How many grams of sodium hydroxide (NaOH) are required to prepare 150.0 mL of a 5.0% w/v
solution?

A. 1.5 g
B. 4.0 g
C. 7.5 g
D. 15.0 g

A

C. 7.5 g

How well did you know this?
1
Not at all
2
3
4
5
Perfectly
222
Q

How many milliliters of glacial acetic acid are needed to prepare 2.0 L of 10.0% v/v acetic acid?

A. 10.0 mL
B. 20.0 mL
C. 100.0 mL
D. 200.0 mL

A

D. 200.0 mL

How well did you know this?
1
Not at all
2
3
4
5
Perfectly
223
Q

A biuret reagent requires preparation of a stock solution containing 9.6 g of copper II sulfate (CuSO4) per liter. How many grams of CuSO4 • 5H2O are needed to prepare 1.0 L of the stock solution?
Atomic weights:
H = 1.0;
Cu = 63.6;
O = 16.0;
S = 32.1

A. 5.4 g
B. 6.1 g
C. 15.0 g
D. 17.0 g

A

C. 15.0 g

How well did you know this?
1
Not at all
2
3
4
5
Perfectly
224
Q

How many milliliters of HNO3 (purity 68.0%, specific gravity 1.42) are needed to prepare 1.0 L
of a 2.0 N solution?
Atomic weights: H = 1.0; N = 14.0; O = 16.0

A. 89.5 mL
B. 126.0 mL
C. 130.5 mL
D. 180.0 mL

A

C. 130.5 mL

How well did you know this?
1
Not at all
2
3
4
5
Perfectly
225
Q

Convert 10.0 mg/dL calcium (atomic weight = 40.1) to International System of Units (SI).

A. 0.25
B. 0.40
C. 2.5
D. 0.4

A

C. 2.5

How well did you know this?
1
Not at all
2
3
4
5
Perfectly
226
Q

Convert 2.0 mEq/L magnesium (atomic weight = 24.3) to milligrams per deciliter.

A. 0.8 mg/dL
B. 1.2 mg/dL
C. 2.4 mg/dL
D. 4.9 mg/dL

A

C. 2.4 mg/dL

How well did you know this?
1
Not at all
2
3
4
5
Perfectly
227
Q

How many milliliters of a 2,000.0 mg/dL glucose stock solution are needed to prepare 100.0 mL of a 150.0 mg/dL glucose working standard?

A. 1.5 mL
B. 7.5 mL
C. 15.0 mL
D. 25.0 mL

A

B. 7.5 mL

How well did you know this?
1
Not at all
2
3
4
5
Perfectly
228
Q

What is the pH of a solution of HNO3, if the hydrogen ion concentration is 2.5 × 10–2 M?

A. 1.0
B. 1.6
C. 2.5
D. 2.8

A

B. 1.6

How well did you know this?
1
Not at all
2
3
4
5
Perfectly
229
Q

Calculate the pH of a solution of 1.5 × 10–5 M NH4OH.

A. 4.2
B. 7.2
C. 9.2
D. 11.2

A

C. 9.2

How well did you know this?
1
Not at all
2
3
4
5
Perfectly
230
Q

How many significant figures should be reported when the pH of a 0.060 M solution of nitric acid
is calculated?

A. 1
B. 2
C. 3
D. 4

A

B. 2

How well did you know this?
1
Not at all
2
3
4
5
Perfectly
231
Q

What is the pH of a 0.05 M solution of acetic acid? Ka = 1.75 × 10–5, pKa = 4.76

A. 1.7
B. 3.0
C. 4.3
D. 4.6

A

B. 3.0

How well did you know this?
1
Not at all
2
3
4
5
Perfectly
232
Q

What is the pH of a buffer containing 40.0 mmol/L NaHC2O4 and 4.0 mmol/L H2C2O4? (pKa = 1.25)

A. 1.35
B. 2.25
C. 5.75
D. 6.12

A

B. 2.25

How well did you know this?
1
Not at all
2
3
4
5
Perfectly
233
Q

A solvent needed for HPLC requires a 20.0 mmol/L phosphoric acid buffer, pH 3.50, made by mixing KH2PO4 and H3PO4. How many grams of KH2PO4 are required to make 1.0 L of this
buffer? Formula weights: KH2PO4 = 136.1; H3PO4 = 98.0; pKa H3PO4 = 2.12

A. 1.96 g
B. 2.61 g
C. 2.72 g
D. 19.2 g

A

B. 2.61 g

How well did you know this?
1
Not at all
2
3
4
5
Perfectly
234
Q

A procedure for cholesterol is calibrated with a serum-based cholesterol standard that was
determined by the Abell–Kendall method to be 200.0 mg/dL. Assuming the same volume of sample and reagent are used, calculate the cholesterol concentration in the patient’s sample from the following results.
Standard Concentration = 200 mg/dL
Absorbance of Reagent Blank = 0.00
Absorbance of Standard = 0.860
Absorbance of Patient Serum = 0.740

A. 123 mg/dL
B. 172 mg/dL
C. 232 mg/dL
D. 314 mg/dL

A

B. 172 mg/dL

How well did you know this?
1
Not at all
2
3
4
5
Perfectly
235
Q

A glycerol kinase method for triglyceride calls for a serum blank in which normal saline is substituted for lipase in order to measure endogenous glycerol. Given the following results, and assuming the same volume of sample and reagent are used for each test, calculate the triglyceride concentration in the patient’s sample.

A. 119 mg/dL
B. 131 mg/dL
C. 156 mg/dL
D. 180 mg/dL

A

B. 131 mg/dL

How well did you know this?
1
Not at all
2
3
4
5
Perfectly
236
Q

A procedure for aspartate aminotransferase (AST) is performed manually because of a repeating error code for nonlinearity obtained on the laboratory’s automated chemistry analyzer; 0.05 mL of serum and 1.0 mL of substrate are used. The reaction rate is measured at 30°C at 340 nm using a 1.0 cM light path, and the delta absorbance (-ΔA) per minute is determined to be 0.382. Based upon a molar absorptivity coefficient for NADH at 340 nm of 6.22 X 103 M–1 cM–1 L–1, calculate the enzyme activity in international units (IUs) per liter.

A. 26 IU/L
B. 326 IU/L
C. 1228 IU/L
D. 1290 IU/L

A

D. 1290 IU/L

How well did you know this?
1
Not at all
2
3
4
5
Perfectly
237
Q

When referring to quality control (QC) results, what parameter usually determines the acceptable
range?

A. The 95% confidence interval for the mean
B. The range that includes 50% of the results
C. The central 68% of results
D. The range encompassed by ±2.5 standard deviations

A

A. The 95% confidence interval for the mean

How well did you know this?
1
Not at all
2
3
4
5
Perfectly
238
Q

Which of the following quality control (QC) rules would be broken 1 out of 20 times by chance alone?

A. 12s
B. 22s
C. 13s
D. 14s

A

A. 12s

How well did you know this?
1
Not at all
2
3
4
5
Perfectly
239
Q

Which of the following conditions is cause for rejecting an analytical run?

A. Two consecutive controls greater than 2 s above or below the mean
B. Three consecutive controls greater than 1 s above the mean
C. Four controls steadily increasing in value but less than ±1 s from the mean
D. One control above +1 s and the other below –1 s from the mean

A

A. Two consecutive controls greater than 2 s above or below the mean

How well did you know this?
1
Not at all
2
3
4
5
Perfectly
240
Q

One of two controls within a run is above +2s and the other control is below –2s from the mean.
What do these results indicate?

A. Poor precision has led to random error (RE)
B. A systematic error (SE) is present
C. Proportional error is present
D. QC material is contaminated

A

A. Poor precision has led to random error (RE)

How well did you know this?
1
Not at all
2
3
4
5
Perfectly
241
Q

Two consecutive controls are both beyond –2s from the mean. How frequently would this occur
on the basis of chance alone?

A. 1:100
B. 5:100
C. 1:400
D. 1:1,600

A

D. 1:1,600

How well did you know this?
1
Not at all
2
3
4
5
Perfectly
242
Q

The term R4S means that:

A. Four consecutive controls are greater than ±1
standard deviation from the mean
B. Two controls in the same run are greater than
4s units apart
C. Two consecutive controls in the same run are
each greater than ±4s from the mean
D. There is a shift above the mean for four
consecutive controls

A

B. Two controls in the same run are greater than
4s units apart

How well did you know this?
1
Not at all
2
3
4
5
Perfectly
243
Q

A trend in QC results is most likely caused by:

A. Deterioration of the reagent
B. Miscalibration of the instrument
C. Improper dilution of standards
D. Electronic noise

A

A. Deterioration of the reagent

How well did you know this?
1
Not at all
2
3
4
5
Perfectly
244
Q

In most circumstances, when two controls within a
run are both greater than ±2s from the mean, what
action should be taken first?

A. Recalibrate, then repeat controls followed by
selected patient samples if quality control is
acceptable
B. Repeat the controls before taking any corrective
action
C. Change the reagent lot, then recalibrate
D. Prepare fresh standards and recalibrate

A

A. Recalibrate, then repeat controls followed by
selected patient samples if quality control is
acceptable

245
Q

When establishing QC limits, which of the
following practices is inappropriate?

A. Using last month’s QC data to determine
current target limits
B. Exclusion of any QC results greater than ±2s
from the mean
C. Using control results from all shifts on which the
assay is performed
D. Using limits determined by reference laboratories
using the same method

A

B. Exclusion of any QC results greater than ±2s
from the mean

246
Q

Which of the following assays has the poorest
precision?
Mean Standard
Analyte (mmol/L) Deviation
A. Ca 2.5 0.3
B. K 4.0 0.4
C. Na 140 4.0
D. Cl 100 2.5

A

A.Ca Mean: 2.5 mmol/L Deviation: 0.3

247
Q

Given the following data, calculate the coefficient
of variation for glucose.
Glucose mean = 76mg/dL; SD = 2.3

A. 3.0%
B. 4.6%
C. 7.6%
D. 33.0%

A

A. 3.0%

248
Q

Which of the following plots is best for detecting
all types of QC errors?

A. Levy–Jennings
B. Tonks–Youden
C. Cusum
D. Linear regression

A

A. Levy–Jennings

249
Q

Which of the following plots is best for
comparison of precision and accuracy among
laboratories?

A. Levy–Jennings
B. Tonks–Youden
C. Cusum
D. Linear regression

A

B. Tonks–Youden

250
Q

Which plot will give the earliest indication of a
shift or trend?

A. Levy–Jennings
B. Tonks–Youden
C. Cusum
D. Histogram

A

C. Cusum

251
Q

All of the following are requirements for a
QC material except:

A. Long-term stability
B. The matrix is similar to the specimens being
tested
C. The concentration of analytes reflects the clinical
range
D. Analyte concentration must be independent of
the method of assay

A

D. Analyte concentration must be independent of
the method of assay

252
Q

What is the minimum requirement for performing
QC for a total protein assay?

A. One level assayed every 8 hours
B. Two levels assayed within 8 hours
C. Two levels assayed within 24 hours
D. Three levels assayed within 24 hours

no. 37 chap 5.4

A

C. Two levels assayed within 24 hours

253
Q

Which of the following statistical tests is used to
compare the means of two methods?

A. Student’s t test
B. F distribution
C. Correlation coefficient (r)
D. Linear regression analysis

A

A. Student’s t test

254
Q

Two freezing point osmometers are compared by
running 40 paired patient samples one time on
each instrument, and the following results are
obtained:
Osmometer A Mean = 280 mOsm/kg; SD = 3.1
Osmometer B Mean = 294 mOsm/kg; SD = 2.8

If the critical value for F = 2.8, then what
conclusion can be drawn regarding the
precision of the two instruments?

A. There is no statistically significant difference in
precision
B. Osmometer A demonstrates better precision that
is statistically significant
C. Osmometer B demonstrates better precision that
is statistically significant
D. Precision cannot be evaluated statistically when
single measurements are made on samples

A

A. There is no statistically significant difference in
precision

255
Q

Two methods for total cholesterol are compared
by running 40 paired patient samples in duplicate
on each instrument. The following results are
obtained:
Method x (reference method) Mean = 235 mg/dL; SD = 3.8
Method y(candidate method) Mean = 246 mg/dL; SD = 3.4

Assuming the samples are collected and stored
in the same way and the analysis done by a
technologist who is familiar with both methods,
what is the bias of method y?

A. 0.4
B. 7.2
C. 10.6
D. 11.0

A

D. 11.0

256
Q

When the magnitude of error increases with
increasing sample concentration, it is called:

A. Constant error
B. Proportional error
C. Random error
D. Bias

A

B. Proportional error

257
Q

In addition to the number of true negatives (TN),
which of the following measurements is needed to
calculate specificity?

A. True positives
B. Prevalence
C. False negatives
D. False positives

no. 46 chap 5.4

A

D. False positives

258
Q

A new tumor marker for ovarian cancer is
evaluated for sensitivity by testing serum samples
from patients who have been diagnosed by staging
biopsy as having malignant or benign lesions.
The following results were obtained:
Number of malignant patients who are positive for
CA 125 = 21 out of 24
Number of benign patients who are negative for
CA 125 = 61 out of 62

What is the sensitivity of the new CA 125 test?

A. 98.4%
B. 95.3%
C. 87.5%
D. 85.0%

A

C. 87.5%

259
Q

A new test for prostate cancer is found to have a
sensitivity of 80.0% and a specificity of 84.0%. If
the prevalence of prostate cancer is 4.0% in men
over 42 years old, what is the predictive value of a
positive test result (PV+) in this group?

A. 96.0%
B. 86.0%
C. 32.4%
D. 17.2%

A

D. 17.2%

260
Q

What measurement in addition to true negatives
and prevalence is required to calculate the
predictive value of a negative test result (PV–)?

A. False negatives
B. Variance
C. True positives
D. False positives

A

A. False negatives

261
Q

A laboratory is establishing a reference range for a
new analyte and wants the range to be determined
by the regional population of adults age 18 and
older. The analyte concentration is known to be
independent of race and gender. Which is the
most appropriate process to follow?

A. Determine the mean and standard deviation of
the analyte from 40 healthy adults and calculate
the ±2s limit
B. Measure the analyte in 120 healthy adults and
calculate the central 95th percentile
C. Measure the analyte in 120 healthy adults and
use the lowest and highest as the reference range
limits
D. Measure the analyte in 60 healthy adults and
60 adults with conditions that affect the analyte
concentration; calculate the concentration of
least overlap

A

B. Measure the analyte in 120 healthy adults and calculate the central 95th percentile

262
Q

When comparing the laboratory’s monthly mean
to its peer group to determine if bias is present,
what statistic is most appropriate?

A. F test
B. Linear regression analysis
C. Correlation coefficient
D. Standard deviation index

A

D. Standard deviation index

263
Q

Which of the following methods is most useful
in order to detect sample misidentification?

A. Cumulative summation
B. Critical limit
C. Delta limit
D. Significant change limit

A

C. Delta limit

264
Q

Which of the following total quality management
tools can be used to calculate the analytical error
rate for an analyte in the clinical laboratory?

A. LEAN
B. Six sigma
C. ISO 9000
D. Laboratory information system

A

B. Six sigma

265
Q

In which circumstances is a validation study
(versus performing routine quality control)
required?

A. Instrument recalibration
B. Source lamp or ion selective electrode change
C. Change in reagent lot
D. Change in calibrator lot
Chemistry/Apply principles of

A

C. Change in reagent lot

266
Q

Creatinine is formed from the:

A. Oxidation of creatine
B. Oxidation of protein
C. Deamination of dibasic amino acids
D. Metabolism of purines

no.1 chap 5.5

A

A. Oxidation of creatine

267
Q

Creatinine is considered the substance of choice to
measure endogenous renal clearance because:

A. The rate of formation per day is independent of
body size
B. It is completely filtered by the glomeruli
C. Plasma levels are highly dependent upon diet
D. Clearance is the same for both men and women

A

B. It is completely filtered by the glomeruli

268
Q

Which statement regarding creatinine is true?

A. Serum levels are elevated in early renal disease
B. High serum levels result from reduced
glomerular filtration
C. Serum creatine has the same diagnostic utility as
serum creatinine
D. Serum creatinine is a more sensitive measure of
renal function than creatinine clearance

A

B. High serum levels result from reduced

269
Q

Which of the following formulas is the correct
expression for creatinine clearance?

A. Creatinine clearance = U/P X V X 1.73/A
B. Creatinine clearance = P/V X U X A/1.73
C. Creatinine clearance = P/V X U X 1.73/A
D. Creatinine clearance = U/V X P X 1.73/A

A

A. Creatinine clearance = U/P X V X 1.73/A

270
Q

Which of the following conditions is most likely to
cause a falsely high creatinine clearance result?

A. The patient uses the midstream void procedure
when collecting his or her urine
B. The patient adds tap water to the urine container
because he or she forgets to save one of the urine
samples
C. The patient does not empty his or her bladder at
the conclusion of the test
D. The patient empties his or her bladder at the start
of the test and adds the urine to the collection

A

D. The patient empties his or her bladder at the start
of the test and adds the urine to the collection

271
Q

The modification of diet in renal disease (MDRD)
formula for calculating eGFR requires which four
parameters?

A. Urine creatinine, serum creatinine, height,
weight
B. Serum creatinine, age, gender, race
C. Serum creatinine, height, weight, age
D. Urine creatinine, gender, weight, age

A

B. Serum creatinine, age, gender, race

272
Q

What substance may be measured as an alternative
to creatinine for evaluating GFR?

A. Plasma urea
B. Cystatin C
C. Uric acid
D. Potassium

A

B. Cystatin C

273
Q

Which of the following enzymes allows creatinine
to be measured by coupling the creatinine
amidohydrolase (creatininase) reaction to the
peroxidase reaction?

A. Glucose-6-phosphate dehydrogenase
B. Creatinine iminohydrolase
C. Sarcosine oxidase
D. Creatine kinase

A

C. Sarcosine oxidase

274
Q

Select the primary reagent used in the Jaffe
method for creatinine.

A. Alkaline copper II sulfate
B. Saturated picric acid and NaOH
C. Sodium nitroprusside and phenol
D. Phosphotungstic acid

A

B. Saturated picric acid and NaOH

275
Q

Interference from other reducing substances can be
partially eliminated in the Jaffe reaction by:

A. Measuring the product at 340 nm
B. Measuring the product with an electrode
C. Measuring the timed rate of product formation
D. Performing a sample blank

A

C. Measuring the timed rate of product formation

276
Q

Which of the following statements is true?

A. Cystatin C is measured immunochemically
B. The calibrator used for cystatin C is traceable to
the National Bureau of Standards calibrator
C. Cystatin C assays have a lower coefficient of
variation than plasma creatinine
D. Enzymatic and rate Jaffe reactions for creatinine
give comparable results

A

A. Cystatin C is measured immunochemically

277
Q

In which case would eGFR derived from the
plasma creatinine likely give a more accurate
measure of GFR than measurement of plasma
cystatin C?

A. Diabetic patient
B. Chronic renal failure
C. Post–renal transplant
D. Chronic hepatitis

A

C. Post–renal transplant

278
Q

A sample of amniotic fluid collected for fetal lung
maturity studies from a woman with a pregnancy
compromised by hemolytic disease of the newborn
(HDN) has a creatinine of 88 mg/dL. What is the
most likely cause of this result?

A. The specimen is contaminated with blood
B. Bilirubin has interfered with the measurement of
creatinine
C. A random error occurred when the absorbance
signal was being processed by the analyzer
D. The fluid is urine from accidental puncture of
the urinary bladder

A

D. The fluid is urine from accidental puncture of
the urinary bladder

279
Q

Which analyte should be reported as a ratio using
creatinine concentration as a reference?

A. Urinary microalbumin
B. Urinary estriol
C. Urinary sodium
D. Urinary urea

A

A. Urinary microalbumin

280
Q

Urea is produced from:

A. The catabolism of proteins and amino acids
B. Oxidation of purines
C. Oxidation of pyrimidines
D. The breakdown of complex carbohydrates

A

A. The catabolism of proteins and amino acids

281
Q

Urea concentration is calculated from the BUN by
multiplying by a factor of:

A. 0.5
B. 2.14
C. 6.45
D. 14

A

B. 2.14

282
Q

Which of the statements below about serum urea
is true?

A. Levels are independent of diet
B. Urea is not reabsorbed by the renal tubules
C. High BUN levels can result from necrotic liver
disease
D. BUN is elevated in prerenal as well as renal
failure

A

D. BUN is elevated in prerenal as well as renal
failure

283
Q

A patient’s BUN is 60 mg/dL and serum
creatinine is 3.0 mg/dL. These results suggest:

A. Laboratory error measuring BUN
B. Renal failure
C. Prerenal failure
D. Patient was not fasting

A

C. Prerenal failure

284
Q

Urinary urea measurements may be used for
calculation of:

A. Glomerular filtration
B. Renal blood flow
C. Nitrogen balance
D. All of these options

A

C. Nitrogen balance

285
Q

BUN is determined electrochemically by coupling
the urease reaction to measurement of:

A. Potential with a urea-selective electrode
B. The timed rate of increase in conductivity
C. The oxidation of ammonia
D. Carbon dioxide

A

B. The timed rate of increase in conductivity

286
Q

In the ultraviolet enzymatic method for BUN, the
urease reaction is coupled to a second enzymatic
reaction using:

A. AST
B. Glutamate dehydrogenase
C. Glutamine synthetase
D. Alanine aminotransferase (ALT)

A

B. Glutamate dehydrogenase

287
Q

Which product is measured in the coupling step of
the urease-UV method for BUN?

A. CO2
B. Dinitrophenylhydrazine
C. Diphenylcarbazone
D. NAD+

A

D. NAD+

288
Q

Which enzyme deficiency is responsible for
phenylketonuria (PKU)?

A. Phenylalanine hydroxylase
B. Tyrosine transaminase
C. p-Hydroxyphenylpyruvic acid oxidase
D. Homogentisic acid oxidase

A

A. Phenylalanine hydroxylase

289
Q

Which of the following conditions is classified as a
renal-type aminoaciduria?

A. Fanconi syndrome
B. Wilson’s disease
C. Hepatitis
D. Homocystinuria

A

A. Fanconi syndrome

290
Q

Which aminoaciduria results in the overflow of
branched chain amino acids?

A. Hartnup’s disease
B. Alkaptonuria
C. Homocystinuria
D. Maple syrup urine disease

A

D. Maple syrup urine disease

291
Q

In addition to phenylketonuria, maple syrup
urine disease, and homocystinuria, what other
aminoaciduria can be detected by tandem MS?

A. Alkaptonuria
B. Hartnup disease
C. Citrullinemia
D. Cystinuria

A

C. Citrullinemia

292
Q

Of the methods used to measure amino acids,
which is capable of measuring fatty acids
simultaneously?

A. Tandem-mass spectroscopy
B. High-performance liquid chromatography
C. Capillary electrophoresis
D. Two-dimensional thin-layer chromatography

A

A. Tandem-mass spectroscopy

293
Q

Blood ammonia levels are usually measured in
order to evaluate:

A. Renal failure
B. Acid–base status
C. Hepatic coma
D. Gastrointestinal malabsorption

A

C. Hepatic coma

294
Q

Enzymatic measurement of ammonia requires
which of the following substrates and coenzymes?

Substrate Coenzyme
A. α–Ketoglutarate NADH
B. Glutamate NADH
C. Glutamine ATP
D. Glutamine NAD+

A

A. Substrate: α–Ketoglutarate
Coenzyme: NADH

295
Q

Which statement about ammonia is true?

A. Normally, most of the plasma ammonia is
derived from peripheral blood deamination of
amino acids
B. Ammonia-induced coma can result from
salicylate poisoning
C. Hepatic coma can result from Reye’s syndrome
D. High plasma ammonia is usually caused by
respiratory alkalosis

A

C. Hepatic coma can result from Reye’s syndrome

296
Q

SITUATION: A sample for ammonia assay is
taken from an IV line that had been capped
and injected with lithium heparin (called a
heparin lock). The sample is drawn in a syringe
containing lithium heparin, and immediately
capped and iced. The plasma is separated and
analyzed within 20 minutes of collection, and
the result is 50 μg/dL higher than one measured
4 hours before. What is the most likely
explanation of these results?

A. Significantly greater physiological variation is
seen with patients having systemic, hepatic, and
gastrointestinal diseases
B. The syringe was contaminated with ammonia
C. One of the two samples was collected from the
wrong patient
D. Stasis of blood in the line caused increased
ammonia

A

D. Stasis of blood in the line caused increased
ammonia

297
Q

Uric acid is derived from the:

A. Oxidation of proteins
B. Catabolism of purines
C. Oxidation of pyrimidines
D. Reduction of catecholamines

A

B. Catabolism of purines

298
Q

Which of the following conditions is associated
with hyperuricemia?

A. Renal failure
B. Chronic liver disease
C. Xanthine oxidase deficiency
D. Paget’s disease of the bone

A

A. Renal failure

299
Q

Orders for uric acid are legitimate stat requests
because:

A. Levels above 10 mg/dL cause urinary tract calculi
B. Uric acid is hepatotoxic
C. High levels induce aplastic anemia
D. High levels cause joint pain

A

A. Levels above 10 mg/dL cause urinary tract calculi

300
Q

Which uric acid method is associated with
negative bias caused by reducing agents?

A. Uricase coupled to the Trinder reaction
B. Ultraviolet uricase reaction coupled to catalase
and alcohol dehydrogenase reactions
C. Measurement of the rate of absorbance decrease
at 290 nm after addition of uricase
D. Phosphotungstic acid using a protein-free filtrate

A

A. Uricase coupled to the Trinder reaction

301
Q

Kjeldahl’s procedure for total protein is based
upon the premise that:

A. Proteins are negatively charged
B. The pKa of proteins is the same
C. The nitrogen content of proteins is constant
D. Proteins have similar tyrosine and tryptophan
content

A

C. The nitrogen content of proteins is constant

302
Q

Upon which principle is the biuret method based?

A. The reaction of phenolic groups with CuIISO4
B. Coordinate bonds between Cu+2 and carbonyl
and imine groups of peptide bonds.
C. The protein error of indicator effect producing
color when dyes bind protein
D. The reaction of phosphomolybdic acid with
protein

A

B. Coordinate bonds between Cu+2 and carbonyl

303
Q

Which statement about the biuret reaction for
total protein is true?

A. It is sensitive to protein levels below 0.1 mg/dL
B. It is suitable for urine, exudates, and transudates
C. Polypeptides and compounds with repeating
imine groups react
D. Hemolysis will not interfere

A

C. Polypeptides and compounds with repeating
imine groups react

304
Q

Which of the following protein methods has the
highest analytical sensitivity?

A. Refractometry
B. Folin–Lowry
C. Turbidimetry
D. Direct ultraviolet absorption

A

B. Folin–Lowry

305
Q

Which of the following statements regarding
proteins is true?

A. Total protein and albumin are about 10% higher
in ambulatory patients
B. Plasma total protein is about 20% higher than
serum levels
C. Albumin normally accounts for about one-third
of the cerebrospinal fluid total protein
D. Transudative serous fluid protein is about
two-thirds of the serum total protein

A

A. Total protein and albumin are about 10% higher
in ambulatory patients

306
Q

Hyperalbuminemia is caused by:

A. Dehydration syndromes
B. Liver disease
C. Burns
D. Gastroenteropathy

A

A. Dehydration syndromes

307
Q

High serum total protein but low albumin is
usually seen in:

A. Multiple myeloma
B. Hepatic cirrhosis
C. Glomerulonephritis
D. Nephrotic syndrome

A

A. Multiple myeloma

308
Q

Which of the following conditions is most
commonly associated with an elevated level
of total protein?

A. Glomerular disease
B. Starvation
C. Liver failure
D. Malignancy

A

D. Malignancy

309
Q

Which of the following dyes is the most specific
for measurement of albumin?

A. Bromcresol green (BCG)
B. Bromcresol purple (BCP)
C. Tetrabromosulfophthalein
D. Tetrabromphenol blue

A

B. Bromcresol purple (BCP)

310
Q

Which of the following factors is most likely
to cause a falsely low result when using the
BCG dye-binding assay for albumin?

A. The presence of penicillin
B. An incubation time of 120 seconds
C. The presence of bilirubin
D. Lipemia

A

A. The presence of penicillin

311
Q

At pH 8.6, proteins are ________ charged and
migrate toward the _________.

A. Negatively, anode
B. Positively, cathode
C. Positively, anode
D. Negatively, cathode

A

A. Negatively, anode

312
Q

Electrophoretic movement of proteins toward the
anode will decrease by increasing the:

A. Buffer pH
B. Ionic strength of the buffer
C. Current
D. Voltage

A

B. Ionic strength of the buffer

313
Q

At pH 8.6, the cathodal movement of γ globulins
is caused by:

A. Electroendosmosis
B. Wick flow
C. A net positive charge
D. Cathodal sample application

A

A. Electroendosmosis

314
Q

Which of the following conditions will prevent
any migration of proteins across an electrophoretic
support medium such as agarose?

A. Using too high a voltage
B. Excessive current during the procedure
C. Loss of contact between a buffer chamber and
the medium
D. Evaporation of solvent from the surface of the
medium

A

C. Loss of contact between a buffer chamber and
the medium

315
Q

Which of the following proteins has the highest pI?

A. Albumin
B. Transferrin
C. Ceruloplasmin
D. IgG

A

D. IgG

316
Q

Which of the following proteins migrates in the
β region at pH 8.6?

A. Haptoglobin
B. Orosomucoprotein
C. Antichymotrypsin
D. Transferrin

A

D. Transferrin

317
Q

Which of the following is one advantage of
high-resolution (HR) agarose electrophoresis
over lower-current electrophoresis?

A. High-resolution procedures detect monoclonal
and oligoclonal bands at a lower concentration
B. A smaller sample volume is used
C. Results are obtained more rapidly
D. Densitometric scanning of HR gels is more
accurate

A

A. High-resolution procedures detect monoclonal
and oligoclonal bands at a lower concentration

318
Q

Which of the following conditions is associated
with “β-γ bridging”?

A. Multiple myeloma
B. Malignancy
C. Hepatic cirrhosis
D. Rheumatoid arthritis

A

C. Hepatic cirrhosis

319
Q

Which support medium can be used to determine
the molecular weight of a protein?

A. Cellulose acetate
B. Polyacrylamide gel
C. Agar gel
D. Agarose gel

A

B. Polyacrylamide gel

320
Q

Which of the following stains is used for
lipoprotein electrophoresis?

A. Oil Red O
B. Coomassie Brilliant Blue
C. Amido Black
D. Ponceau S

A

A. Oil Red O

321
Q

Which of the following serum protein
electrophoresis results suggests an acute
inflammatory process?

Albumin α1 α2 β γ
Decreased Increased Decreased Normal Normal
Normal Increased Normal Increased Increased
Decreased Increased Increased Normal Normal
Increased Increased Increased Increased Increase

A

C. Albumin - Decreased
α1 - Increased
α2 - Increased
β - Normal
γ - Normal

322
Q

Which of the following conditions is usually
associated with an acute inflammatory pattern?

A. Myocardial infarction (MI)
B. Malignancy
C. Rheumatoid arthritis
D. Hepatitis

A

A. Myocardial infarction (MI)

323
Q

What is the clinical utility of testing for serum
prealbumin?

A. Low levels are associated with increased free
cortisol
B. High levels are an indicator of acute
inflammation
C. Serial low levels indicate compromised
nutritional status
D. Levels correlate with glomerular injury in
patients with diabetes mellitus

A

C. Serial low levels indicate compromised
nutritional status

324
Q

Which serum protein should be measured in a
patient suspected of having Wilson’s disease?

A. Hemopexin
B. Alpha-1 antitrypsin
C. Haptoglobin
D. Ceruloplasmin

A

D. Ceruloplasmin

325
Q

A patient with hemolytic-uremic syndrome
associated with septicemia has a haptoglobin
level that is normal, although the plasma free
hemoglobin is elevated and hemoglobinuria is
present. Which test would be more appropriate
than haptoglobin to measure this patient’s
hemolytic episode?

A. Hemopexin
B. Alpha-1 antitrypsin
C. C-reactive protein
D. Transferrin

A

A. Hemopexin

326
Q

Quantitative determination of Hgb A2 and Hgb F
are best performed by:

A. High-performance liquid chromatography
B. Alkali denaturation
C. Electrophoresis
D. Direct bichromatic spectrophotometry

A

A. High-performance liquid chromatography

327
Q

Select the correct order of Hgb migration on
agarose or cellulose acetate at pH 8.6.

A. – C→F→S→A +
B. – S→C→A→F +
C. – C→S→F→A +
D. – S→F→A→C +

A

C. – C→S→F→A +

328
Q

Which of the following abnormal types of Hgb
migrates to the same position as Hgb S on agarose
or cellulose acetate at pH 8.6?

A. Hgb C
B. Hgb DPunjab
C. Hgb OArab
D. Hgb E

A

B. Hgb DPunjab

329
Q

Which Hgb is a β-δ chain hybrid and migrates to
the same position as Hgb S at pH 8.6?

A. Hgb CHarlem
B. HgbLepore
C. Hgb GPhiladelphia
D. Hgb DPunjab

A

B. HgbLepore

330
Q

Select the correct order of Hgb migration on
citrate agar at pH 6.2.

A. – F→S→C→A +
B. – F→A→S→C +
C. – A→S→F→C +
D. – A→C→S→F +

A

B. – F→A→S→C +

331
Q

Which Hgb separates from Hgb S on citrate (acid)
agar, but not agarose or cellulose acetate?

A. Hgb DPunjab
B. Hgb E
C. Hgb CHarlem (Georgetown)
D. Hgb OArab

A

A. Hgb DPunjab

332
Q

Which statement best describes immunofixation
electrophoresis (IEF)?

A. Proteins are separated by electrophoresis
followed by overlay of monospecific
anti-immunoglobulins
B. Proteins react with monospecific antisera
followed by electrophoresis
C. Antisera are electrophoresed, then diffused
against patient’s serum
D. Serum is electrophoresed; the separated
immunoglobulins diffuse against specific
antisera placed into troughs

A

A. Proteins are separated by electrophoresis

333
Q

In double immunodiffusion reactions, the
precipitin band is:

A. Invisible before the equivalence point is reached
B. Concave to the protein of greatest molecular
weight
C. Closest to the well containing the highest level of
antigen
D. Located in an area of antibody excess

A

B. Concave to the protein of greatest molecular
weight

334
Q

Which of the following statements regarding the
identification of monoclonal proteins by IFE is
true?

A. The monoclonal band must be present in the
γ region
B. When testing for a monoclonal gammopathy,
both serum and urine must be examined
C. A diagnosis of monoclonal gammopathy is based
upon quantitation of IgG, IgA, and IgM
D. A monoclonal band always indicates a malignant
disorder

A

B. When testing for a monoclonal gammopathy,
both serum and urine must be examined

335
Q

Which of the following statements regarding
paraproteins is true?

A. Oligoclonal banding is seen in the CSF of greater
than 90% of multiple sclerosis cases
B. The Bence–Jones protein heat test is
confirmatory for monoclonal light chains
C. Light chains found in urine are always derived
from monoclonal protein
D. The IgA band is usually cathodal to the IgG
precipitin band

A

A. Oligoclonal banding is seen in the CSF of greater
than 90% of multiple sclerosis cases

336
Q

Which statement regarding IFE is true?

A. Serum containing a monoclonal protein should
have a κ:λ ratio of 0.5
B. A monoclonal band seen with monospecific
antiserum should not be visible in the lane where
polyvalent antiserum or sulfosalicylic acid was
added
C. CSF should be concentrated 50- to100-fold
before performing IFE
D. When oligoclonal bands are seen in the CSF,
they must also be present in serum to indicate
multiple sclerosis

A

C. CSF should be concentrated 50- to100-fold
before performing IFE

337
Q

Which test is the most sensitive in detecting early
monoclonal gammopathies?

A. High-resolution serum protein electrophoresis
B. Urinary electrophoresis for monoclonal light
chains
C. Capillary electrophoresis of serum and urine
D. Serum-free light chain immunoassay

A

D. Serum-free light chain immunoassay

338
Q

Which test is the most useful way to evaluate the
response to treatment for multiple myeloma?

A. Measure of total immunoglobulin
B. Measurement of 24-hour urinary light chain
concentration (Bence–Jones protein)
C. Capillary electrophoresis of M-protein recurrence
D. Measurement of serum-free light chains

A

D. Measurement of serum-free light chains

339
Q

Which of the following is more commonly
associated with a nonmalignant form of
monoclonal gammopathy (MGUS)?

A. Bone marrow plasma cells comprise 20% of
nucleated cells
B. Monoclonal protein (M-protein) concentration
is 3.5 g/dL
C. M-protein is IgG
D. Age greater than 60 at the time of monoclonal
protein discovery

A

D. Age greater than 60 at the time of monoclonal
protein discovery

340
Q

Capillary electrophoresis differs from agarose gel
electrophoresis in which respect?

A. A stationary support is not used
B. An acidic buffer is used
C. A low voltage is used
D. Electroendosmosis does not occur

A

A. A stationary support is not used

341
Q

Select the order of mobility of lipoproteins
electrophoresed on cellulose acetate or agarose at
pH 8.6.

A. – Chylomicrons→pre-β →β→α+
B. – β→pre-β→α→chylomicrons +
C. – Chylomicrons →β→pre-β→α +
D. – α→β→pre-β→chylomicrons +

A

C. – Chylomicrons →β→pre-β→α +

342
Q

Following ultracentrifugation of plasma, which
fraction correlates with pre-β lipoprotein?

A. Very low-density lipoprotein (VLDL)
B. Low-density lipoprotein (LDL)
C. High-density lipoprotein (HDL)
D. Chylomicrons

A

A. Very low-density lipoprotein (VLDL)

343
Q

Select the lipoprotein fraction that carries most of
the endogenous triglycerides.

A. VLDL
B. LDL
C. HDL
D. Chylomicrons

A

A. VLDL

344
Q

The protein composition of HDL is what
percentage by weight?

A. Less than 2%
B. 25%
C. 50%
D. 90%

A

C. 50%

345
Q

Which apoprotein is inversely related to risk of
coronary heart disease?

A. Apoprotein A-I
B. Apoprotein B100
C. Apoprotein C-II
D. Apoprotein E4

A

A. Apoprotein A-I

346
Q

In familial β dyslipoproteinemia (formerly
type III hyperlipoproteinemia), which lipoprotein
accumulates?

A. Chylomicrons
B. VLDL
C. IDL
D. VLDL

A

C. IDL

347
Q

Which of the following mechanisms accounts
for the elevated plasma level of β lipoproteins
seen in familial hypercholesterolemia (formerly
type II hyperlipoproteinemia)?

A. Hyperinsulinemia
B. ApoB-100 receptor defect
C. ApoC-II activated lipase deficiency
D. ApoE3 deficiency

A

B. ApoB-100 receptor defect

348
Q

Which enzyme deficiency is most commonly
associated with familial hypertriglyceridemia
associated with fasting plasma cholomicrons
(formerly type I hyperlipoproteinemia)?

A. β Glucocerebrosidase deficiency
B. Post–heparin-activated lipoprotein lipase
deficiency
C. Apo-B deficiency
D. Apo-C-III deficiency

A

B. Post–heparin-activated lipoprotein lipase
deficiency

349
Q

Which of the following conditions is most
consistently associated with secondary
hypercholesterolemia?

A. Hypothyroidism
B. Pancreatitis
C. Oral contraceptive therapy
D. Diabetes mellitus

A

A. Hypothyroidism

350
Q

Which of the following is associated with Tangier
disease?

A. Apoprotein C-II deficiency
B. Homozygous apo-B100 deficiency
C. Apoprotein C-II activated lipase
D. Apoprotein A-I deficiency

A

D. Apoprotein A-I deficiency

351
Q

Which of the following statements is correct?

A. Both HDL and LDL are homogenous
B. There are several subfractions of LDL but
not HDL
C. There are several subfractions of HDL but
not LDL
D. There are several subfractions of both HDL
and LDL

A

D. There are several subfractions of both HDL
and LDL

352
Q

What is the lipid testing protocol for adults
recommended by the National Cholesterol
Education Program (NCEP) to evaluate risk for
atherosclerosis beginning at age 20?

A. Total cholesterol, fasting or nonfasting every year
B. Total cholesterol, fasting, every 2 years
C. Lipid profile, fasting, every 5 years
D. LDL cholesterol, fasting, every 2 years

A

C. Lipid profile, fasting, every 5 years

353
Q

What is the most appropriate fasting procedure
when a lipid study of triglyceride, total cholesterol,
HDL cholesterol, and LDL cholesterol tests are
ordered?

A. 8 hours; nothing but water allowed
B. 10 hours; water, smoking, coffee, tea (no sugar
or cream) allowed
C. 12 hours; nothing but water allowed
D. 16 hours; water, smoking, coffee, tea (no sugar
or cream) allowed

A

C. 12 hours; nothing but water allowed

354
Q

Treatment recommendations for patients with
coronary heart disease are based upon
measurement of which analyte?

A. HDL cholesterol
B. Apo-B100
C. LDL cholesterol
D. Total cholesterol

A

C. LDL cholesterol

355
Q

What is the HDL cholesterol cutpoint recommend
by NCEP?

A. <30 mg/dL
B. <40 mg/dL
C. <30 mg/dL for males and < 40 mg/dL
for females
D. <45 mg/dL for males and < 50 mg/dL
for females

A

B. <40 mg/dL

356
Q

An EDTA blood sample is collected from a
nonfasting person for a CBC. The physician
collected the sample from the femoral vein because
venipuncture from the arm was unsuccessful. He
called the lab 15 minutes after the sample arrived
and requested a lipid study including triglyceride,
total cholesterol, HDL cholesterol, and LDL
cholesterol. Which test results should be used to
evaluate the patient’s risk for coronary artery
disease?

A. Total cholesterol and LDL cholesterol
B. LDL cholesterol and triglyceride
C. Total cholesterol and HDL cholesterol
D. Total cholesterol and triglyceride

A

C. Total cholesterol and HDL cholesterol

357
Q

Which of the following diseases is caused by a
deficiency of sphingomyelinase?

A. Gaucher disease
B. Fabry disease
C. Niemann–Pick disease
D. Tay–Sachs disease

A

C. Niemann–Pick disease

358
Q

Which method is considered the candidate reference method for triglyceride measurement?

A. Glycerol kinase-ultraviolet
B. CDC modification of van Handel and Zilversmit
C. Hantzsch condensation
D. Glycerol kinase coupled to peroxidase

A

B. CDC modification of van Handel and Zilversmit

359
Q

Which of the following enzymes is common to all
enzymatic methods for triglyceride measurement?

A. Glycerol phosphate oxidase
B. Glycerol phosphate dehydrogenase
C. Glycerol kinase
D. Pyruvate kinase

A

C. Glycerol kinase

360
Q

Select the reagent needed in the coupling enzyme
reaction used to generate a colored product in the
cholesterol oxidase method for cholesterol.

A. Cholestahexaene
B. H2O2
C. 4-Aminoantipyrine
D. Cholest-4-ene-3-one

A

C. 4-Aminoantipyrine

361
Q

What is the purpose of the saponification step used
in the Abell–Kendall method for cholesterol
measurement?

A. Remove phospholipids
B. Reduce sterol molecules structurally similar to
cholesterol
C. Convert cholesterol esters to free cholesterol
D. Remove proteins that can interfere with color
formation

A

C. Convert cholesterol esters to free cholesterol

362
Q

Which of the following methods for HDL
cholesterol is the reference method?

A. Manganese–heparin
B. Magnesium–phosphotungstate
C. Magnesium–dextran
D. Ultracentrifugation

A

D. Ultracentrifugation

363
Q

Cholesterol esterase is used in enzymatic assays to:

A. Oxidize cholesterol to form peroxide
B. Hydrolyze fatty acids bound to the third carbon
atom of cholesterol
C. Separate cholesterol from apoproteins A-I
and A-II by hydrolysis
D. Reduce NAD+ to NADH

A

B. Hydrolyze fatty acids bound to the third carbon
atom of cholesterol

364
Q

Which of the following reagents is used in the
direct HDL cholesterol method?

A. Sulfated cyclodextrin
B. Magnesium sulfate and dextran sulfate
C. Anti-apoA-I
D. Manganese heparin
Chemistry/Apply knowled

A

A. Sulfated cyclodextrin

365
Q

What do “direct” or homogenous methods for
LDL cholesterol assay have in common?

A. They are inaccurate when plasma triglyceride is
above 250 mg/dL
B. All use a detergent to facilitate selective reactivity
with reagent enzymes
C. All use monoclonal antibodies to apo A1 and C
D. All are free of interference from abnormal
lipoproteins

A

B. All use a detergent to facilitate selective reactivity
with reagent enzymes

366
Q

Lipoprotein (a), or Lp(a), is significant when
elevated in serum because it:

A. Is an independent risk factor for atherosclerosis
B. Blocks the clearance of VLDLs
C. Displaces apo-AI from HDLs
D. Is linked closely to a gene for obesity

A

A. Is an independent risk factor for atherosclerosis

367
Q

Which type of dietary fatty acid is not associated
with an increase in serum LDL cholesterol
production?

A. Monounsaturated trans fatty acids
B. Saturated fatty acids
C. Monounsaturated cis fatty acids
D. Monounsaturated trans Ω-9 fatty acids

A

C. Monounsaturated cis fatty acids

368
Q

SITUATION: A lipemic specimen collected from
an adult after a 12-hour fast was assayed for total
cholesterol, triglycerides, and HDL cholesterol
using a direct HDL method. Following are the
results:
Total cholesterol = 220 mg/dL
HDL cholesterol = 40 mg/dL
Triglyceride = 420 mg/dL

The physician requests an LDL cholesterol assay
after receiving the results. How should the LDL
cholesterol be determined?

A. Dilute the specimen 1:10 and repeat all tests;
calculate LDL cholesterol using the Friedewald
equation
B. Perform a direct LDL cholesterol assay
C. Ultracentrifuge the sample and repeat the HDL
cholesterol on the infranate. Use the new result
to calculate the LDL cholesterol
D. Repeat the HDL cholesterol using the
manganese heparin precipitation method. Use
the new result to calculate the LDL cholesterol

A

B. Perform a direct LDL cholesterol assay

369
Q

A person has a fasting triglyceride level of
240 mg/dL. The physician wishes to know the
patient’s non-HDL cholesterol level. What
cholesterol fractions should be measured?

A. Total cholesterol and HDL cholesterol
B. Total cholesterol and LDL cholesterol
C. HDL cholesterol and LDL cholesterol
D. Total cholesterol and chylomicrons

A

A. Total cholesterol and HDL cholesterol

370
Q

An international unit (IU) of enzyme activity is the
quantity of enzyme that:

A. Converts 1 μmol of substrate to product per liter
B. Forms 1 mg of product per deciliter
C. Converts 1 μmol of substrate to product
per minute
D. Forms 1 μmol of product per liter

A

C. Converts 1 μmol of substrate to product
per minute

371
Q

Which of the following statements describes a
nonkinetic enzyme assay?

A. Initial absorbance is measured followed by a
second reading after 5 minutes
B. Absorbance is measured at 10-second intervals
for 100 seconds
C. Absorbance is monitored continuously for
1 minute using a chart recorder
D. Reflectance is measured from a xenon source
lamp pulsing at 60 Hz

A

A. Initial absorbance is measured followed by a
second reading after 5 minutes

372
Q

Which of the following statements regarding
enzymatic reactions is true?

A. The enzyme shifts the equilibrium of the reaction
to the right
B. The enzyme alters the equilibrium constant of
the reaction
C. The enzyme increases the rate of the reaction
D. The enzyme alters the energy difference between
reactants and products

A

C. The enzyme increases the rate of the reaction

373
Q

Which statement about enzymes is true?

A. An enzyme alters the Gibb’s free energy of the
reaction
B. Enzymes cause a reaction with a positive free
energy to occur spontaneously
C. An enzyme’s natural substrate has the highest Km
D. A competitive inhibitor will alter the apparent
Km of the reaction

A

D. A competitive inhibitor will alter the apparent
Km of the reaction

374
Q

Which substrate concentration is needed to
achieve zero-order conditions?

A. Greater than 99 × Km
B. [S] = Km
C. Less than 10 × Km
D. [S] = 0

A

A. Greater than 99 × Km

375
Q

Which of the following statements is true?

A. Apoenzyme + prosthetic group = holoenzyme
B. A coenzyme is an inorganic molecule required
for activity
C. Cofactors are as tightly bound to the enzyme as
prosthetic groups
D. All enzymes have optimal activity at pH 7.00

A

A. Apoenzyme + prosthetic group = holoenzyme

376
Q

Which of the following statements about
enzymatic reactions is true?

A. NADH has absorbance maximas at 340
and 366 nm
B. Enzyme concentration must be in excess to
achieve zero-order kinetics
C. Rate is proportional to substrate concentration in
a zero-order reaction
D. Accumulation of the product increases the
reaction rate

A

A. NADH has absorbance maximas at 340
and 366 nm

377
Q

The increase in the level of serum enzymes used to
detect cholestatic liver disease is caused mainly by:

A. Enzyme release from dead cells
B. Leakage from cells with altered membrane
permeability
C. Decreased perfusion of the tissue
D. Increased production and secretion by cells

A

D. Increased production and secretion by cells

378
Q

Which of the following enzymes is considered
most tissue specific?

A. Creatine kinase (CK)
B. Amylase
C. Alkaline phosphatase (ALP)
D. Alcohol dehydrogenase (ADH)

A

D. Alcohol dehydrogenase (ADH)

379
Q

Which of the following enzymes is activated by
calcium ions?

A. CK
B. Amylase
C. ALP
D. LD

A

B. Amylase

380
Q

Which of the following enzymes is a transferase?

A. ALP
B. CK
C. Amylase
D. LD

A

B. CK

381
Q

Which statement about methods for measuring
LD is true?

A. The formation of pyruvate from lactate (forward
reaction) generates NAD+
B. The pyruvate-to-lactate reaction proceeds at
about twice the rate as the forward reaction
C. The lactate-to-pyruvate reaction is optimized at
pH 7.4
D. The negative-rate reaction is preferred

A

B. The pyruvate-to-lactate reaction proceeds at
about twice the rate as the forward reaction

382
Q

Which condition produces the highest elevation
of serum lactate dehydrogenase?

A. Pernicious anemia
B. Myocardial infarction
C. Acute hepatitis
D. Muscular dystrophy

A

A. Pernicious anemia

383
Q

In which condition is the LD most likely to be
within normal limits?

A. Hepatic carcinoma
B. Pulmonary infarction
C. Acute appendicitis
D. Crush injury

A

C. Acute appendicitis

384
Q

The LD pleural fluid:serum ratio for a transudative
fluid is usually:

A. 3:1 or higher
B. 2:1
C. 1:1
D. 1:2 or less

A

D. 1:2 or less

385
Q

In which type of liver disease would you expect the
greatest elevation of LD?

A. Toxic hepatitis
B. Alcoholic hepatitis
C. Cirrhosis
D. Acute viral hepatitis

A

A. Toxic hepatitis

386
Q

Which of the following conditions will interfere
with the measurement of LD?

A. Slight hemolysis during sample collection
B. Storage at 4°C for 3 days
C. Storage at room temperature for 16 hours
D. Use of plasma collected in heparin

A

A. Slight hemolysis during sample collection

387
Q

In the Oliver–Rosalki method, the reverse reaction
is used to measure CK activity. The enzyme(s)
used in the coupling reactions is (are):

A. Hexokinase and G-6-PD
B. Pyruvate kinase and LD
C. Luciferase
D. Adenylate kinase

A

A. Hexokinase and G-6-PD

388
Q

In the Oliver–Rosalki method for CK, adenosine
monophosphate (AMP) is added to the substrate
in order to:

A. Inhibit adenylate kinase
B. Block the oxidation of glutathione
C. Increase the amount of ADP that is available
D. Block the action of diadenosine pentaphosphate

A

A. Inhibit adenylate kinase

389
Q

Which substance is used in the CK assay to
activate the enzyme?

A. Flavin adenine dinucleotide (FAD)
B. Imidazole
C. N-acetylcysteine
D. Pyridoxyl-5´-phosphate

A

C. N-acetylcysteine

390
Q

SITUATION: A specimen for CK performed
on an automated analyzer using an optimized
Oliver–Rosalki method gives an error flag
indicating substrate depletion. The sample is
diluted 1:2 and 1:4 by the serial dilution technique
and reassayed. After correcting for the dilution,
the results are as follows:
1:2 Dilution = 3,000 IU/L
1:4 Dilution = 3,600 IU/L

Dilutions are made a second time and assayed
again but give identical results. What is the most
likely explanation?

A. The serum became contaminated prior to
making the 1:4 dilution
B. The wrong pipet was used to make one of the
dilutions
C. An endogenous competitive inhibitor is present
in the serum
D. An error has been made in calculating the
enzyme activity of one of the two dilutions

A

C. An endogenous competitive inhibitor is present
in the serum

391
Q

SITUATION: A physician calls to request a CK
on a sample already sent to the laboratory for
coagulation studies. The sample is 2-hour-old
citrated blood and has been stored at 4°C. The
plasma shows very slight hemolysis. What is the
best course of action and the reason for it?

A. Perform the CK assay on the sample because no
interferent is present
B. Reject the sample because it is slightly hemolyzed
C. Reject the sample because it has been stored
too long
D. Reject the sample because the citrate will
interfere

A

D. Reject the sample because the citrate will
interfere

392
Q

Which of the following statements regarding total
CK is true?

A. Levels are unaffected by strenuous exercise
B. Levels are unaffected by repeated intramuscular
injections
C. Highest levels are seen in Duchenne’s muscular
dystrophy
D. The enzyme is highly specific for heart injury

A

C. Highest levels are seen in Duchenne’s muscular
dystrophy

393
Q

Which of the following statements regarding the
clinical use of CK-MB (CK-2) is true?

A. CK-MB becomes elevated before myoglobin
after an AMI
B. CK-MB levels are usually increased in cases of
cardiac ischemia
C. CK-MB is more specific than myoglobin
D. An elevated CK-MB is always accompanied by
an elevated total CK

A

C. CK-MB is more specific than myoglobin

394
Q

A patient’s CK-MB is reported as 18 μg/L and the
total CK as 560 IU/L. What is the CK relative
index (CKI)?

A. 0.10%
B. 3.2%
C. 10.0%
D. 30.0%

A

B. 3.2%

395
Q

In a nonmyocardial as opposed to a myocardial
cause of an increased serum or plasma CK-MB,
which would be expected?

A. An increase in CK-MB that is persistent
B. An increase in the percent CK-MB as well as
concentration
C. The presence of increased TnI
D. A more modest increase in total CK than
CK-MB

A

A. An increase in CK-MB that is persistent

396
Q

Which statement best describes the clinical utility
of plasma or serum myoglobin?

A. Levels greater than 100 μg/L are diagnostic
of AMI
B. Levels below 100 μg/L on admission and
2–4 hours postadmission help to exclude a
diagnosis of AMI
C. Myoglobin peaks after the cardiac troponins
but is more sensitive
D. The persistence of myoglobin > 110 μg/L for
3 days following chest pain favors a diagnosis
of AMI

A

B. Levels below 100 μg/L on admission and
2–4 hours postadmission help to exclude a
diagnosis of AMI

397
Q

What is the typical time course for plasma
myoglobin following an AMI?

A. Abnormal before 1 hour; peaks within 3 hours;
returns to normal in 8 hours
B. Abnormal within 3 hours; peaks within 6 hours;
returns to normal in 18 hours
C. Abnormal within 2 hours; peaks within
12 hours; returns to normal in 36 hours
D. Abnormal within 6 hours; peaks within
24 hours; returns to normal in 72 hours

A

C. Abnormal within 2 hours; peaks within
12 hours; returns to normal in 36 hours

398
Q

What is the typical time course for plasma TnI or
TnT following an AMI?

A. Abnormal within 3 hours; peaks within
12 hours; returns to normal in 24 hours
B. Abnormal within 4 hours; peaks within
18 hours; returns to normal in 48 hours
C. Abnormal within 4 hours; peaks within
24 hours; returns to normal in 1 week
D. Abnormal within 6 hours; peaks within
36 hours; returns to normal in 5 days

A

C. Abnormal within 4 hours; peaks within
24 hours; returns to normal in 1 week

399
Q

Which of the following is the most effective serial
sampling time for ruling out AMI using both
myoglobin and a cardiac specific marker in an
emergency department environment?

A. Admission and every hour for the next 3 hours or
until positive
B. Admission, 2 hours, 4 hours, and 6 hours or
until positive
C. Admission, 3 hours, 6 hours, and a final sample
within 12 hours
D. Admission and one sample every 8 hours for
48 hoursv

A

C. Admission, 3 hours, 6 hours, and a final sample
within 12 hours

400
Q

What is the recommended troponin T and I cutoff
(upper limit of normal) for detecting myocardial
infarction?

A. The cutoff varies with the method of assay but
should be no lower than 0.2 ng/mL
B. The upper 99th percentile or lowest level that
can be measured with 10% CV
C. The concentration corresponding to the lowest
level of calibrator used
D. The highest value fitting under the area of the
curve for the 95% confidence interval

A

B. The upper 99th percentile or lowest level that
can be measured with 10% CV

401
Q

Which of the following cardiac markers is
consistently increased in persons who exhibit
unstable angina?

A. Troponin C
B. Troponin T
C. CK-MB
D. Myoglobin

A

B. Troponin T

402
Q

A patient has a plasma myoglobin of 10 μg/L at
admission. Three hours later, the myoglobin is
14 μg/L and the troponin I is 0.02 μg/L (reference
range 0–0.03 μg/L). These results are consistent
with which condition?

A. Skeletal muscle injury
B. Acute myocardial infarction
C. Unstable angina
D. No evidence of myocardial or skeletal muscle
injury

A

D. No evidence of myocardial or skeletal muscle
injury

403
Q

A patient has a plasma CK-MB of 14 μg/L at
admission and a total CK of 170 IU/L. Serum
myoglobin is 130 μg/L and TnI is 1.6 μg/L.
Three hours later, the TnI is 3.0 μg/L. Which
statement best describes this situation?

A. This patient has had an AMI and further testing
is unnecessary
B. A second CK-MB and myoglobin test should
have been performed at 3 hours postadmission to
confirm AMI
C. These results are consistent with skeletal muscle
damage associated with a crush injury that
elevated the CK-MB
D. Further testing 6–12 hours postadmission is
required to establish a diagnosis of AMI

A

A. This patient has had an AMI and further testing
is unnecessary

404
Q

SITUATION: An EDTA sample for TnI assay
gives a result of 0.04 ng/mL (reference range
0–0.03 ng/mL). The test is repeated 3 hours
later on a new specimen and the result is
0.06 ng/mL. A third sample collected 6 hours
later gives a result of 0.07 ng/mL. The EKG
showed no evidence of ST segment elevation
(STEMI). What is the most likely explanation?

A. A false-positive result occurred due to matrix
interference
B. Heparin should have been used instead of
EDTA, which causes false positives
C. The patient has suffered cardiac injury
D. The patient has had an ischemic episode without
cardiac injury

A

C. The patient has suffered cardiac injury

405
Q

Which of the following laboratory tests is a marker
for ischemic heart disease?

A. CK-MB isoforms
B. Myosin light chain 1
C. Albumin cobalt binding
D. Free fatty acid binding protein

A

C. Albumin cobalt binding

406
Q

Which test becomes abnormal in the earliest stage
of the acute coronary syndrome?

A. Myosin light chain 1
B. CK-MB isoforms
C. Myoglobin
D. High-sensitivity C-reactive protein

A

D. High-sensitivity C-reactive protein

407
Q

Which statement best describes the clinical utility
of B-type natriuretic peptide (BNP)?

A. Abnormal levels may be caused by obstructive
lung disease
B. A positive test indicates prior myocardial
damage caused by AMI that occurred within
the last 3 months
C. A normal test result (<100 pg/mL) helps rule
out congestive heart failure in persons with
symptoms associated with coronary insufficiency
D. A level above 100 pg/mL is not significant if
evidence of congestive heart failure is absent

A

C. A normal test result (<100 pg/mL) helps rule
out congestive heart failure in persons with
symptoms associated with coronary insufficiency

408
Q

Which statement best describes the clinical utility
of plasma homocysteine?

A. Levels are directly related to the quantity of LDL
cholesterol in plasma
B. High plasma levels are associated with
atherosclerosis and increased risk of thrombosis
C. Persons who have an elevated plasma
homocysteine will also have an increased
plasma Lp(a)
D. Plasma levels are increased only when there is an
inborn error of amino acid metabolism

A

B. High plasma levels are associated with
atherosclerosis and increased risk of thrombosis

409
Q

Which of the following cardiac markers derived
from neutrophils predicts an increased risk for
myocardial infarction?

A. Phospholipase A2 (PLA2)
B. Glycogen phosphorylase BB (GPBB)
C. Soluble CD40 ligand (sCD40l)
D. Myeloperoxidase (MPO)

A

D. Myeloperoxidase (MPO)

410
Q

Which of the following statements about the
aminotransferases (AST and ALT) is true?

A. Isoenzymes of AST and ALT are not found in
humans
B. Both transfer an amino group to α–ketoglutarate
C. Both require NADP+ as a coenzyme
D. Both utilize four carbon amino acids as substrates

A

B. Both transfer an amino group to α–ketoglutarate

411
Q

Select the products formed from the forward
reaction of AST.

A. Alanine and α–ketoglutarate
B. Oxaloacetate and glutamate
C. Aspartate and glutamine
D. Glutamate and NADH

A

B. Oxaloacetate and glutamate

412
Q

Select the products formed from the forward
reaction of ALT.

A. Aspartate and alanine
B. Alanine and α–ketoglutarate
C. Pyruvate and glutamate
D. Glutamine and NAD+

A

C. Pyruvate and glutamate

413
Q

Which of the statements below regarding the
methods of Henry for AST and ALT is correct?

A. Hemolysis will cause positive interference in
both AST and ALT assays
B. Loss of activity occurs if samples are frozen
at –20°C
C. The absorbance at the start of the reaction should
not exceed 1.0 A
D. Reaction rates are unaffected by addition of
P-5´-P to the substrate

A

A. Hemolysis will cause positive interference in
both AST and ALT assays

414
Q

Select the coupling enzyme used in the kinetic
AST reaction of Henry.

A. LD
B. Malate dehydrogenase
C. Glutamate dehydrogenase
D. G-6-PD

A

B. Malate dehydrogenase

415
Q

What is the purpose of LD in the kinetic method
of Henry for AST?

A. Forms NADH, enabling the reaction to be
monitored at 340 nm
B. Rapidly exhausts endogenous pyruvate in the
lag phase
C. Reduces oxaloacetate, preventing product
inhibition
D. Generates lactate, which activates AST

A

B. Rapidly exhausts endogenous pyruvate in the
lag phase

416
Q

Which of the following statements regarding the
naming of transaminases is true?

A. Serum glutamic oxaloacetic transaminase
(SGOT) is the older abbreviation for ALT
B. Serum glutamic pyruvic transaminase (SGPT) is
the older abbreviation for AST
C. SGPT is the older abbreviation for ALT
D. SGOT is the newer abbreviation for AST

A

C. SGPT is the older abbreviation for ALT

417
Q

Which statement accurately describes serum
transaminase levels in AMI?

A. ALT is increased 5- to 10-fold after an AMI
B. AST peaks 24–48 hours after an AMI and
returns to normal within 4–6 days
C. AST levels are usually 20–50 times the upper
limit of normal after an AMI
D. Isoenzymes of AST are of greater diagnostic
utility than the total enzyme level

A

B. AST peaks 24–48 hours after an AMI and
returns to normal within 4–6 days

418
Q

Which condition gives rise to the highest serum
level of transaminases?

A. Acute hepatitis
B. Alcoholic cirrhosis
C. Obstructive biliary disease
D. Diffuse intrahepatic cholestasis

A

A. Acute hepatitis

419
Q

In which liver disease is the DeRitis ratio
(ALT:AST) usually greater than 1.0?

A. Acute hepatitis
B. Chronic hepatitis
C. Hepatic cirrhosis
D. Hepatic carcinoma

A

A. Acute hepatitis

420
Q

Which of the following liver diseases produces the
highest levels of transaminases?

A. Hepatic cirrhosis
B. Obstructive jaundice
C. Hepatic cancer
D. Alcoholic hepatitis

A

C. Hepatic cancer

421
Q

Which of the following statements regarding
transaminases is true?

A. ALT is often increased in muscular disease,
pancreatitis, and lymphoma
B. ALT is increased in infectious mononucleosis,
but AST is usually normal
C. ALT is far more specific for liver diseases than
is AST
D. Substrate depletion seldom occurs in assays of
serum from hepatitis cases

A

C. ALT is far more specific for liver diseases than
is AST

422
Q

Select the most sensitive marker for alcoholic liver
disease.

A. GLD
B. ALT
C. AST
D. γ-Glutamyltransferase (GGT)

A

D. γ-Glutamyltransferase (GGT)

423
Q

Which enzyme is least useful in differentiating
necrotic from obstructive jaundice?

A. GGT
B. ALT
C. 5’ Nucleotidase
D. LD

A

D. LD

424
Q

Which of the following statements about the
phosphatases is true?

A. They hydrolyze adenosine triphosphate and
related compounds
B. They are divided into two classes based upon pH
needed for activity
C. They exhibit a high specificity for substrate
D. They are activated by Pi

A

B. They are divided into two classes based upon pH
needed for activity

425
Q

Which of the following statements regarding ALP
is true?

A. In normal adults, the primary tissue source is
fast-twitch skeletal muscle
B. Geriatric patients have a lower serum ALP than
other adults
C. Serum ALP levels are lower in children than in
adults
D. Pregnant women have a higher level of serum
ALP than other adults

A

D. Pregnant women have a higher level of serum
ALP than other adults

426
Q

Which isoenzyme of ALP is most heat stable?

A. Bone
B. Liver
C. Intestinal
D. Placental

A

D. Placental

427
Q

Which isoenzyme of ALP migrates farthest toward
the anode when electrophoresed at pH 8.6?

A. Placental
B. Bone
C. Liver
D. Intestinal

A

C. Liver

428
Q

Which statement regarding bone-specific ALP
is true?

A. The bone isoenzyme can be measured
immunochemically
B. Bone ALP is increased in bone resorption
C. Bone ALP is used for the diagnosis of
osteoporosis
D. There are two distinct bone isoenzymes

A

A. The bone isoenzyme can be measured
immunochemically

429
Q

Which of the following statements regarding ALP
is true?

A. All isoenzymes of ALP are antigenically distinct
and can be identified by specific antibodies
B. Highest serum levels are seen in intrahepatic
obstruction
C. Elevated serum ALP seen with elevated GGT
suggests a hepatic source
D. When jaundice is present, an elevated ALP
suggests acute hepatitis

A

C. Elevated serum ALP seen with elevated GGT
suggests a hepatic source

430
Q

In which condition would an elevated serum
alkaline phosphatase be likely to occur?

A. Small cell lung carcinoma
B. Hemolytic anemia
C. Prostate cancer
D. Acute myocardial infarction

A

A. Small cell lung carcinoma

431
Q

Which condition is least likely to be associated
with increased serum ALP?

A. Osteomalacia
B. Biliary obstruction
C. Hyperparathyroidism and hyperthyroidism
D. Osteoporosis

A

D. Osteoporosis

432
Q

Which substrate is used in the Bowers–McComb
method for ALP?

A. p-Nitrophenyl phosphate
B. β-Glycerophosphate
C. Phenylphosphate
D. α-Naphthylphosphate

A

A. p-Nitrophenyl phosphate

433
Q

Which of the following buffers is used in the IFCC
recommended method for ALP?

A. Glycine
B. Phosphate
C. 2-Amino-2-methyl-1-propanol
D. Citrate

A

C. 2-Amino-2-methyl-1-propanol

434
Q

A serum ALP level greater than twice the elevation
of GGT suggests:

A. Misidentification of the specimen
B. Focal intrahepatic obstruction
C. Acute alcoholic hepatitis
D. Bone disease or malignancy

A

D. Bone disease or malignancy

435
Q

In which condition is the measurement of acid
phosphatase clinically useful?

A. Measuring the prostatic isoenzyme to screen for
prostate cancer
B. Measuring the enzyme in a vaginal swab extract
C. The diagnosis of hemolytic anemia
D. As a marker for bone regeneration

A

B. Measuring the enzyme in a vaginal swab extract

436
Q

Which definition best describes the catalytic
activity of amylase?

A. Hydrolyzes second α 1–4 glycosidic linkages of
starch, glycogen, and other polyglucans
B. Hydrolyzes all polyglucans completely to
produce glucose
C. Oxidatively degrades polysaccharides containing
glucose
D. Splits polysaccharides and disaccharides by
addition of water

A

A. Hydrolyzes second α 1–4 glycosidic linkages of
starch, glycogen, and other polyglucans

437
Q

Which of the following amylase substrates is
recommended by the IFCC?

A. Starch
B. Maltodextrose
C. Maltotetrose
D. Blocked maltohepatoside

A

D. Blocked maltohepatoside

438
Q

How soon following acute abdominal pain due to
pancreatitis is the serum amylase expected to rise?

A. 1–2 hours
B. 2–12 hours
C. 3–4 days
D. 5–6 days

A

B. 2–12 hours

439
Q

Which of the following statements regarding the
diagnosis of pancreatitis is correct?

A. Amylase and lipase are as predictive in chronic as
in acute pancreatitis
B. Diagnostic sensitivity is increased by assaying
both amylase and lipase
C. Measuring the urinary amylase:creatinine ratio is
useful only when patients have renal failure
D. Serum lipase peaks several hours before amylase
after an episode of acute pancreatitis

A

B. Diagnostic sensitivity is increased by assaying
both amylase and lipase

440
Q

Which of the following conditions is associated
with a high level of S-type amylase?

A. Mumps
B. Intestinal obstruction
C. Alcoholic liver disease
D. Peptic ulcers

A

A. Mumps

441
Q

Which of the following statements regarding
amylase methods is true?

A. Requires sulfhydryl compounds for full activity
B. Activity will vary depending on the method used
C. Amyloclastic methods measure the production of
glucose
D. Overrange samples are diluted in deionized water

A

B. Activity will vary depending on the method used

442
Q

Which of the following statements regarding
amylase methods is true?

A. Dilution of serum may result in lower than
expected activity
B. Methods generating NADH are preferred
because they have higher sensitivity
C. Synthetic substrates can be conjugated to
p-nitrophenol (PNP) for a kinetic assay
D. The reference range is consistent from method
to method

A

C. Synthetic substrates can be conjugated to
p-nitrophenol (PNP) for a kinetic assay

443
Q

The reference method for lipase uses olive oil as
the substrate because:

A. Other esterases can hydrolyze triglyceride and
synthetic diglycerides
B. The reaction product can be coupled to NADH
generating reactions
C. Synthetic substrates are less soluble than olive oil
in aqueous reagents
D. Triglyceride substrates cause product inhibition

A

A. Other esterases can hydrolyze triglyceride and
synthetic diglycerides

444
Q

Which statement about the clinical utility of
plasma or serum lipase is true?

A. Lipase is not increased in mumps, malignancy,
or ectopic pregnancy
B. Lipase is not increased as dramatically as amylase
in acute pancreatitis
C. Increased plasma or serum lipase is specific for
pancreatitis
D. Lipase levels are elevated in both acute and
chronic pancreatitis

A

A. Lipase is not increased in mumps, malignancy,
or ectopic pregnancy

445
Q

The reference method for serum lipase is based
upon:

A. Assay of triglycerides following incubation of
serum with olive oil
B. Rate turbidimetry
C. Titration of fatty acids with dilute NaOH
following controlled incubation of serum with
olive oil
D. Immunochemical assay

A

C. Titration of fatty acids with dilute NaOH
following controlled incubation of serum with
olive oil

446
Q

The most commonly employed method of assay
for plasma or serum lipase is based on:

A. Hydrolysis of olive oil
B. Rate turbidimetry
C. Immunoassay
D. Peroxidase coupling

A

D. Peroxidase coupling

447
Q

Which of the following enzymes is usually
depressed in liver disease?

A. Elastase-1
B. GLD
C. Pseudocholinesterase
D. Aldolase

A

C. Pseudocholinesterase

448
Q

Which enzyme is most likely to be elevated in the
plasma of a person suffering from a muscle wasting
disorder?

A. 5´-Nucleotidase
B. Pseudocholinesterase
C. Aldolase
D. Glutamate dehydrogenase

A

C. Aldolase

449
Q

Which enzyme is measured in whole blood?

A. Chymotrypsin
B. Glucose-6-phosphate dehydrogenase
C. Glycogen phosphorylase
D. Lipase

A

B. Glucose-6-phosphate dehydrogenase

450
Q

Which of the following hormones is often
decreased by approximately 25% in the serum
of pregnant women who have a fetus with Down
syndrome?

A. Estriol (E3)
B. Human chorionic gonadotropin (hCG)
C. Progesterone
D. Estradiol (E2)

A

A. Estriol (E3)

451
Q

The syndrome of inappropriate antidiuretic
hormone secretion (SIADH) causes:

A. Low serum vasopressin
B. Hypernatremia
C. Urine osmolality to be lower than plasma
D. Low serum electrolytes

A

D. Low serum electrolytes

452
Q

Select the hormone which when elevated is
associated with galactorrhea, pituitary adenoma,
and amenorrhea.

A. E2
B. Progesterone
C. Follicle-stimulating hormone (FSH)
D. Prolactin

A

D. Prolactin

453
Q

Zollinger–Ellison (Z–E) syndrome is characterized
by great (e.g., 20-fold) elevation of:

A. Gastrin
B. Cholecystokinin
C. Pepsin
D. Glucagon

A

A. Gastrin

454
Q

Which statement about multiple endocrine
neoplasia (MEN) is true?

A. It is associated with hyperplasia or neoplasia of at
least two endocrine organs
B. Insulinoma is always present when the pituitary
is involved
C. It is inherited as an autosomal recessive disorder
D. Plasma hormone levels from affected organs are
elevated at least 10-fold

A

A. It is associated with hyperplasia or neoplasia of at
least two endocrine organs

455
Q

Select the main estrogen produced by the ovaries
and used to evaluate ovarian function.

A. Estriol (E3 )
B. Estradiol (E2 )
C. Epiestriol
D. Hydroxyestrone

A

B. Estradiol (E2 )

456
Q

Which statement best describes the relationship
between luteinizing hormone (LH) and folliclestimulating hormone (FSH) in cases of
dysmenorrhea?

A. Both are usually increased when there is pituitary
adenoma
B. Increases in both hormones and a decrease in
estrogen signal a pituitary cause of ovarian failure
C. Both hormones normally peak 1–2 days before
ovulation
D. In menopause, the LH level at the midcycle peak
is higher than the level of FSH

A

C. Both hormones normally peak 1–2 days before
ovulation

457
Q

When pituitary adenoma is the cause of decreased
estrogen production, an increase of which
hormone is most frequently responsible?

A. Prolactin
B. FSH
C. LH
D. Thyroid-stimulating hormone (TSH)

A

A. Prolactin

458
Q

Which set of results is most likely in an adult male
with primary testicular failure?

A. Increased LH, FSH, and decreased testosterone
B. Decreased LH, FSH, and testosterone
C. Decreased testosterone, androstenedione,
and FSH
D. Increased androstenedione, decreased
testosterone, and normal FSH

A

A. Increased LH, FSH, and decreased testosterone

459
Q

When should progesterone be measured when
evaluating an adult female for anovulation?

A. At the onset of menses
B. During the first 7 days of the menstrual cycle
C. At the midcycle just after LH peaks
D. At the end of the menstrual cycle

A

C. At the midcycle just after LH peaks

460
Q

A female with severe excessive pubic and facial hair
growth (hirsutism) should be tested for which of
the following hormones?

A. Estrogen and progesterone
B. Chorionic gonadotropin
C. Growth hormone
D. Testosterone and dehydroepiandrosterone sulfate

A

D. Testosterone and dehydroepiandrosterone sulfate

461
Q

Which set of results is most likely in a female with
hypogonadotropic ovarian failure?

A. Increased LH, FSH, and estrogen
B. Decreased LH, FSH, and estrogen
C. Decreased prolactin and estrogen
D. Increased LH and FSH, and decreased estrogen

A

B. Decreased LH, FSH, and estrogen

462
Q

The onset of menopause is usually associated with
what hormone changes?

A. Decreased estrogen, testosterone, and androgens
B. Decreased estrogen, FSH, LH, and progesterone
C. Decreased estrogen and progesterone, and
increased LH and FSH
D. Decreased estrogen and progesterone, normal
LH and FSH

A

C. Decreased estrogen and progesterone, and
increased LH and FSH

463
Q

Which of the following statements is correct in
assessing GH deficiency?

A. Pituitary failure may involve one, several, or all
adenohypophyseal hormones; but GH deficiency
is usually found
B. A normal random serum level of GH in a child
under 6 years old rules out GH deficiency
C. Administration of arginine, insulin, or glucagon
will suppress GH release
D. GH levels in the blood show little variation
within a 24-hour period

A

A. Pituitary failure may involve one, several, or all
adenohypophyseal hormones; but GH deficiency
is usually found

464
Q

Which statement best describes the level of GH in
patients with pituitary adenoma associated with
acromegaly?

A. The fasting GH level is always elevated at least
twofold
B. Some patients will require a glucose suppression
test to establish a diagnosis
C. A normal fasting GH level rules out acromegaly
D. Patients produce a lower concentration of
insulin-like growth factor I (IGF-1) than
expected from their GH level

A

B. Some patients will require a glucose suppression
test to establish a diagnosis

465
Q

Hyperparathyroidism is most consistently
associated with:

A. Hypocalcemia
B. Hypocalciuria
C. Hypophosphatemia
D. Metabolic alkalosis

A

C. Hypophosphatemia

466
Q

Which statement regarding the use of PTH
is true?

A. Determination of serum PTH level is the best
screening test for disorders of calcium
metabolism
B. PTH levels differentiate primary and secondary
causes of hypoparathyroidism
C. PTH levels differentiate primary and secondary
causes of hypocalcemia
D. PTH levels are low in patients with
pseudohypoparathyroidism

A

C. PTH levels differentiate primary and secondary
causes of hypocalcemia

467
Q

The best method of analysis for serum PTH
involves using antibodies that detect:

A. The amino-terminal fragment of PTH
B. The carboxy-terminal end of PTH
C. Both the amino-terminal fragment and
intact PTH
D. All fragments of PTH as well as intact hormone

A

C. Both the amino-terminal fragment and
intact PTH

468
Q

Which of the following is most often elevated in
hypercalcemia associated with malignancy?

A. Parathyroid-derived PTH
B. Ectopic PTH
C. Parathyroid hormone–related protein (PTHRP)
D. Calcitonin

A

C. Parathyroid hormone–related protein (PTHRP)

469
Q

Which is normally the most abundant
corticosteroid hormone secreted by the adrenal
cortex?

A. Cortisol
B. Dehydroepiandrosterone
C. Aldosterone
D. Corticosterone

A

A. Cortisol

470
Q

Which of the following statements regarding
adrenal cortical dysfunction is true?

A. Patients with Cushing’s syndrome usually have
hyperkalemia
B. Cushing’s syndrome is associated with glucose
intolerance
C. Addison’s disease is associated with
hypernatremia
D. Addison’s disease is caused by elevated levels of
cortisol

A

B. Cushing’s syndrome is associated with glucose
intolerance

471
Q

Which of the following statements about cortisol
in Cushing’s syndrome is true?

A. Twenty-four–hour urinary free cortisol is a more
sensitive test than plasma total cortisol
B. Patients with Cushing’s disease show
pronounced diurnal variation in serum cortisol
C. Free cortisol is increased by a high-serum
cortisol-binding protein concentration
D. An elevated serum total cortisol level is
diagnostic of Cushing’s syndrome

A

A. Twenty-four–hour urinary free cortisol is a more
sensitive test than plasma total cortisol

472
Q

Which of the following conditions is characterized
by primary hyperaldosteronism caused by adrenal
adenoma, carcinoma, or hyperplasia?

A. Cushing’s syndrome
B. Addison’s disease
C. Conn’s syndrome
D. Pheochromocytoma

A

C. Conn’s syndrome

473
Q

Which of the following is the most common cause
of Cushing’s syndrome?

A. Pituitary adenoma
B. Adrenal hyperplasia
C. Overuse of corticosteroids
D. Ectopic adrenocorticotropic hormone (ACTH)
production by tumors

A

C. Overuse of corticosteroids

474
Q

Which of the following is the mechanism causing
Cushing’s disease?

A. Excess secretion of pituitary ACTH
B. Adrenal adenoma
C. Treatment with corticosteroids
D. Ectopic ACTH production by tumors

A

A. Excess secretion of pituitary ACTH

475
Q

In which situation is the plasma or 24-hour
urinary cortisol not consistent with the clinical
picture?

A. In pregnant patients
B. In patients with a positive overnight
dexamethasone suppression test
C. In congenital adrenal hyperplasia
D. In Cushing’s syndrome caused by ectopic ACTH
producing tumors

A

C. In congenital adrenal hyperplasia

476
Q

Which test is used to distinguish Cushing’s disease
(pituitary Cushing’s) from Cushing’s syndrome
caused by adrenal tumors?

A. Low-dose overnight dexamethasone suppression
B. Petrosal sinus sampling
C. Serum ACTH
D. Twenty-four–hour urinary free cortisol

A

C. Serum ACTH

477
Q

Which is the most widely used screening test for
Cushing’s syndrome?

A. Overnight low-dose dexamethasone
suppression test
B. Corticotropin-releasing hormone stimulation
test
C. Petrosal sinus sampling
D. Metyrapone stimulation test

A

A. Overnight low-dose dexamethasone
suppression test

478
Q

Which test is the most specific for establishing a
diagnosis of Cushing’s disease (pituitary
Cushing’s)?

A. Low-dose dexamethasone suppression
B. High-dose dexamethasone suppression
C. Twenty-four–hour urinary free cortisol
D. Petrosal sinus sampling following
corticotropin-releasing hormone stimulation

A

D. Petrosal sinus sampling following
corticotropin-releasing hormone stimulation

479
Q

Which of the following statements about the
diagnosis of Addison’s disease is true?

A. Patients with primary Addison’s disease show a
normal response to ACTH stimulation
B. Primary and secondary Addison’s disease can
often be differentiated by plasma ACTH
C. Twenty-four–hour urinary free cortisol is normal
in Addison’s disease
D. Pituitary ACTH reserves are normal in secondary
Addison’s disease

A

B. Primary and secondary Addison’s disease can
often be differentiated by plasma ACTH

480
Q

Which of the following statements regarding the
catecholamines is true?

A. They are derived from tryptophan
B. They are produced by the zona glomerulosa of
the adrenal cortex
C. Plasma levels show both diurnal and pulsed
variation
D. They are excreted in urine primarily as free
catecholamines

A

C. Plasma levels show both diurnal and pulsed
variation

481
Q

Which assay using 24-hour urine is considered the
best single screening test for pheochromocytoma?

A. Total urinary catecholamines
B. VMA
C. Homovanillic acid (HVA)
D. Metanephrines

A

D. Metanephrines

482
Q

Which metabolite is most often increased in
carcinoid tumors of the intestine?

A. 5-Hydroxyindolacetic acid (5-HIAA)
B. 3-Methoxy-4-hydroxyphenylglycol (MHPG)
C. 3-Methoxydopamine
D. HVA

A

A. 5-Hydroxyindolacetic acid (5-HIAA)

483
Q

Which statement regarding the measurement of
urinary catecholamines is true?

A. An increased excretion of total urinary
catecholamines is specific for pheochromocytoma
B. Twenty-four–hour urinary catecholamine assay
avoids pulse variations associated with
measurement of plasma catecholamines
C. Total urinary catecholamine measurement
provides greater specificity than measurement of
urinary free catecholamines
D. Total urinary catecholamines are not affected by
exercise

A

B. Twenty-four–hour urinary catecholamine assay
avoids pulse variations associated with
measurement of plasma catecholamines

484
Q

Which method is most often used to measure
fractionated catecholamines (epinephrine,
norepinephrine, and dopamine)?

A. Measurement of fluorescence following oxidation
by potassium ferricyanide
B. Measurement by HPLC with electrochemical
detection
C. Measure of radioactivity after conversion by
catechol-O-methyltransferase (COMT) to
tritiated metanephrines
D. Measurement by HPLC with fluorescence
detection

A

B. Measurement by HPLC with electrochemical
detection

485
Q

Which statement about sample collection for
catecholamines and metabolites is true?

A. Blood for catecholamines is collected in the usual
manner following a 12-hour fast
B. Twenty-four–hour urine for vanillylmandelic
acid, catecholamines, or metanephrines is
collected in 1 mL of boric acid
C. Twenty-four–hour urine creatinine should be
measured with vanillylmandelic acid,
homovanillic acid, or metanephrines
D. There is no need to discontinue medications if a
24-hour urine collection is used

A

C. Twenty-four–hour urine creatinine should be
measured with vanillylmandelic acid,
homovanillic acid, or metanephrines

486
Q

Which of the following statements applies to both
measurement of VMA and metanephrines in
urine?

A. Both can be oxidized to vanillin and measured at
360 nm without interference from dietary
compounds
B. Both can be measured immunochemically after
hydrolysis and derivatization
C. Both require acid hydrolysis prior to
measurement
D. Both can be measured by specific HPLC
and MS assays

A

D. Both can be measured by specific HPLC
and MS assays

487
Q

Urinary HVA is most often assayed to detect:

A. Pheochromocytoma
B. Neuroblastoma
C. Adrenal medullary carcinoma
D. Psychiatric disorders such as manic depression

A

B. Neuroblastoma

488
Q

Thyroid hormones are derived from the
amino acid:

A. Phenylalanine
B. Methionine
C. Tyrosine
D. Histidine

A

C. Tyrosine

489
Q

Which statement regarding thyroid hormones
is true?

A. Circulating levels of T3 and T4 are about equal
B. T3 is about 10-fold more active than T4
C. The rate of formation of monoiodotyrosine and
diiodotyrosine is about equal
D. Most of the T3 present in plasma is from its
direct release from thyroid storage sites

A

B. T3 is about 10-fold more active than T4

490
Q

Which of the following statements regarding
thyroid hormones is true?

A. Both protein-bound and free T3 and T4 are
physiologically active
B. Total T3 and T4 are influenced by the level of
thyroxine-binding globulin
C. Variation in thyroxine-binding protein levels
affects both free T3 and T4
D. An elevated serum total T4 and T3 is diagnostic
of hyperthyroidism

A

B. Total T3 and T4 are influenced by the level of
thyroxine-binding globulin

491
Q

Which of the following conditions will increase
total T4 by increasing TBG?

A. Acute illness
B. Anabolic steroid use
C. Nephrotic syndrome
D. Pregnancy or estrogens

A

D. Pregnancy or estrogens

492
Q

Select the most appropriate single screening test
for thyroid disease.

A. Free thyroxine index
B. Total T3 assay
C. Total T4
D. TSH assay

A

D. TSH assay

493
Q

The serum TSH level is almost absent in:

A. Primary hyperthyroidism
B. Primary hypothyroidism
C. Secondary hyperthyroidism
D. Euthyroid sick syndrome

A

A. Primary hyperthyroidism

494
Q

Which assay is used to confirm difficult cases of
hypothyroidism?

A. Free T3 assay
B. Free thyroxine index
C. Thyrotropin-releasing hormone (TRH)
stimulation test
D. TBG assay

A

C. Thyrotropin-releasing hormone (TRH)
stimulation test

495
Q

Which of the following statements is true
regarding reverse T3 (rT3)?

A. Formed in the blood by degradation of T4
B. Physiologically active, but less than T3
C. Decreased in euthyroid sick syndrome
D. Interferes with the measurement of serum T3

A

A. Formed in the blood by degradation of T4

496
Q

A patient has an elevated serum T3 and free T4 and
undetectable TSH. What is the most likely cause
of these results?

A. Primary hyperthyroidism
B. Secondary hyperthyroidism
C. Euthyroid with increased thyroxine-binding
proteins
D. Euthyroid sick syndrome

A

A. Primary hyperthyroidism

497
Q

A serum thyroid panel reveals an increase in total
T4, normal TSH, and normal free T4. What is the
most likely cause of these results?

A. Primary hyperthyroidism
B. Secondary hyperthyroidism
C. Euthyroid with increased thyroxine-binding
protein
D. Subclinical hypothyroidism

A

C. Euthyroid with increased thyroxine-binding
protein

498
Q

Which statement about TSH and T4 in early
pregnancy is correct?

A. TSH and thyroid hormones fall
B. TSH falls and thyroid hormones rise
C. TSH and thyroid hormones both rise
D. TSH rises and thyroid hormones fall

A

B. TSH falls and thyroid hormones rise

499
Q

In which case might a very low plasma TSH result
not correlate with thyroid status?

A. Euthyroid sick syndrome
B. Congenital hypothyroidism
C. When TBG is elevated
D. After high-dose corticosteroid treatment

A

D. After high-dose corticosteroid treatment

500
Q

In which of the following cases is qualitative
analysis of the drug usually adequate?

A. To determine whether the dose of a drug with a
low therapeutic index is likely to be toxic
B. To determine whether a patient is complying
with the physician’s instructions
C. To adjust dose if individual differences or disease
alter expected response
D. To determine whether the patient has been
taking amphetamines

A

D. To determine whether the patient has been
taking amphetamines

501
Q

The term pharmacokinetics refers to the:

A. Relationship between drug dose and the drug
blood level
B. Concentration of drug at its sites of action
C. Relationship between blood concentration and
therapeutic response
D. The relationship between blood and tissue drug
levels

A

A. Relationship between drug dose and the drug
blood level

502
Q

The term pharmacodynamics is an expression of
the relationship between:

A. Dose and physiological effect
B. Drug concentration at target sites and
physiological effect
C. Time and serum drug concentration
D. Blood and tissue drug levels

A

B. Drug concentration at target sites and
physiological effect

503
Q

The study of pharmacogenomics involves which
type of testing?

A. Family studies to determine the inheritance of
drug resistance
B. Testing drugs with cell cultures to determine the
minimum toxic dosage
C. Testing for single nucleotide polymorphisms
known to affect drug metabolism
D. Comparison of dose-response curves between
family members

A

C. Testing for single nucleotide polymorphisms
known to affect drug metabolism

504
Q

Select the five pharmacological parameters that
determine serum drug concentration.

A. Absorption, anabolism, perfusion, bioactivation,
excretion
B. Liberation, equilibration, biotransformation,
reabsorption, elimination
C. Liberation, absorption, distribution, metabolism,
excretion
D. Ingestion, conjugation, integration, metabolism,
elimination

A

C. Liberation, absorption, distribution, metabolism,
excretion

505
Q

Which route of administration is associated with
100% bioavailability?

A. Sublingual
B. Intramuscular
C. Oral
D. Intravenous

A

D. Intravenous

506
Q

The phrase “first-pass hepatic metabolism”
means that:

A. One hundred percent of a drug is excreted by
the liver
B. All drug is inactivated by hepatic enzymes after
one pass through the liver
C. Some drug is metabolized from the portal
circulation, reducing bioavailability
D. The drug must be metabolized in the liver to an
active form

A

C. Some drug is metabolized from the portal
circulation, reducing bioavailability

507
Q

Which formula can be used to estimate dosage
needed to give a desired steady-state blood level?

A. Dose per hour = clearance (milligrams per hour)
× average concentration at steady state ÷ f
B. Dose per day = fraction absorbed – fraction
excreted
C. Dose = fraction absorbed × (1/protein-bound
fraction)
D. Dose per day = half-life × log Vd (volume
distribution)

A

A. Dose per hour = clearance (milligrams per hour)
× average concentration at steady state ÷ f

508
Q

Which statement is true regarding the volume
distribution (Vd) of a drug?

A. Vd is equal to the peak blood concentration
divided by the dose given
B. Vd is the theoretical volume in liters into which
the drug distributes
C. The higher the Vd, the lower the dose needed to
reach the desired blood level of drug
D. The Vd is the principal determinant of the dosing
interval

A

B. Vd is the theoretical volume in liters into which
the drug distributes

509
Q

For drugs with first-order elimination, which
statement about drug clearance is true?

A. Clearance = elimination rate ÷ serum level
B. It is most often performed by the liver
C. It is directly related to half-life
D. Clearance rate is independent of dose

A

A. Clearance = elimination rate ÷ serum level

510
Q

Which statement about steady-state drug levels
is true?

A. The absorbed drug must be greater than the
amount excreted
B. Steady state can be measured after two
elimination half-lives
C. Constant intravenous infusion will give the same
minima and maxima as an oral dose
D. Oral dosing intervals give peaks and troughs in
the dose-response curve

A

D. Oral dosing intervals give peaks and troughs in
the dose-response curve

511
Q

If too small a peak–trough difference is seen for a
drug given orally, then:

A. The dose should be decreased
B. Time between doses should be decreased
C. Dose interval should be increased
D. Dose per day and time between doses should be
decreased

A

C. Dose interval should be increased

512
Q

If the peak level is appropriate but the trough level
too low at steady state, then the dose interval
should:

A. Be lengthened without changing the dose
per day
B. Be lengthened and dose rate decreased
C. Not be changed, but dose per day increased
D. Be shortened, but dose per day not changed

A

D. Be shortened, but dose per day not changed

513
Q

If the steady-state drug level is too high, the best
course of action is to:

A. Decrease the dose
B. Decrease the dose interval
C. Decrease the dose and decrease the dose interval
D. Change the route of administration

A

A. Decrease the dose

514
Q

When should blood samples for trough drug levels
be collected?

A. 30 minutes after peak levels
B. 45 minutes before the next dose
C. 1–2 hours after the last dose
D. Immediately before the next dose is given

A

D. Immediately before the next dose is given

515
Q

Blood sample collection time for peak drug levels:

A. Varies with the drug, depending on its rate of
absorption
B. Is independent of drug formulation
C. Is independent of the route of administration
D. Is 30 minutes after a bolus intravenous injection
is completed

A

A. Varies with the drug, depending on its rate of
absorption

516
Q

Which could account for drug toxicity following a
normally prescribed dose?

A. Decreased renal clearance caused by kidney disease
B. Discontinuance or administration of another drug
C. Altered serum protein binding caused by disease
D. All of these options

A

D. All of these options

517
Q

Select the elimination model that best describes
most oral drugs.

A. One compartment, linear first-order elimination
B. Michaelis–Menton or concentration-dependent
elimination
C. Two compartment with a biphasic elimination
curve
D. Logarithmic elimination

A

A. One compartment, linear first-order elimination

518
Q

Drugs rapidly infused intravenously usually follow
which elimination model?

A. One compartment, first order
B. One compartment, logarithmic
C. Biphasic or two compartment with serum level
rapidly falling in the first phase
D. Michaelis–Menton or concentration-dependent
elimination

A

C. Biphasic or two compartment with serum level
rapidly falling in the first phase

519
Q

Which fact must be considered when evaluating a
patient who displays signs of drug toxicity?

A. Drug metabolites (e.g., N-acetylprocainamide)
may need to be measured as well as parent drug
B. If the concentration of total drug is within
therapeutic limits, the concentration of free drug
cannot be toxic
C. If the drug has a wide therapeutic index, then it
will not be toxic
D. A drug level cannot be toxic if the trough is
within the published therapeutic range

A

A. Drug metabolites (e.g., N-acetylprocainamide)
may need to be measured as well as parent drug

520
Q

When a therapeutic drug is suspected of causing
toxicity, which specimen is the most appropriate
for an initial investigation?

A. Trough blood sample
B. Peak blood sample
C. Urine at the time of symptoms
D. Gastric fluid at the time of symptoms

A

B. Peak blood sample

521
Q

For a drug that follows first-order pharmacokinetics,
adjustment of dosage to achieve the desired blood
level can be made using which formula?

A. New dose = (current dose/concentration at
steady state) × desired concentration
B. New dose = (current dose/desired concentration) × concentration at
steady state
C. New dose = (concentration at steady state/desired concentration) × half-life
D. New dose = (concentration at steady state/current dose) × desired concentration

A

A. New dose = (current dose/concentration at
steady state) × desired concentration

522
Q

For which drug group are both peak and trough
measurements usually required?

A. Antiarrhythmics
B. Analgesics
C. Tricyclic antidepressants
D. Aminoglycoside antibiotics

A

D. Aminoglycoside antibiotics

523
Q

Which of the following statements about TLC for
drug screening is true?

A. Acidic drugs are extracted in an alkaline
nonpolar solvent
B. A drug is identified by comparing its Rf value
and staining to standards
C. Testing must be performed using a urine sample
D. Opiates and other alkaloids are extracted at an
acid pH

A

B. A drug is identified by comparing its Rf value
and staining to standards

524
Q

The EMIT for drugs of abuse uses an:

A. Antibody conjugated to a drug
B. Enzyme conjugated to an antibody
C. Enzyme conjugated to a drug
D. Antibody bound to a solid phase

A

C. Enzyme conjugated to a drug

525
Q

Which statement about EMIT is true?

A. Enzyme activity is inversely proportional to drug
level
B. Formation of NADH is monitored at 340 nm
C. ALP is the commonly used conjugate
D. Assay use is restricted to serum

A

B. Formation of NADH is monitored at 340 nm

526
Q

Which statement regarding cloned enzyme donor
immunoassay (CEDIA) is true?

A. The enzyme used is glucose-6-phosphate
dehydrogenase
B. The enzyme donor and acceptor molecules are
fragments of β-galactosidase
C. Drug concentration is inversely related to
fluorescence
D. The antibody is covalently linked to the enzyme
donor

A

B. The enzyme donor and acceptor molecules are
fragments of β-galactosidase

527
Q

Which statement is true regarding particle-enhanced
turbidimetric inhibition immunoassay methods for
therapeutic drugs?

A. Drug concentration is proportional to light
scatter
B. Magnetic separation is needed to remove
unbound conjugate
C. When particle-bound drug binds to antibody,
light scattering is increased
D. Two antibodies to the drug are needed

A

C. When particle-bound drug binds to antibody,
light scattering is increased

528
Q

Quantitation of a drug by gas chromatography–mass
spectroscopy (GC-MS) is usually performed in
which mode?

A. Total ion chromatography
B. Selective ion monitoring
C. Ion subtraction
D. Selective reaction monitoring

A

B. Selective ion monitoring

529
Q

SITUATION: A urine sample is received in the
laboratory with the appropriate custody control
form, and a request for drug of abuse screening.
Which test result would be cause for rejecting
the sample?

A. Temperature after collection 95°F
B. pH 5.0
C. Specific gravity 1.005
D. Creatinine 5 mg/dL

A

D. Creatinine 5 mg/dL

530
Q

Which substance has the longest detection time?

A. Amphetamines
B. Cocaine
C. Benzodiazepines
D. Marijuana

A

D. Marijuana

531
Q

Which statement about the measurement of
carboxyhemoglobin is true?

A. Treatment with alkaline dithionite is used to
convert carboxyhemoglobin to oxyhemoglobin
B. Oxyhemoglobin has no absorbance at
540 nm, but carboxyhemoglobin does
C. Bichromatic analysis is required in order to
eliminate interference by oxyhemoglobin
D. Carboxyhemoglobin can be measured by
potentiometry

A

C. Bichromatic analysis is required in order to
eliminate interference by oxyhemoglobin

532
Q

Which of the following statements about blood
alcohol measurement is correct?

A. Symptoms of intoxication usually begin when
the level exceeds 0.05% w/v
B. The skin puncture site should be disinfected
with isopropanol
C. The reference method is based upon enzymatic
oxidation of ethanol by alcohol dehydrogenase
D. Gas chromatography methods require extraction
of ethanol from serum

A

A. Symptoms of intoxication usually begin when
the level exceeds 0.05% w/v

533
Q

Which specimen is the sample of choice for lead
screening?

A. Whole blood
B. Hair
C. Serum
D. Urine

A

A. Whole blood

534
Q

Which of the following enzymes can be used to
measure plasma or serum salicylate?

A. Peroxidase
B. Salicylate esterase
C. Salicylate hydroxylase
D. p-Aminosalicylate oxidase

A

C. Salicylate hydroxylase

535
Q

Which of the following tests is least essential to the
operation of an emergency department at a general
hospital?

A. Carboxyhemoglobin
B. Osmolality
C. Salicylate
D. Lead

A

D. Lead

536
Q

Which of the following trace elements is
considered an essential micronutrient?

A. Thallium
B. Aluminum
C. Mercury
D. Selenium

A

D. Selenium

537
Q

When measuring trace metals in blood other
than lead, what type of tube should be used?

A. Navy blue top
B. Green top
C. Purple top
D. Red top

A

A. Navy blue top

538
Q

Which whole-blood level is suggestive of excessive
exposure to lead in children but not adults?

A. 4 μg/dL
B. 14 μg/dL
C. 28 μg/dL
D. 32 μg/dL

A

B. 14 μg/dL

539
Q

Which whole-blood level is suggestive of excessive
exposure to lead in children but not adults?

A. 4 μg/dL
B. 14 μg/dL
C. 28 μg/dL
D. 32 μg/dL

A

A. 4 μg/dL

540
Q

Which of the following tumor markers is classified
as a tumor suppressor gene?

A. BRCA-1
B. Carcinoembryonic antigen (CEA)
C. Human chorionic gonadotropin (hCG)
D. Nuclear matrix protein

A

A. BRCA-1

541
Q

In general, in which of the following situations is
the analysis of a tumor marker most useful?

A. Testing for recurrence
B. Prognosis
C. Screening
D. Diagnosis

A

A. Testing for recurrence

542
Q

Which of the following enzymes is increased in
persons with prostate and small-cell lung cancer?

A. Creatine kinase-1 (CK-1)
B. Gamma glutamyl transferase (GGT)
C. Amylase
D. Lactate dehydrogenase

A

A. Creatine kinase-1 (CK-1)

543
Q

Which of the following is the best analyte to
monitor for recurrence of ovarian cancer?

A. CA-15-3
B. CA-19-9
C. CA-125
D. CEA

A

C. CA-125

544
Q

Which tumor marker is associated with cancer of
the urinary bladder?

A. CA-19-9
B. CA-72-4
C. Nuclear matrix protein
D. Cathepsin-D

A

C. Nuclear matrix protein

545
Q

A person presents with a cushingoid appearance
and an elevated 24-hour urinary cortisol level. The
plasma adrenocotropic hormone (ACTH) is very
elevated, and the physician suspects the cause is
ectopic ACTH production. Which test would be
most useful in substantiating this diagnosis?

A. Plasma cortisol
B. CA-50
C. Alkaline phosphatase isoenzymes
D. AFP

A

C. Alkaline phosphatase isoenzymes

546
Q

Which of the following tumor markers is used to
monitor persons with breast cancer for recurrence
of disease?

A. Cathepsin-D
B. CA-15-3
C. Retinoblastoma gene
D. Estrogen receptor (ER)

A

B. CA-15-3

547
Q

Which of the following statements regarding the
Philadelphia chromosome is true?

A. It is seen exclusively in chronic myelogenous
leukemia
B. It results from a translocation
C. It appears as a short-arm deletion of
chromosome 21
D. It is associated with a poor prognosis

A

B. It results from a translocation

548
Q

Which of the following statements regarding the
Philadelphia chromosome is true?

A. It is seen exclusively in chronic myelogenous
leukemia
B. It results from a translocation
C. It appears as a short-arm deletion of
chromosome 21
D. It is associated with a poor prognosis

A

D. It is associated with a poor prognosis

549
Q

Which tumor marker is used to determine the
usefulness of trastuzumab (Herceptin) therapy for
breast cancer?

A. PR
B. CEA
C. HER-2/neu
D. Myc

A

C. HER-2/neu

550
Q

A person is suspected of having testicular cancer.
Which type of hCG test would be most useful?

A. Plasma immunoassay for intact hCG only
B. Plasma immunoassay for intact hCG and
the β-hCG subunit
C. Plasma immunoassay for the free alpha
and β-hCG subunits
D. Urine assay for hCG β core

A

B. Plasma immunoassay for intact hCG and
the β-hCG subunit

551
Q

A patient treated for a germ cell tumor has a total
and free β-hCG assay performed prior to surgery.
The result is 40,000 mIU/mL. One week
following surgery, the hCG is 5,000 mIU/mL.
Chemotherapy is started, and the hCG is
measured 1 week later and found to be
10,000 mIU/mL. What does this indicate?

A. Recurrence of the tumor
B. Falsely increased hCG owing to drug
interference with the assay
C. Analytical error with the test reported as
5,000 mIU/mL
D. Transient hCG increase caused by chemotherapy

A

D. Transient hCG increase caused by chemotherapy

552
Q

Which set of results for ER and PR is associated
with the highest likelihood of a favorable response
to treatment with estrogen-suppression therapy
(tamoxifen)?

A. ER positive, PR positive
B. ER positive, PR negative
C. ER negative, PR positive
D. ER negative, PR negative

A

A. ER positive, PR positive

553
Q

Which type of cancer is associated with the highest
level of AFP?

A. Hepatoma
B. Ovarian cancer
C. Testicular cancer
D. Breast cancer

A

A. Hepatoma

554
Q

Which of the following assays is recommended as
a screening test for colorectal cancer in persons
over 50 years old?

A. CEA
B. AFP
C. Occult blood
D. Fecal trypsin

A

C. Occult blood

555
Q

Which of the following assays is used to determine
the risk of developing cancer?

A. Epidermal growth factor receptor (EGF-R)
B. Squamous cell carcinoma antigen (SCC)
C. c-erb B-2 gene expression
D. p53 gene mutation

A

D. p53 gene mutation

556
Q

homovanillic acid (HVA) and vanillymandelic
acid (VMA). Urinary metanephrines,
chromogranin A, and neuron-specific enolase
are also elevated but 5-hydroxyindoleacetic
acid is within the reference range. What is
the most likely diagnosis?

A. Carcinoid tumors of the intestine
B. Pheochromocytoma
C. Neuroblastoma
D. Pancreatic cancer

A

C. Neuroblastoma

557
Q

In which of the following conditions is PSA least
likely to be increased?

A. Precancerous lesions of the prostate
B. Postprostate biopsy
C. Benign prostatic hypertrophy
D. Post–digital rectal examination

A

D. Post–digital rectal examination

558
Q

Which of the following statements regarding PSA
is true?

A. Complexed PSA in plasma is normally less than
free PSA
B. Free PSA below 25% is associated with
malignant disease
C. A total PSA below 4 ng/mL rules out malignant
disease
D. A total PSA above 10 ng/mL is diagnostic of
malignant disease

A

B. Free PSA below 25% is associated with
malignant disease

559
Q

Which of the following statements regarding PSA
is true?

A. Complexed PSA in plasma is normally less than
free PSA
B. Free PSA below 25% is associated with
malignant disease
C. A total PSA below 4 ng/mL rules out malignant
disease
D. A total PSA above 10 ng/mL is diagnostic of
malignant disease

A

A. Complexed PSA in plasma is normally less than
free PSA

560
Q

Which of the following procedures can be used to
detect proportional error in a new method for
glucose?

A. Compare the standard deviation of 40 patient
samples to the hexokinase method
B. Measure a mixture made from equal parts of
normal and high-QC sera
C. Add 5.0 mg of glucose to 1.0 mL of a serum of
known concentration and measure
D. Compare the mean of 40 normal samples to the
hexokinase method

A

C. Add 5.0 mg of glucose to 1.0 mL of a serum of
known concentration and measure

561
Q

Which of two instruments can be assumed to have
the narrower bandpass? Assume that wavelength is
accurately calibrated.

A. The instrument giving the highest absorbance for
a solution of 0.1 mmol/L NADH at 340 nm
B. The instrument giving the lowest %T for a
solution of nickel sulfate at 700 nm
C. The instrument giving the highest %T reading
for 1.0% v/v HCl at 350 nm
D. The instrument giving the most linear plot of
absorbance versus concentration

A

A. The instrument giving the highest absorbance for
a solution of 0.1 mmol/L NADH at 340 nm

562
Q

Which of two instruments can be assumed to have
the narrower bandpass? Assume that wavelength is
accurately calibrated.

A. The instrument giving the highest absorbance for
a solution of 0.1 mmol/L NADH at 340 nm
B. The instrument giving the lowest %T for a
solution of nickel sulfate at 700 nm
C. The instrument giving the highest %T reading
for 1.0% v/v HCl at 350 nm
D. The instrument giving the most linear plot of
absorbance versus concentration

A

D. The instrument giving the most linear plot of
absorbance versus concentration

563
Q

SITUATION: A 22S QC error occurs for serum
calcium by atomic absorption. Fresh standards
prepared in 5.0% w/v albumin are found to be
linear, but repeating the controls with fresh
material does not improve the QC results. Select
the most likely cause of this problem.

A. Matrix effect caused by a viscosity difference
between the standards and QC sera
B. Chemical interference caused incomplete
atomization
C. Incomplete deconjugation of protein-bound
calcium
D. Ionization interference caused by excessive heat

A

B. Chemical interference caused incomplete
atomization

564
Q

SITUATION: A serum osmolality measured in
the emergency department is 326 mOsm/kg.
Two hours later, chemistry results are:
Na = 135 mmol/L
BUN = 18 mg/dL
glucose = 72 mg/dL measured osmolality = 318 mOsm/kg

What do these results suggest?

A. Laboratory error in electrolyte or glucose
measurement
B. Drug or alcohol intoxication
C. Specimen misidentification
D. Successful rehydration of the patient

A

B. Drug or alcohol intoxication

565
Q

When calibrating a pH meter, unstable readings
occur for both pH 7.00 and 4.00 calibrators,
although both can be set to within 0.1 pH unit.
Select the most appropriate course of action.

A. Measure the pH of the sample and report to the
nearest 0.1 pH
B. Replace both calibrators with unopened buffers
and recalibrate
C. Examine the reference electrode junction for salt
crystals
D. Move the electrodes to another pH meter and
calibrate

A

C. Examine the reference electrode junction for salt
crystals

566
Q

A method calls for extracting an acidic drug from
urine with an anion exchange column. The pKa
of the drug is 6.5. Extraction is enhanced by
adjusting the sample pH to:

A. 8.5
B. 6.5
C. 5.5
D. 4.5

A

A. 8.5

567
Q

SITUATION: A patient who has a positive
urinalysis for glucose and ketones has a glycated
Hgb of 4.0%. A fasting glucose performed the
previous day was 180 mg/dL. Assuming acceptable
QC, you would:

A. Report the glycosylated Hgb
B. Request a new specimen and repeat the
glycosylated Hgb
C. Perform a Hgb electrophoresis on the sample
D. Perform a glucose measurement on the sample

A

B. Request a new specimen and repeat the
glycosylated Hgb

568
Q

Quality control results for uric acid are as follows:

     Run 1   Run 2    Run 3     Run 4           Mean           s QC1   3.5       3.8         4.1     4.2 mg/dL  3.6 mg/dL    0.40 QC2   6.8       7.2         7.4     7.5 mg/dL  7.0 mg/dL    0.25

Results should be reported from:

A. Run 1 only
B. Runs 1 and 2
C. Runs 1, 2, and 3
D. Runs 1, 2, 3, and 4

A

C. Runs 1, 2, and 3

569
Q

SITUATION: A peak blood level for orally
administered theophylline (therapeutic
range 8–20 mg/L) measured at 8 a.m. is
5.0 mg/L. The preceding trough level was
4.6 mg/L. What is the most likely explanation
of these results?

A. Laboratory error made on peak measurement
B. Specimen for peak level was collected from
wrong patient
C. Blood for peak level was drawn too soon
D. Elimination rate has reached maximum

A

C. Blood for peak level was drawn too soon

570
Q

SITUATION: A patient breathing room air has the
following arterial blood gas and electrolyte results:

pH = 7.54
PCO2 = 18.5 mm Hg
PO2 = 145 mm Hg
HCO3 = 18 mmol/L
Na = 135 mmol/L
K = 4.6 mmol/L
Cl = 98 mmol/L
TCO2 = 20 mmol/L

The best explanation for these results is:

A. Blood for electrolytes was drawn above an IV
B. Serum sample was hemolyzed
C. Venous blood was sampled for arterial blood
gases
D. Blood gas sample was exposed to air

A

D. Blood gas sample was exposed to air

571
Q

SITUATION: The following lab results are
reported. Which result is most likely to be
erroneous?

Arterial blood gases:
pH = 7.42
pO2 = 90 mm Hg
pCO2 = 38.0 mm Hg bicarbonate = 24 mmol/L.
Plasma electrolytes:
Na = 135 mmol/L
Cl = 98 mmol/L
K = 4.6 mmol/L
TCO2 = 33 mmol/L

A. pH
B. Na
C. K
D. TCO2

A

D. TCO2

572
Q

SITUATION: Laboratory results on a patient from
the emergency department are:

glucose = 1,100 mg/dL
Na = 155 mmol/L
K = 1.2 mmol/L
Cl = 115 mmol/L
TCO2 = 3.0 mmol/L

What is the most likely explanation of these
results?

A. Sample drawn above an IV
B. Metabolic acidosis with increased anion gap
C. Diabetic ketoacidosis
D. Laboratory error measuring electrolytes caused
by hyperglycemia

A

A. Sample drawn above an IV

573
Q

SITUATION: A plasma sample from a person in a
coma as a result of an automobile accident gave the
following results:

Total CK 480 IU/L
CK-MB 8 μg/L
Myoglobin 800 μg/L
Troponin I 0.02 μg/L

What is the best interpretation of these results?

A. The person had a heart attack that caused the
accident
B. The accident caused traumatic injury, but no
heart attack occurred
C. A heart attack occurred in addition to a stroke
D. It is not possible to tell whether a heart attack
occurred because of the extensive trauma

A

B. The accident caused traumatic injury, but no
heart attack occurred

574
Q

SITUATION: A patient has the following
electrolyte results:

Na = 130 mmol/L
K = 4.8 mmol/L
Cl = 105 mmol/L
TCO2 = 26 mmol/L

Assuming acceptable QC, select the best course
of action.

A. Report these results
B. Check the albumin, total protein, Ca, P, and
Mg results; if normal, repeat the sodium test
C. Request a new sample
D. Recalibrate and repeat the potassium test

A

B. Check the albumin, total protein, Ca, P, and
Mg results; if normal, repeat the sodium test

575
Q

A stat plasma lithium determined using an
ion-selective electrode is measured at
14.0 mmol/L. Select the most appropriate
course of action.

A. Immediately report this result
B. Check sample for hemolysis
C. Call for a new specimen
D. Rerun the lithium calibrators

A

C. Call for a new specimen

576
Q

A chromatogram for blood alcohol (GC) gives
broad trailing peaks and increased retention times
for ethanol and internal standard. This is most
likely caused by:

A. A contaminated injection syringe
B. Water contamination of the column packing
C. Carrier gas flow rate that is too fast
D. Oven temperature that is too high

A

B. Water contamination of the column packing

577
Q

SITUATION: An amylase result is 550 U/L. A
1:4 dilution of the specimen in NaCl gives
180 U/L (before mathematical correction for
dilution). The dilution is repeated with the same
results. The technologist should:

A. Report the amylase as 550 U/L
B. Report the amylase as 720 U/L
C. Report the amylase as 900 U/L
D. Dilute the sample 1:10 in distilled water
and repeat

A

B. Report the amylase as 720 U/L

578
Q

SITUATION: A patient’s biochemistry results are:

ALT = 55 IU/L
AST = 165 IU/L
glucose = 87 mg/dL
LD = 340 IU/L
Na = 142 mmol/L
K = 6.8 mmol/L
Ca = 8.4 mg/dL
Pi = 7.2 mg/dL

Select the best course of action.

A. Report results along with an estimate of the
degree of hemolysis
B. Repeat LD but report all other results
C. Request a new sample
D. Dilute the serum 1:2 and repeat AST and LD

A

A. Report results along with an estimate of the
degree of hemolysis

579
Q

A blood sample is left on a phlebotomy tray for
4.5 hours before it is delivered to the laboratory.
Which group of tests could be performed?

A. Glucose, Na, K, Cl, TCO2
B. Uric acid, BUN, creatinine
C. Total and direct bilirubin
D. CK, ALT, ALP, AST

A

B. Uric acid, BUN, creatinine

580
Q

An HPLC assay for procainamide gives an
internal standard peak that is 15% greater in area
and height for sample 1 than sample 2. The
technologist should suspect that:

A. The column pressure increased while sample 2
was being analyzed
B. Less recovery from sample 2 occurred in the
extraction step
C. The pH of the mobile phase increased during
chromatography of sample 2
D. There was more procainamide in sample 1 than
sample 2

A

B. Less recovery from sample 2 occurred in the
extraction step

581
Q

After staining a silica gel plate to determine the
L/S ratio, the technologist notes that the lipid
standards both migrated 1 cm faster than usual.
The technologist should:

A. Repeat the separation on a new silica gel plate
B. Check the pH of the developing solvent
C. Prepare fresh developing solvent and repeat
the assay
D. Reduce solvent migration time for all
subsequent runs

A

C. Prepare fresh developing solvent and repeat
the assay

582
Q

A quantitative urine glucose was determined to
be 160 mg/dL by the Trinder glucose oxidase
method. The sample was refrigerated overnight.
The next day, the glucose is repeated and found to
be 240 mg/dL using a polarographic method.
What is the most likely cause of this discrepancy?

A. Poor precision when performing one of the
methods
B. Contamination resulting from overnight storage
C. High levels of reducing substances interfering
with the Trinder reaction
D. Positive interference in the polarographic
method caused by hematuria

A

C. High levels of reducing substances interfering
with the Trinder reaction

583
Q

SITUATION: Results of an iron profile are:

serum Fe = 40 μg/dL
TIBC = 400 μg/dL
ferritin = 40μg/L
transferrin = 300 mg/dL
(reference range 15–200)

These results indicate:

A. Error in calculation of TIBC
B. Serum iron falls before ferritin in iron deficiency
C. A defect in iron transport and not Fe deficiency
D. Excess release of ferritin caused by injury

A

D. Excess release of ferritin caused by injury

584
Q

SITUATION: Results of an iron profile are:

Serum Fe = 40 μg/dL
TIBC = 400 μg/dL
ferritin = 50 μg/L

All of the following tests are useful in establishing
a diagnosis of Fe deficiency except:.

A. Protein electrophoresis
B. Erythrocyte zinc protoporphyrin
C. Serum transferrin
D. Hgb electrophoresis

A

D. Hgb electrophoresis

585
Q

Serum protein and immunofixation electrophoresis
are ordered on a patient. The former is performed,
but there is no evidence of a monoclonal protein.
Select the best course of action.

A. Perform quantitative Ig G, A, M
B. Perform the IFE on the serum
C. Report the result; request a urine sample for
protein electrophoresis
D. Perform IFE on the serum and request a urine
sample for IFE

A

C. Report the result; request a urine sample for
protein electrophoresis

586
Q

SITUATION: Hgb electrophoresis is performed
and all of the Hgbs have greater anodal mobility
than usual. A fast Hgb (Hgb H) is at the edge of
the gel and bands are blurred. The voltage is set
correctly, but the current reading on the ammeter
is too low. Select the course of action that would
correct this problem.

A. Reduce the voltage
B. Dilute the buffer and adjust the pH
C. Prepare fresh buffer and repeat the test
D. Reduce the running time

A

C. Prepare fresh buffer and repeat the test

587
Q

A technologist is asked to use the serum from a
clot tube left over from a chemistry profile run at
8 a.m. for a stat ionized calcium (Cai) at 11 a.m.
The technologist should:

A. Perform the assay on the 8 a.m. sample
B. Perform the test only if the serum container was
tightly capped
C. Perform the assay on the 8 a.m. sample only if it
was refrigerated
D. Request a new sample

A

D. Request a new sample

588
Q

SITUATION: A patient’s biochemistry results are:
Na = 125 mmol/L
Cl = 106 mmol/L
K = 4.5 mmol/L
TCO2 = 19 mmol/L
chol = 240 mg/dL
triglyceride = 640 mg/dL
glucose = 107 mg/dL
AST = 16 IU/L
ALT = 11 IU/L
amylase = 200 U/L

Select the most likely cause of these results.

A. The sample is hemolyzed
B. Serum was not separated from cells in
sufficient time
C. Lipemia is causing in vitro interference
D. The specimen is contaminated

A

C. Lipemia is causing in vitro interference

589
Q

A gastric fluid from a patient suspected of having
taken an overdose of amphetamine is sent to the
laboratory for analysis. The technologist should:

A. Perform an EMIT assay for amphetamine
B. Refuse the sample and request serum or urine
C. Dilute 1:10 with H2O and filter; perform TLC
for amphetamines
D. Titrate to pH 7.0, then follow procedure for
measuring amphetamine in urine

A

C. Dilute 1:10 with H2O and filter; perform TLC
for amphetamines

590
Q

SITUATION: Results of biochemistry tests are:

Na = 138 mmol/L
K = 4.2 mmol/L
Cl = 94 mmol/L
TCO2 = 20 mmol/L
glucose = 100 mg/dL
T bili = 1.2 mg/dL
BUN = 6.8 mg/dL
creat = 1.0 mg/dL
albumin = 4.8 g/dL
T protein = 5.1 g/dL

What should be done next?

A. Request a new specimen
B. Repeat the total protein
C. Repeat all tests
D. Perform a protein electrophoresis

A

B. Repeat the total protein

591
Q

The following chart compares the monthly total
bilirubin mean of Laboratory A to the monthly
mean of Laboratory B, which uses the same
control materials, analyzer, and method.

Level 1 Control
Mean CV
Lab A 1.1 mg/dL 2.1%
Lab B 1.4 mg/dL 2.2%

Level 2 Control
Mean CV
Lab A 6.7 mg/dL 3.2%
Lab B 7.0 mg/dL 3.6%

Both laboratories performed controls at the
beginning of each shift using commercially
prepared liquid QC serum stored at –20°C. Which
of the following conditions would explain these
differences?

A. Improper handling of the control material by
Laboratory A resulted in loss of bilirubin due to
photodegradation
B. The laboratories used a different source of
bilirubin calibrator
C. Laboratory B obtained higher results because its
precision was poorer
D. Carryover from another reagent falsely elevated
the results of Laboratory B

A

B. The laboratories used a different source of
bilirubin calibrator

592
Q

After installing a new analyzer and reviewing
the results of patients for 1 month, the lead
technologist notices a greater frequency of patients
with abnormally high triglyceride results. Analysis
of all chemistry profiles run the next day indicated
that triglyceride results are abnormal whenever the
test is run immediately after any sample that is
measured for lipase. These observations point to
which type of error?

A. Specificity of the triglyceride reagents
B. Precision in pipetting of lipemic samples
C. Bias caused by sequence of analysis
D. Reagent carryover

A

D. Reagent carryover

593
Q

SITUATION: A digoxin result from a stable
patient with a normal electrocardiogram (EKG)
is reported as 7.4 ng/mL (URL 2.6 ng/mL) using
an immunofluorescent method. Renal function
tests were normal and the patient was not taking
any other medications. The assay was repeated
and results were the same. The sample was
frozen and sent to a reference laboratory for
confirmation. The result was 1.6 ng/mL measured
by a competitive chemiluminescent procedure.
Which best explains the discrepancy in results?

A. The fluorescent immunoassay was performed
improperly
B. Digoxin was lower by the chemiluminescent
method because it is less sensitive
C. An interfering substance was present that
cross-reacted with the antibody in the
fluorescent immunoassay
D. Freezing the specimen caused lower results by
converting the digoxin to an inactive metabolite

A

C. An interfering substance was present that
cross-reacted with the antibody in the
fluorescent immunoassay

594
Q

The following results are reported on an adult
male patient being evaluated for chest pain:

                       Myoglobin                   Troponin I                        CK-MB
                 (Cutoff = 100 μg/L)      (Cutoff = 0.03 μg/L)     (Cutoff = 4 μg/L) Admission          12 μg/L                         1.1 μg/L                        18 μg/L 3 hours              360 μg/L                        1.8 μg/L                       26 μg/L postadmission   6 hours              300 μg/L                        2.4 μg/L                       40 μg/L postadmission

What is the most likely cause of these results?

A. The wrong sample was assayed for the
first myoglobin
B. The patient did not suffer an MI until after
admission
C. Hemolysis caused interference with the 3-hour
and 6-hour myoglobin result
D. The patient is experiencing unstable angina

A

A. The wrong sample was assayed for the
first myoglobin

595
Q

Analysis of normal and abnormal QCs performed
at the beginning of the evening shift revealed a
22s error across levels for triglyceride. Both
controls were within the 3s limit. The controls
were assayed again, and one control was within
the acceptable range and the other was slightly
above the 2s limit. No further action was taken
and the patient results that were part of the run
were reported. Which statement best describes
this situation?

A. Appropriate operating procedures were followed
B. Remedial evaluation should have been taken,
but otherwise, the actions were appropriate
C. Corrective action should have been taken before
the controls were repeated
D. The controls should have been run twice before
reporting results

A

C. Corrective action should have been taken before
the controls were repeated

596
Q

A biochemical profile routinely performed
bimonthly on a renal dialysis patient showed a
decreased serum calcium and decreased PTH level.
Such a lab result may be explained by which of
the following circumstances?

A. Malignancy
B. Aluminum toxicity
C. Hypervitaminosis D
D. Acidosis

A

B. Aluminum toxicity

597
Q

Which set of the following laboratory results is
most likely from a patient who has suffered an
AMI? Reference intervals are in parenthesis.

Total CK CK-MB CK index
(10–110 U/L) (1–4 μg/L) (1%–2.5%)
A. 760 U/L 16 μg/L 2.1%
B. 170 U/L 14 μg/L 8.2%
C. 160 U/L 4 μg/L 2.5%
D. 80 U/L 2 μg/L 2.5%

A

B

598
Q

Hemoglobin electrophoresis performed on agarose
at pH 8.8 gives the following results:

A2 Position S Position F Position A Position
35% 30% 5% 30%

All components of the Hgb C, S, F, A control
hemolysate were within the acceptable range.
What is the most likely cause of this patient’s
result?

A. HgbLepore
B. Hgb S-β-thalassemia (Hgb S/β+)
C. Hgb SC disease post-transfusion
D. Specimen contamination

A

C. Hgb SC disease post-transfusion

599
Q

Na K Cl HCO3 BUN Glucose Creatinine Uric Acid
140 mmol/L 5.8 mmol/L 102 mmol/L 18 mmol/L 2.6 mg/dL 20 mg/dL DL DL
132 mmol/L 4.8 mmol/L 98 mmol/L 24 mmol/L DL DL DL DL
DL = Detection limit flag (absorbance below detectable limit)

Two consecutive serum samples give the results
shown in the table above for a metabolic function profile.
The instrument is a random access analyzer
that uses two sample probes. The first probe
aspirates a variable amount of serum for the
spectrophotometric chemistry tests, and the
second probe makes a 1:50 dilution of serum
for electrolyte measurements. What is the most
likely cause of these results?

A. Both patients have renal failure
B. There is an insufficient amount of sample in both
serum tubes
C. There is a fibrin strand in the probe used for the
spectrophotometric chemistry tests
D. The same patient’s sample was accidentally run
twice

A

C. There is a fibrin strand in the probe used for the
spectrophotometric chemistry tests

600
Q

SITUATION: A blood sample in a red-stoppered
tube is delivered to the laboratory for electrolytes,
calcium, and phosphorus. The tube is
approximately half full and is accompanied by a
purple-stoppered tube for a complete blood count
that is approximately three-quarters full. The
chemistry results are as follows:

Na = 135 mmol/L
K = 11.2 mmol/L
Cl = 103 mmol/L
HCO3 = 14 mmol/L
Ca = 2.6 mg/dL
InP = 3.8 mg/dL

What is the most likely explanation of these serum
calcium results?

A. Severe hemolysis during sample collection
B. Laboratory error in the calcium measurement
C. The wrong order of draw was used for vacuum
tube collection
D. Some anticoagulated blood was added to the
red-stoppered tube

A

D. Some anticoagulated blood was added to the
red-stoppered tube

601
Q

SITUATION: A patient previously diagnosed with
primary hypothyroidism and started on thyroxine
replacement therapy is seen for follow-up testing
after 2 weeks. The serum-free T4 is normal but the
TSH is still elevated. What is the most likely
explanation for these results?

A. Laboratory error in measurement of free T4
B. Laboratory error in measurement of TSH
C. In vitro drug interference with the free T4 assay
D. Results are consistent with a euthyroid patient in
the early phase of therapy

A

D. Results are consistent with a euthyroid patient in
the early phase of therapy

602
Q

SITUATION: A 6-year-old child being treated
with phenytoin was recently placed on valproic
acid for better control of seizures. After displaying
signs of phenytoin toxicity including ataxia,
a stat phenytoin is determined to be 15.0 mg/L
(reference range 10–20 mg/L). A peak blood level
drawn 5 hours after the last dose is 18.0 mg/L.
The valproic acid measured at the same time is
within therapeutic limits. Quality control is within
acceptable limits for all tests, but the physician
questions the accuracy of the results. What is the
most appropriate next course of action?

A. Repeat the valproic acid level using the last
specimen
B. Repeat the phenytoin on both trough and peak
samples using a different method
C. Recommend measurement of free phenytoin
using the last specimen
D. Recommend a second trough level be measured

A

C. Recommend measurement of free phenytoin
using the last specimen

603
Q

Na K Cl HCO3 BUN Glucose Creatinine Uric Acid
140 mmol/L 3.6 mmol/L 100 mmol/L 28 mmol/L 130 mg/dL 110 mg/dL 1.2 mg/dL 4.8 mg/dL
148 mmol/L 4.2 mmol/L 110 mmol/L 24 mmol/L 135 mg/dL 86 mg/dL 0.8 mg/dL 3.9 mg/dL
138 mmol/L 4.0 mmol/L 105 mmol/L 22 mmol/L 142 mg/dL 190 mg/dL 1.0 mg/dL 4.6 mg/dL

The results shown in the table above are
obtained from three consecutive serum samples
using an automated random access analyzer
that samples directly from a bar-coded tube.
Calibration and QC performed at the start of
the shift are within the acceptable range, and no
error codes are reported by the analyzer for
any tests on the three samples. Upon results
verification, what is the most appropriate course
of action?

A. Report the results and proceed with other tests
since no analytical problems are noted
B. Repeat the controls before continuing with
further testing, but report the results
C. Check sample identification prior to reporting
D. Do not report BUN results for these patients or
continue BUN testing

A

D. Do not report BUN results for these patients or
continue BUN testing

604
Q

An AFP measured on a 30-year-old pregnant
woman at approximately 12 weeks gestation is
2.5 multiples of the median (MOM). What
course of action is most appropriate?

A. Repeat the serum AFP in 2 weeks
B. Recommend AFP assay on amniotic fluid
C. Repeat the AFP using the same sample by
another method
D. Repeat the AFP using the sample by the same
method

A

A. Repeat the serum AFP in 2 weeks

605
Q

AST ALT ALP LD CK GGT TP ALB TBIL GLU TG CA InP BUN
U/L U/L U/L U/L U/L U/L g/dL g/dL mg/dL mg/dL mg/dL mg/dL mg/dL mg/dL
D1 20 15 40 100 15 40 8.2 3.6 0.8 84 140 8.7 4.2 16
D2 22 14 65 90 20 36 8.3 3.8 1.0 128 190 8.8 5.2 26

SITUATION: Biochemistry tests are performed
24 hours apart on a patient and delta-check
flag is reported for inorganic phosphorus by
the laboratory information system. Given the
results shown in the table above, identify the
most likely cause.

A. Results suggest altered metabolic status caused by
poor insulin control
B. The patient was not fasting when the sample was
collected on day 2
C. The samples were drawn from two different
patients
D. The delta-check limit is invalid when samples are
collected 24 or more hours apart

A

B. The patient was not fasting when the sample was
collected on day 2

606
Q

A quantitative sandwich enzyme immunoassay for
intact serum hCG was performed on week 4 and
the result was 40,000 mIU/mL (reference range
10,000–80,000 mIU/mL). The physician
suspected a molar pregnancy and requested that
the laboratory repeat the test checking for the
hook effect. Which process would identify this
problem?

A. Obtain a new plasma specimen and heat
inactivate before testing
B. Obtain a urine specimen and perform the assay
C. Perform a qualitative pregnancy test
D. Perform a serial dilution of the sample and repeat
the test

A

D. Perform a serial dilution of the sample and repeat
the test

607
Q

A patient presents to the emergency department
with symptoms of intoxication including impaired
speech and movement. The plasma osmolality was
measured and found to be 330 mOs/kg. The
osmolal gap was 40 mOsm/Kg. A blood alcohol
was measured by the alcohol dehydrogenase
method and found to be 0.15% w/v (150 mg/dL).
Electrolyte results showed an increased anion gap.
Ethylene glycol intoxication was suspected because
the osmolal gap was greater than could be explained
by ethanol alone, but gas chromatography was not
available. Which of the following would be
abnormal if this suspicion proved correct?

A. Arterial blood gases
B. Lactic acid
C. Urinary ketones
D. Glucose

A

A. Arterial blood gases

608
Q

Serial TnI assays are ordered on a patient at
admission, 3 hours, and 6 hours afterwards.
The samples were collected in heparinized
plasma separator tubes. Following are the results
(reference range 0–0.03 μg/L)
Admission = 0.03 μg/L
3 hours = 0.07 μg/L
6 hours = 0.02 μg/L

These results indicate:

A. A positive test for acute myocardial infarction
B. Unstable angina
C. Cardiac injury of severity less than myocardial
infarction
D. Random error with the 3-hour sample

A

C. Cardiac injury of severity less than myocardial
infarction